You are on page 1of 468

Plastic Surgery Exam Questions &

Answers
A Guide to the Plastic Surgery Exit Exam

RMJose
WYChan
J Jagadeesan
S Enoch

Forewordby:
D A Mc Grouther

2
Plastic Surgery Exam Questions &
Answers
A Guide to the Plastic Surgery Exit Exam

Authors
Rajive Mathew Jose MBBS, MS, DNB (Gen Surg), MCh (Plast), FRCS (Plast), Dip Hand Surg
(BSSH)
Consultant Plastic and Reconstructive Surgeon
Queen Elizabeth Hospital and University Hospital of Birmingham
Birmingham, United Kingdom
Woan-Yi Chan MSc, MRCS, MEd
Academic Clinical Fellow Speciality Registrar Plastic Surgery and MedicalEducation
The South West Peninsula Deanery
Plymouth, United Kingdom
Jagajeevan Jagadeesan MBBS, MSc, MRCS
Specialist Registrar in Plastic and Reconstructive Surgery
Queen Elizabeth Hospital and University Hospital of Birmingham
Birmingham, United Kingdom
Stuart Enoch MBBS, MRCSEd, PGCert (Med Sci), MRCS (Eng), PhD
Programme Director, Doctors Academy Group, Cardiff, United Kingdom
Clinical Director - Centre for Studies in Wound Care and Burns;
Visiting Professor, Dept. of Biomedical Research, Noorul Islam University, India

Foreword by:
Professor Duncan A McGrouther MBChB, MD (Hons), MSc, FRCS (Glas),FRCSEd, FRCS (Eng)
Professor of Plastic & Reconstructive Surgery, University Hospitals of Manchester
Clinical Director – Manchester Integrating Medicine and Innovative Technology
University of Manchester, United Kingdom

3
Preface to 1st Edition
While preparing for the FRCS Plastic Surgery exam, one is surprised about
the paucity of books to practice focused, exam-oriented questions with
appropriate explanation for the answers. Although a comprehensive
understanding of the essential principles in Plastic Surgery can only be
obtained by reading established textbooks, in the lead-up to the exam it is
vital to recall and test the acquired knowledge by solving sample exam
questions. This is particularly important for the first part of the FRCS Plastic
Surgery exam, in which candidates are tested by means of Single Best
Answer and Extended Matching Question formats.
Plastic Surgery Exam Questions and Answers is written with the specific
aim of helping candidates preparing for the Plastic Surgical exit exam. Each
question is followed by a correct answer and an informative but concise
explanation. With over 350 questions, both in Single Best Answer and
Extended Matching Question formats, we hope this book will be a valuable
companion for those preparing for the FRCS Plastic Surgery examination in
the United Kingdom as well as those taking similar examinations elsewherein
the world. In addition, with the proposed re certification process, whichmay
include a theoretical component, it is envisaged that this book mayalso serve
as a useful resource for those already qualified.
Although we have attempted to cover most of the syllabus for the FRCS
Plastic Surgery examination, due to the wide range of conditions and
disorders covered by this speciality, it needs to be acknowledged that
covering every topic in-depth is beyond the scope of this book. It is hence
suggested that this book is used in conjunction with time-honoured Plastic
Surgical textbooks and used as a complementary resource rather than to
supplement existing text books.

RMJ
WY
CSE

4
Preface to 2nd Edition
Three years ago we published the first edition of this book. The feedbac k
and response from readers have been very positive and encouraging. This
has given us the impetus to bring forth the second edition.
Medicine is a continually evolving subject and exam revisions books have to
keep pace with the advances in the specialty as well as the changes in the
exam structure. We have thus included questions related to new
developments in plastic surgery and have expanded on all sections with
additional SBAs and EMQs. There are also separate sections on Ethics and
Statistics, which are particularly relevant to the FRCS(Plast) exam in the UK.
We would like to thank all trainees and colleagues who have given their
constructive comments and criticism to the first edition. We have indeed
borne this in mind while preparing the updated and improved version. We
sincerely hope that this book proves useful to trainees in Plastic Surgery in
their academic pursuits.
-RMJ
- WYC
- JJ
- SE

5
Foreword
When you approach multiple-choice examination in Plastic Surgery, if you know little or nothing and rely
on guess-work you can’t succeed; if you have read widely then you should pass, but if you have very
extensive experience you can become confused by terms such as ‘commonly’ or ‘never’.
This innovative text tackles all of these knowledge levels by following the multiple-choice and extended
matching questions with an explanation of the subject matter. The uninitiated will learn, the widelyread
should pass and the widely but selectively experienced reader will derive a perspective from the
explanation.
The text is nicely set-out, generally providing the answer on the same page, but through some optical
illusion, it does not catch the eye until the question has been answered. The experience of single best
answer and extended matching question format should prepare the candidate for examination in not
only the UK but in many other countries with a similar style of examination.
The specific aim is to ‘help candidates prepare’ for examination and, apart from a knowledge test, it
reminds the reader of how enormously important it is to read the question. In the heat of the
examination it is so easy to gloss over such adjectives as ‘accurately’ or ‘commonly’ and even for the
widely read the discipline of running through multiple choice questions before an examination is an
invaluable preparation for exam conditions.
I am able to practice Plastic Surgery without remembering all of the classifications in this book but
perhaps that makes me a poor teacher and highlights my own need for CPD.

Professor Duncan A McGrouther


MBChB, MD (Hons), MSc, FRCS (Glas), FRCSEd, FRCS (Eng)
Professor of Plastic & Reconstructive Surgery, University Hospitals of Manchester
Clinical Director – Manchester Integrating Medicine and Innovative Technology
University of Manchester, United Kingdom

6
Abbreviations
A&E : Accident and Emergency
AIDS : Acquired Immuno Deficiency Syndrome
AJCC : American Joint Committee on Cancer
AK : Actinic Keratosis
ALT : Antero-Lateral Thigh (flap)
ANOVA : ANalysis Of Variance
AR : ARTICULARE
ATLS : Advanced Trauma Life Support
BAPRAS : British Association of Plastic Reconstructive and Aesthetic
Surgeons
BCC : Basal Cell Carcinoma
BMI : Body Mass Index
BMPs : Bone Morphogenic Proteins
BRCA : BReast CAncer Susceptibility Gene
CCT : Certificate of Completion of Training
CD4 : Cluster of Differentiation 4
CESR : Certificate of Eligibility for Specialist Registration
CMCJ : Carpometacarpal Joint
CN : Cranial Nerve
COHb : Carboxyhaemoglobin
CRPS : Complex Regional Pain Syndrome
CT : Computerised Tomography
DCIS : Ductal Carcinoma In-Situ
DFSP : Dermatofibrosarcoma Protuberans
DIEA : Deep Inferior Epigastric Artery
DIEP(DIEAP) : Deep Inferior Epigastric Artery Perforator (flap)
DNA : Deoxyribonucleic acid
DRUJ : Distal Radio-Ulnar Joint
EBM : Evidence-based medicine
ECG : Electrocardiogram
ECM : Extracellular Matrix
ECRB : Extensor Carpi Radialis Brevis
ECRL : Extensor Carpi Radialis Longus
EDC : Extensor Digitorum Communis

7
EIP : Extensor Indicis Proprius
EMLA : Eutectic Mixture of Local Anaesthetics
EMSB : Emergency Management of Severe Burns
EPB : Extensor Pollicis Brevis
EPL : Extensor Pollicis Longus
FCR : Flexor Carpi Radialis
FCU : Flexor Carpi Ulnaris
FDG : Fluoro-Deoxyglucose
FDP : Flexor Digitorum Profundus
FDS : Flexor Digitorum Superficialis
FGFR : Fibroblast Growth Factor Receptor Flap
FPL : Flexor Pollicis Longus
FU : Fluorouracil
GMC : General Medical Council
GN : GNATHION
GO : GONION
H&N : Head & Neck
HIV : Human Immunodeficiency Virus
HLA-D: Human Leucocyte Antigen-D
HLA-DR4: Human Leucocyte Antigen-DR4
IGF: Insulin-like Growth Factor
IL : Interleukin
IMF : Inframammary Fold
IPL : Intense Pulsed Light
ISB : Intercollegiate Specialty Boards
ISCP : Intercollegiate Surgical Curriculum Project
JCHST : Joint Committee on Higher Surgical Training
JCST : Joint Committee on Surgical Training
JRA : Juvenile Rheumatoid Arthritis
KA : Keratoacanthoma
KTP Laser : Potassium Titanyl Phosphate Laser
LA : Local Anaesthetic
LCIS : Lobular Carcinoma-In-Situ
LD : Lethal Dose
MCPJ : Metacarpophalangeal Joint
ME : MENTON
MHC : Major Histocompatibility Complex

8
MMC : Modernizing Medical Careers
MMP : Matrix Metalloproteinases
MRC : Medical Research Council
MRI : Magnetic Resonance Imaging
MRSA : Methicillin Resistant Staphylococcus Aureus
MU : Mouse Units
N : NASION
NAC : Nipple-Areola Complex
NCEPOD :National Confidential Enquiry into Patient Outcome and Deaths /
National Confidential Enquiry into Perioperative Deaths
Nd: YAG Laser : Neodymium:Yttrium-Aluminum-Garnet Laser
NSAID : Non-Steroidal Anti Inflammatory Drug
ODP : Operating department practitioner
OR : ORBITALE
PDS : Polydioxanone
PET : Positron Emission Tomography
PG : POGONION
PIPJ : Proximal Interphalangeal Joint
PMETB : Postgraduate Medical Education Training Board
PO : PORION
PT : Pronator Teres
PTFE : Polytetrafluoroethylene
RA : Rheumatoid Arthiritis
ROOF :Retro-Orbicularis Oculi Fat
RTA : Road Traffic Accident
SAC : Specialist Advisory Committee
SCC : Squamous Cell Carcinoma
SCIA : Superficial Circumflex Iliac Artery
SD : SUPRADENTALE
SGAP : Superior Gluteal Artery Perforator
SJS : Steven-Johnson Syndrome
SLE : Systemic Lupus Erythematosis
SMAS : Superficial Musculo-Aponeurotic System
SNLB : Sentinel Lymph Node Biopsy
SOOF : Sub-Orbicularis Oculi Fat
SPAIR : Short-Scar Periareolar Inferior Pedicle Reduction
SSG : Split Skin Graft

9
TAR : Thromobocytopenia Absent Radius
TBSA : Total Body Surface Area
TEN : Toxic Epidermal Necrolysis
TIMP : Tissue Inhibitors of Matrix Metalloproteinases
TNF : Tumour Necrosis Factor
TNM : Tumour Node Metastasis
TRAM : Transverse Rectus Abdominus Muscle/Myocutaneous (flap)
UV : Ultra Violet
VAC : Vacuum Assisted Closure
VATER/VACTERL : Vertebral, Anal, Cardiac, Tracheal, Oesophageal, Renal
and Limb Anomalies
VEGF : Vascular Endothelial Growth Factor
VITAMIN : Venous, Infectious, Traumatic, Arterial, Metabolic, Inflammatory,
Neoplastic and Neuropathic
VPI : Velopharyngeal Incompetence
WHO : World Health Organization
ZPA : Zone of Polarising Activity

10
Contents
Aesthetic Surgery
Burns
Ethics and Consent
Fundamental Principles in Plastic Surgery
Hand and Upper Extremity
Head and Neck
Paediatric Plastic Surgery
Skin and Soft Tissues
Statistics
Trauma
Miscellaneous
Bibliography

11
Aesthetic Surgery

Single Best Answers


1) Which of the following is true regarding the blood supply to the
face?
A) The blood supply to the upper eyelid is primarily from branches of the
facial artery
B) The submental artery is a branch of the sublingual artery
C) The buccal artery arises from the facial artery at the insertion of the
buccinator muscle
D) The nasal tip is supplied by the angular artery
E) The maxillary artery arises from the internal carotid artery within the
parotid gland
Ans: See end of page
Explanation
The nasal tip receives its blood supply from the angular artery, a branch of
the facial artery. There are rich anastomoses from branches of the internal
and external carotid arteries that perfuse the periorbital areas. The lower
eyelid is supplied predominantly by branches of the facial artery whereas
the upper eyelid is supplied by branches of the ophthalmic artery.
Near the angle of the mouth the facial artery gives off the superior and
inferior labial arteries and the submental artery. The lingual artery arises
from the external carotid artery and passes deep to the hyoglossus into the
substance of the tongue. The buccal artery is a terminal branch of the
maxillary artery, which arises from the external carotid artery within the
parotid gland posterior to the neck of the mandible.
Ans: D

12
2) Which of the following statement is true regarding nasal anatomy?

A) The nasal valve corresponds to the medial crus of the lower lateral
cartilage
B) The soft triangle is the area of the nasolabial angle
C) The nose is divided into nine aesthetic units
D) Alar collapse results in nasal obstruction due to collapse of the internal
nasal valve
E) The nasion represents a soft-tissue landmark of the deepest point of the
nasofrontal angle
Ans: See end of page
Explanation
The nasal valve is the area inside the nose between the caudal edge of the
upper lateral cartilage and the nasal septum, measuring 10-15º. Also termed
the internal nasal valve, it is an important anatomical site for nasal
obstruction. The external nasal valve is the area involving the alar sidewalls,
the colummellar sidewalls, the caudal septum and the soft tissue around the
piriform aperture. Weak sidewalls of the ala collapse with inspiration
resulting in nasal obstruction.
The soft triangle is the apex of the nostrils beneath the lobules. The nose
can be divided into nine aesthetic subunits comprising of soft triangle, alae
and sidewalls on each side, dorsum, tip and columella. The nasion is a bony
landmark at the nasofrontal suture. The sellion is the deepest soft-tissue
landmark of the nasofrontal angle. The glabella is the frontal prominence
between the brows above the root of the nose.
Ans: C

13
3) What is the number of aesthetic subunits into which the nose can
be divided?

A) Five
B) Six
C) Seven
D) Eight
E) Nine
Ans: See end of page
Explanation
The concept of subunits is important to achieve a good aesthetic and
functional result in nasal reconstruction. The cover, lining and support are
pertinent structures that give the appearance of a normal nose. The
reproduction of contours and landmarks gives the required appearance of
the normal nose. The nasal surface is made up of several concave and
convex surfaces. The nose can be divided into nine subunits, comprising of
the dorsum, tip, columella and the paired lateral nasal wall units, the paired
alar units and paired soft tissue triangles. In reconstructing parts of the
nose, it is important to place the border scars in such manner that would
reflect lines of light and cast linear shadows. When a large part of a subunit
is lost, replacing the entire subunit often gives a superior result rather than
simply patching the defect. Furthermore, the various subunits have different
skin qualities. Over the bone, the skin is more mobile, whereas over the alar
cartilages the skin is fixed deeply. Sebaceous activity normally increases
from the bridge of the nose to the tip. The option of primary closure
diminishes towards the nasal tip. Reference: Burget GC, Menick FJ.
Subunit principle in nasal reconstruction. Plast Reconstr Surg 1985; 76:
239-247
Ans: E

14
4) The surface marking for great auricular nerve as it crosses the
sternocleidomastoid muscle is:
A) 9.5 cm below the external auditory canal
B) 5 cm below the external auditory canal
C) 4 cm below the external auditory canal
D) 2 cm below the external auditory canal
E) 6.5 cm below the external auditory canal
Ans: See end of page
Explanation
The great auricular nerve is a branch from the cervical plexus and is an
important structure to be preserved in facelift operations. Its surface
marking was described by McKinney and Gottlieb as follows:
“With the head turned 45°, the great auricular nerve consistently crosses the
belly of the sternocleidomastoid muscle at its midpoint some 6.5 cm below
the caudal edge of the bony external auditory canal.”
Ans: E

15
5) Pitanguy’s line for the surface marking of frontal branch of facial
nerve is a line joining:

A) A point 1 cm below the tragus to a point 1 cm above the lateral end of


eyebrow
B) A point 1 cm below the ear lobe to a point 1 cm above and lateral to
lateral end of eyebrow
C) A point 0.5 cm below the ear lobe to a point 1.5 cm lateral to lateral end
of eyebrow
D) A point 0.5 cm below the tragus to a point 1.5 cm above the lateral end
of eyebrow
E) A point 2 cm below the tragus to a point 1 cm above and lateral to lateral
end of eyebrow
Ans: See end of page
Explanation
The course of the facial nerve and its branches is important during
rhytidectomy. Identification of exact location of the frontal branch of the
facial nerve and its relation to the SMAS is crucial while dissecting the
temple.
In 1966, Pitanguy and Silveira Ramos described the frontal branch of the
facial nerve as coursing roughly along a line from 0.5 cm below the tragus to
1.5cm above the lateral end of the eyebrow. The anterior branch of the
superficial temporal artery accompanies the nerve. They also noted that the
frontal branch may course as high as 4 cm above the lateral canthus.
Ans: D

16
6) Which amongst the following statements regarding the aesthetics
of the eye is true?

A) Puffy lid results from hypertrophy of the orbicularis muscle


B) Blepharochalasis is a hereditary condition
C) Brow ptosis cannot be altered by blepharoplasty alone
D) The medial half of the brow normally lies 8 mm above the palpable
superior orbital rim
E) The orbital septum lies anterior to the orbicularis muscle
Ans: See end of page
Explanation
Puffy or baggy eyelids result from pseudoherniation due to laxity in the
orbital septum combined with continued gravitational forces on the orbital
fat. Blepharochalasis is redundancy of the eyelid skin that occurs in the aged
face. It may cause skin overhang over the upper eyelashes resulting in visual
field defects. Dermatochalasis is a hereditary condition with hypertrophy of
skin and orbicularis muscle that occurs in a young person and results in a
hooded appearance of the upper lid.
Blepharoplasty alone cannot alter brow ptosis, lateral crow’s feet and malar
bags. Brow ptosis can be corrected through a brow lift operation, either by
an open technique or endoscopically. The medial half of the eye brow
normally lies at or just above the superior orbital rim. The lateral half slants
up slightly higher above the rim. The orbital septum is a fascial condensation
from the periosteum of the orbital rim to the tarsal plate and lies just deep to
the orbicularis muscle.
Ans: C

17
7) Which of the following statements regarding rhinoplasty is true?

A) The transcartilaginous incision is made between the lower lateral and


upper lateral cartilage to gain access to the nasal dorsum
B) Transdomal sutures are used to decrease projection of the tip
C) An ‘open roof’ results from nasal bone in-fracture
D) The ideal nasolabial angle in a man is about 90 degrees
E) Nasal tip projection is measured from the root of the nose to the nasal tip
Ans: See end of page
Explanation
The transcartilaginous incision is made through the alar cartilage and is used
for the conservative volume reduction of the alar cartilage. The
intercartilaginous incision gains access to the nasal dorsum between the
lower lateral and upper lateral cartilages. Transdomal sutures are used to
create a better tip definition. An open roof is a flat and wide dorsum
resulting from a bony ‘hump reduction’ and is treated by nasal bone in-
fracture to re-establish the nasal bone pyramid.
The ideal nasolabial angle is about 90 degrees in men and about 95 to 100
degrees in women. Nasal tip projection is the distance from the nasal spine
to the nasal tip. The nasal length is measured from the nasofrontal groove to
the nasal tip.
Ans: D

18
8) Which of the following statements concerning the ageing skin is
true?

A) The proportion of type I collagen increases with age


B) Pacinian and Meissner’s corpuscles decrease in number in the aging skin
C) The skin appendages are not affected by ageing
D) Caffeine may reduce type III collagen synthesis by as much as 15%
E) Actinic skin conditions are characterised by atrophied elastic fibers in the
dermis
Ans: See end of page
Explanation
Human skin predominantly contains collagen, which comprises 70 – 80% of
dermal dry weight. In normal skin, the type I and type III collagen are
present in a ratio of about 6:1. With age, this ratio becomes smaller as the
proportion of type III collagen increases. With increasing age, sebaceous
glands tend to increase in size, although their number remains relatively
constant. Pacinian and Meissner’s corpuscles also decrease in number in
the aging skin.
Ingested caffeine in daily concentrations of approximately 10mcg/ml
(equivalent to 3 cups of coffee) may reduce type I collagen synthesis. Skin
with actinic damage is typically characterised by thickened, degraded elastic
fibers in the dermis (elastosis).
Ans: B

19
9) Which of the following statements regarding Botox® is correct?

A) It is extremely labile if frozen


B) Its activity is measured in Mouse Units
C) The effects of migration of toxin are irreversible
D) It s a patented trade name for botulinum toxin B
E) The peak onset of action after injection into muscle is at 72 hours
Ans: See end of page
Explanation
Botox® must be kept frozen until use due to its extreme heat lability. Botox®
activity is measured in Mouse Units (MU); one MU is equal to the amount
that kills 50% of a group of 18-22 grams Swiss Webster mice when
intraperitoneally. Commercially, Botox® is available as vials of 100 units of
botulinum toxin A. Botulinum toxin B (NeuroBloc®) comes in vials of 5,000
unit/mL and is used in spasmodic torticollis.
Botox® injected into striated muscle results in paralysis of the muscle with a
peak effect at 5 to 7 days after injection lasting for 2 to 6 months. It acts
through presynaptic inhibition of acetylcholine release. Inadvertent injection
of Botox® injected into ocular muscles can result in ptosis and diplopia. The
symptoms usually resolve spontaneously after 2 weeks.
Ans: B

20
10) The lethal dose of Botox® in humans is:

A) 300 Units
B) 3,000 Units
C) 30,000 Units
D) 300,000 Units
E) 3,000,000 units
Ans: See end of page
Explanation
Botulinum toxin is produced by the gram positive bacterium Clostridium
botulinum. The toxin inhibits release of acetylcholine, a neurotransmitter
responsible for activation of muscle contraction. Botulinum toxins are
currently used to treat several disorders including strabismus, hemi-facial
spasms, focal dystonias, spasticity, hyperhidrosis, achalasia and sphincter
dysfunction. The clinical effect of botulinum toxin is measured in Mouse Units
(MU). The lethal dose (LD) in humans for Botox® (botulinum toxin – type A)
is approximately 3,000 units. For most cosmetic procedures, less than 100
units will be sufficient.
Ans: B

21
11) In which amongst the following conditions can facelift be

performed?
A) Ehlers-Danlos syndrome
B) Progeria
C) Cutis laxa
D) Werner’s syndrome
E) Cutis hyperelastica
Ans: See end of page
Explanation
Cutis laxa is a disorder of elastin fibres but with a normal collagen synthesis.
Facelift and other aesthetic procedures can therefore be undertaken in
these patients.
Ehlers-Danlos (cutis hyperelastica) is characterised by thin, hyperelastic skin
and is associated with post-operative bleeding and poor wound healing.
Progeria (also known as Hutchinson-Gilford syndrome) is characterised by
growth retardation, craniofacial disproportion, alopecia, arteriosclerosis and
cardiac disease. Werner’s syndrome (adult progeria) is associated with
scleroderma-like patches in the skin, premature ageing and cardiac disease.
Due to poor long-term prognosis or significant risk of complications, facelifts
should be avoided in all the patients.
Ans: C

22
12) A 10-year-old boy who underwent bilateral pinnaplasty using
cartilage scoring technique is brought to the A&E six hours after the
procedure with complaint of increasing pain on the right side. The
most likely cause is:

A) Tight dressings
B) Haematoma
C) Wound infection
D) Cartilage necrosis
E) Great auricular nerve injury

Ans: See end of page


Explanation
One of the early compications of pinnaplasty is haematoma under the skin.
In techniques that involve dissection of the anterior skin off the cartilage,
haematoma can compromise the circulation of the skin. Increasing pain,
especially on one side, as seen in this child, should raise the suspicion of a
haematoma and necessitates prompt evacuation. If the haematoma
becomes organised, it will fill the convolutions of the cartilage and may lead
to a permanent deformity. Moreover, if the skin circulation is compromised,
it can cause skin necrosis.
Ans: B

23
13) An infiltrate to aspirate ratio of 3:1 in liposuction is known as:

A) Dry liposuction
B) Wet liposuction
C) Superwet liposuction
D) Tumescent liposuction
E) Liposculpture
Ans: See end of page
Explanation
Common practice in liposuction is to use an infiltrate, the commonest of
which is Klein’s solution (a mixture of xylocaine and adrenaline in Hartmann’s
solution). This helps to minimise the blood loss, helps to aspirate the fat and
also provides analgesia. Depending on the amount of infiltrate used
liposuction techniques are classified as Wet, Superwet and Tumescent. In
wet liposuction, the volume of infiltrate is 200-300mLper area. In super wet
technique, the ratio of infiltrate to aspirate is 1:1 and in tumescent it is 3:1.
In dry liposuction, no infiltrate is used. Liposculpture is a term used to
desribe autologous fat injection to correct contour deformities..
Ans: D

24
14) A patient is evaluated for rhinoplasty. She has nasal obstruction
on right side and is positive for Cottle’s manoeuvre. Internal nasal
valve measures 10 degrees. The best way to correct the obstruction
is:

A) Turbinectomy on the same side


B) Use of spreader grafts
C) Septoplasty
D) Submucous resection
E) Lateral osteotomy

Ans: See end of page


Explanation
Internal nasal valve is the angle between the caudal end of the upper lateral
cartilage and the septum. It is the narrowest portion of the nasal cavity and
measures between 10-15 degrees. Narrowing of the internal nasal valve
presents with nasal obstruction and can be diagnosed using Cottle’s
manoeuvre (traction on the cheek improves the nasal air flow). The
narrowing of the internal nasal valve can be corrected by using spreader
grafts, which are matchstick-shaped cartilage grafts inserted between the
upper lateral cartilage and septum.
Ans: B

25
15) Injury to which amongst the following nerves may result in
numbness along the anterolateral aspect of the thigh following an
abdominoplasty?

A) Obturator nerve
B) Genitofemoral nerve
C) Ilioinguinal nerve
D) Intermediate femoral cutaneous nerve
E) Lateral cutaneous nerve of thigh

Ans: See end of page


Explanation
The anterolateral aspect of the thigh is supplied by the lateral cutaneous
nerve of the thigh, which arises from the lumbar plexus. It enters the thigh
about 1 cm below and medial to the anterior superior iliac spine. This nerve
is at risk during an abdominoplasty, particularly in low incisions; although
uncommon, one series has reported an incidence as high as 10%.
The overall complication rate for abdominoplasty is approximately 30% but
most of them are minor and related to wound healing problems.
Complications are increased in obese patients and smokers.
Ans: E

26
16) The Mustarde technique for pinnaplasty includes:

A) Sutures from concha to mastoid


B) Scoring of the cartilage on the anterior aspect
C) Stutures from the scaphoid fossa to concha to create an antihelix
D) Scoring of the cartilage on the posterior aspect
E) Excision of a crescent of conchal cartilage
Ans: See end of page
Explanation
J.C. Mustarde first described a technique in pinnaplasty in 1963 where he
used silk sutures from scaphoid fossa to concha to create an antihelical fold.
However, this technique fell into disrepute due to suture related
complications. More recently, it has been rediscovered and modified by
David Gault. He introduced an adipofascial flap to cover the non-absorbable
sutures.
Conchomastoid sutures were described by Furnas and anterior cartilage
scoring technique is named after Chongchet technique.
Ans: C

27
17) The blood loss in a tumescent liposuction is approximately:
A) 1% of the aspirate
B) 3% of the aspirate
C) 10% of the aspirate
D) 20% of the aspirate
E) 40% of the aspirate
Ans: See end of page
Explanation
Tumescent liposuction uses a ratio of 3:1 for infiltration and the amount of
blood loss is approximately 1% of the total aspirate. The blood loss in other
techniques is as follows:
Dry technique: 20 - 40%
Wet technique: 4 - 25%
Superwet technique: 1%
The dry technique is therefore not preferred due to the excessive blood loss.
Ans: A

28
18) The two fat pads in the upper eyelid are separated by the:

A) Interpad septum
B) Superior oblique muscle
C) Superior rectus muscle
D) Levator muscle
E) Extension of medial canthal tendon
Ans: See end of page
Explanation
There are distinct fat pads located behind the orbital septum (post-septal
fat). There are two fat pad in the upper eyelid, which are separated by the
superior oblique muscle. The medial fat pad is lighter in colour and firmer in
consistency. There are three fat pads in the lower eyelid.
There are fat deposits beneath the orbicularis muscle called pre-septal fat.
On the upper eyelid it is called retro-orbicularis oculi fat (ROOF) and on the
lower eyelid it is called suborbicularis oculi fat (SOOF).
Ans: B

29
19) The recognised incidence of blindness following blepharoplasty is:
A) 0.4%
B) 0.04%
C) 0.004%
D) 4%
E) 0.1%
Ans: See end of page
Explanation
The incidence of blindness following blepharoplasty is estimated to be in the
range of 0.04%. This occurs as a consequence of bleeding and retrobulbar
haematoma, which causes optic nerve damage by compression of the
central retinal vessels.
Treatment of a retrobulbar haematoma should be urgent and involves
release of the orbital septum to evacuate the haematoma and medical
measures to reduce the intraocular pressure.
Ans: B

30
20) The muscle most frequently injured during a blepharoplasty is:

A) Inferior rectus
B) Inferior oblique
C) Superior rectus
D) Superior oblique
E) Levator palpebrae superioris
Ans: See end of page
Explanation
Diplopia is a recognized complication of blepharoplasty. Temporary diplopia
is attributed to oedema of periorbital tissues, whereas permanent diplopia
can result from damage to extraocular muscles. The muscle most commonly
injured is inferior oblique followed closely by the superior oblique muscle.
The mechanism is believed to be an excessive use of cautery or direct
trauma from injection of local anaesthetic. Risk of diplopia is increased in
revision blepharoplasty.
Ans: B

31
21) The Baker-Gordon formula for chemical peel contains:

A) Trichloro-acetic acid
B) Glycolic acid
C) Phenol
D) Alpha hydroxyl acids
E) Beta hydroxyl acids
Ans: See end of page
Explanation
There are several types of chemical peels used for facial rejuvenation.
These include alpha-hydroxy acids, salicylic acid, glycolic acid, trichloro-
acetic acid and phenol.
There are also combination formulae used for chemical peeling and one of
the well-known mixture is the Gordon Baker formula.
In 1961, Baker described a formula for chemical peels and the composition
is as follows:
· Phenol USP 3 mL
· Tap water 2 mL
· Liquid soap 8 drops
· Croton oil 3 drops
Ans: C

32
22) Skin slough, a complication of facelift, occurs most commonly in
the:

A) Temporal scalp
B) Pre-auricular area
C) Post-auricular area
D) Neck
E) Nasolabial fold area
Ans: See end of page
Explanation
Skin slough following a facelift is a sequel of haematoma or infection and
occurs commonly in the post-auricular area. The incidence ranges from 0-
14% in a subcutaneous facelift and 0-2.4% in a sub-SMAS facelift. Smoking
is a well-known risk factor for this complication. Patients should be advised
to refrain from tobacco prior to rhytidectomy. Small vessel occlusive disease
has been noticed in rhytidectomy patients who are smokers, and this
correlates with a higher incidence of skin slough afer surgery
Ans: C

33
23) In blepharoplasty:

A) It is important to remove most of the fat from the central-lateral


compartment
B) Vector-negative patients are at a higher risk of post-operative ectropion
C) The supratarsal fold is deficient in the ‘Oriental’ eyelid due to reduced
amount of orbital fat
D) Total loss of vision occurs when the intraocular pressure raises above 5
mmHg
E) Inadvertent removal of the lacrimal gland has no deleterious
consequences
Ans: See end of page
Explanation
The relative position of the globe in relation to the zygoma is important in
predicting the risk of post-operative ectropion. If the globe is anterior
(vector-negative), there is a high risk of ectropion and a lateral canthopexy
or malar augmentation should be considered. Excessive removal of central
fat may reduce the ‘ball-bearing’ mechanism of the septo-aponeurotic
vehicle, causing a sunken appearance of the eye.
The septum and aponeurosis may fuse at variable levels in the eyelids; in
Orientals it is usually more inferior. The supratarsal fold is formed by fibrous
insertions of the levator aponeurosis into the skin. Retrobulbar haemorrhage
occurs due to bleeding posterior to the orbital septum. Excessive pressure
on the globe may lead to irreversible loss of vision. Intraocular pressure
above 40 mmHg requires immediate treatment (normal intraocular pressure
is 10 – 22 mmHg). Inadvertent removal of the lacrimal gland may cause dry
eyes.
Ans: B

34
24) The main anatomical reason for the difference in the aesthetics of
Oriental and Caucasian eye lid is the

A) absence of orbital septum


B) absence of pre tarsal orbicularis muscle
C) lower level fusion of the levator aponeurosis and orbital septum
D) large pre septal fat pad
E) lower origin of levator palpabrae superioris
Ans: See end of page
Explanation
The orbital septum forms the anterior border of the orbit, originating from
the arcus of the bony orbit and fusing with the levator aponeurosis. The
fusion of these two structures forms a sling that houses the orbital fat pad. It
is the difference in fusion of the orbital septum and the levator aponeuosis
that contributes to the difference in aesthetics of the Oriental and Caucasian
eye lids. In the Oriental eyelid, this fusion happens at a lower level resulting
in the fat pad and the sling to descend lower. In addition, the levator
expansions do not penetrate the orbital septum in the Oriental eyelid. These
manifest as an absent palpebral fold in the upper eyelids of Orientals.
Similarly, in the lower eyelid Orientals do not have a palpebral crease as the
fibrous bands from the orbital septum do not penetrate the orbicularis oculi.
Ans: C

35
25) The type of LASER most commonly used for tattoo removal is the

A) CO2
B) Nd-YAG
C) Q-switched
D) Erbium
E) Argon
Ans: See end of page
Explanation
There are several methods for tattoo removal, ranging from LASER to
surgical excision with or without skin grafting. Different types of LASER can
be used for tattoo removal; Q-switch LASER is the most commonly used
type. LASER works by the process of Selective photothermolysis by
targeting specific pigment of the tattoo. The pigments are localized intra
cellular in the mast cells, macrophages and fibroblasts around the blood
vessels. The possible mechanism of action is the LASER induced
thermolysis causes the pigment to be moved extra cellularly and drained by
the lymphatics where phagocytosis causes the breakdown of them into
small pigment particles.
Ans: C

36
26) The main disadvantage of supra-brow excision as a technique of
brow lift is the
A) scar
B) injury to supra orbital nerve
C) loss of eyebrow
D) relapse
E) worsens blepharoptosis
Ans: See end of page
Explanation
Supra brow skin excision involves an excision of ellipse of forehead skin just
above the eyebrow. The skin excision can range from small ellipse along the
lateral cantus to large area along the forehead rhytides. Supra brow skin
excision is a quick and simple technique for correcting brow ptosis. As it only
involves excision of skin, there is no risk of damage to deeper structures.
But it does not address the deeper soft tissues that contribute mainly to the
brow ptosis. Although the results are acceptable in the immediate post-
operative period, there is a high incidence of relapse.
Ans: D

37
Aesthetic Surgery
Extended Matching Questions
1) BLEPHAROPLASTY
Options
A) Skin muscle flap technique
B) Transconjunctival technique
C) Skin flap technique
D) Invagination technique
E) Canthopexy
F) Tarsorrhaphy
G) Dacryorhinostomy
H) McGregor flap
From the list of options above, choose the most appropriate option. Each
option may be used once, more than once or not at all.
Questions
1. Preferred method in the presence of fat herniation with minimal skin
excess
2. Preferred method in the lower lid blepharoplasty for fat removal only
without skin excision or canthopexy
3. Used in vector-negative patients as an adjunctive procedure

38
Answers

1-A
2-B
3-E
Explanation
The skin-muscle flap technique involves a subciliary incision 2-3 mm below
the eyelashes in a horizontal natural skin crease over the orbital rim and then
carried through the orbicularis muscle exposing the orbital septum. The
periorbital fat is removed through an incision in the orbital septum. The plane
of dissection posterior to the muscle is avascular and easy to identify with
minimal riskof button-holeing the flap. The skin-muscle flap is not effective in
patients with marked skin redundancy.
The transconjunctival approach involves an incision in the conjunctiva on the
inner aspect of the lower lid and is useful when only fat removal is
performed. It minimizes postoperative lower lid retraction. The main
disadvantage of the transconjunctival approach is limited exposure and
difficulty in removing fat, especially from the lateral compartment.
Lateral canthopexy is relatively simple and provides long-lasting aesthetic
results in lower lid and midface rejuvenation. It can be approached through a
sub-ciliary incision, upper lid incision or forehead lift. Through an extension of
the suborbicularis dissection supraperiosteally, the malar tuft can be
anchored into the orbital rim. It is indicated in patients with a negative vector
(globe anterior to the malar bone) to prevent post-operative ectropion. It is
also useful in patients with laxity of the lower eyelid.

39
3) FACIAL MUSCLES

Options
A) Masseter
B) Platysma
C) Risorius
D) Zygomaticus major
E) Levator anguli oris
F) Procerus
G) Nasalis
H) Corrugator supercilii
From the list of options above select the single correct answer. Each option
may be used once, more than once or not at all.
Questions
1. The main muscle involved in smiling
2. This muscle is not innervated by the facial nerve
3. The nasolabial fold is formed by the insertion of this muscle

40
Answers

1-D
2-A
3-D
Explanation
The muscles of facial expression are derived from the second branchial arch
and therefore supplied by facial nerve. The muscles of mastication
(temporalis, masseter, lateral and medial pterygoid) are derived from the
first branchial arch and are supplied by the mandibular division of trigeminal
nerve.
Smile is a facial expression formed by contraction of the facial muscles near
both ends of the mouth. The French physician, Guillaume Duchenne, has
researched the anatomy of smile and identified two distinct types of smile: a
Duchenne smile (spontaneous smile) involves contraction of zygomaticus
major and orbicularis oculi whereas a non-Duchenne smile involves only the
zygomaticus major. The zygomaticus major draws the angle of the mouth
superolaterally. The zygomaticus minor helps to raise the upper lip when
showing contempt or to deepen the nasolabial sulcus when showing
sadness.
The nasolabial fold is formed by the insertion of the thinned SMAS and
muscles originating on the zygoma, i.e., zygomaticus major and minor and
levator labii superioris.

41
4) SKIN TYPES

Options
A) Fitzpatrick type I
B) Fitzpatrick type II
C) Fitzpatrick type III
D) Fitzpatrick type IV
E) Fitzpatrick type V
F) Fitzpatrick type VI
G) Fitzpatrick type VII
H) Fitzpatrick type VIII
From the list of options above select the corresponding skin type to the
following statements. Each option may be used once, more than once or
not at all.
Questions
1. Burns sometimes and is sensitive to ultraviolet light
2. Always burns, never tans
3. Never burns, tans easily with the dark brown skin

42
Answers

1-C
2-A
3-E
Explanation
Fitzpatrick classified skin types according to pigmentation and susceptibility
to ultraviolet light (Fitzpatrick TB, 1988). In his classification there are 6 skin
types. If the Fitzpatrick grade (i.e., I or II) is lower then the pigmentation of
the skin is less.
Fitzpatrick I: White, always burns, never tans, very sensitive to UV
Fitzpatrick II: White, always burns, sometimes tans, very sensitive to UV
Fitzpatrick III: White, burns sometimes, tans gradually, average sensitive
Fitzpatrick IV: Light brown, burns rarely, tans with ease, moderately
sensitive
Fitzpatrick V: Brown, never burns, tans easily, minimally sensitive
Fitzpatrick VI: Black, never burns, deeply tans

43
5) PREMATURE AGING

Options
A) Ehlers-Danlos syndrome
B) Werner’s syndrome
C) Hutchinson-Gilford syndrome (progeria)
D) Cutis laxa
E) Meretoga’s syndrome
F) Pseudoxanthoma elasticum
G) Idiopathic skin laxity
H) Cutis hyperelastica
From the list above, select the single most correct answer for the following
descriptions. Each option may be used once, more than once or not at all.
Questions
1. Affects the individual with generalized loose skin in all regions of the body,
which is inelastic with normal skin healing
2. Inheritance is autosomal recessive and is typically associated with severe
microangiopathy and manifest in early adult life
3. Characterized by degeneration of elastic fibers in the dermis

44
Answers

1-D
2-B
3-D
Explanation
Cutis laxa is characterized by degeneration of elastic fibers in the dermis.
There is laxity of skin, which is inelastic and does not recoil after stretching.
There is no hyperextensibility of the joints and wound healing is normal. The
coarsely textured lax skin lies all over the body and presents in the neonatal
period.
In contrast, Ehlers-Danlos syndrome (cutis hyperelastica) is characterized
by skin friable and hyperextensile. An important feature is the ability of the
skin to stretch to a large extent and when released the skin recoils back.
Patients affected with this syndrome also have wound healing problems.
Werner’s syndrome (adult progeria) is often diagnosed in early adulthood
with indurated and variably pigmented skin, aged facies, alopecia, cataracts,
short stature and premature arteriosclerosis. Because patients exhibit
severe microangiopathy, elective surgery is (relatively) contraindicated and
facial surgery should be avoided.
Hutchinson-Guilford syndrome (Progeria) manifests as growth retardation,
micrognathia, alopecia, craniofacial disproportion and lax, irregularly
contoured skin with loss of subcutaneous fat. The disease progresses
rapidly and early death can be expected.
Meretoga’s syndrome manifests as excessively lax skin of the face after the
age of 20 years due to facial polyneuropathy with amyloid deposits in the
peripheral nerves.

45
6) LASERS IN PLASTIC SURGERY

Options
A) Pulsed Dye LASER
B) Nd-YAG LASER
C) Intense Pulsed Light LASER
D) CO¬2 LASER
E) Argon LASER
F) Alexandrite LASER
G) Q-switched ruby LASER
H) Diode LASER
For the questions given below, select the most appropriate answer from the
list of options above. Each option may be used once, more than once or not
at all.
Questions
1. The LASER that can be used in tattoo removal but can result in
permanent scar because of its deeper penetration into the dermis
2. The LASER currently most commonly used to treat port wine stain
3. The LASER used in commercially available devices for hair removal

46
Answers

1–D
2–A
3–C
Explanation
CO2 LASER works by targeted destruction of the tissues by selective
photothermolysis. It can be used for ablation of superficial skin lesions (e.g.,
warts) and to remove tattoos. However, due to thermal damage to the
dermis delayed healing and hypertropic scaring may occur with these
LASERs.
The pulsed dye LASER is a visible light type of LASER that targets
oxyhaemoglobin in blood vessels. It has a 517 nm wavelength with a depth
of penetration of 1mm. Hence it is useful in superficial vascular lesions such
as port wine stains.
The commercially available portable devices for hair removal use Intense
Pulsed Light (IPL). It is not a LASER as such but rather a flash-lamp device
that produces light of high intensity during a very short period. Unlike
LASERs, these devices emit non-coherent pulsed light of wavelength
between 551 – 1,200 nm. This variable wavelength makes it suitable for
used in different types of skin lesions. Hair removal using IPL is cheaper and
quicker compared to LASERs though its effectiveness is not as good as
Alexandrite or Nd-YAG LASERs.

47
7) LASERS IN PLASTIC SURGERY

Options
A) CO2
B) Nd-YAG
C) Argon
D) Long-pulse ruby
E) Erbium
F) Q-switched Nd-YAG
G) KTP
H) IPL
From the list above select the type of LASER that corresponds to the
following statements. Each option may be used once, more than once or
not at all.
Questions
1. It has a wavelength of 10,600nm
2. Useful in resurfacing very thin skin and bleeding from dermal vessels may
occur when this type is used
3. It is usually used for tattoo removal

48
Answers

1-A
2-E
3-F
Explanation
LASER is an acronym for Light Amplification by Stimulated Emission of
Radiation.
Carbon dioxide LASER is a gas LASER which has the highest wavelength
10,600nm and is used for skin resurfacing by tissue vaporization. It can also
be used for excision of lesions. The chromophore for CO2 LASER is water.
Erbium LASER has a wavelength of 2,940nm and can be used for
resurfacing very thin skin. Compared to Carbondioxide LASER more passes
are required for the same depth of penetration with much less thermal
damage. As it is not a coagulating LASER, bleeding may occur from dermal
vessels.
Q-switched mode delivers rapidly pulsed LASER energy, which penetrates
to the level of the upper papillary dermis and can selectively target ink
particles. The heated particles explode, producing smaller particles that can
be removed by phagocytosis. Q switched Nd-YAG LASERs are useful for
removal of tattoo pigments. Q-switched Alexandrite is useful for removal of
blue-black and green pigments.

49
Breast and Trunk Reconstruction
Single Best Answers
1) Breast develops from the:
A) Ectoderm
B) Endoderm
C) Mesoderm
D) Ectoderm and mesoderm
E) Neural crest cells
Ans: See end of page

Explanation
The breast develops during the sixth week of gestation from a collection of
ectodermal cells along the milk lines or primitive mammary ridges, extending
from the axilla to the groin. Most of these ridges, apart from the pectoral
ridge at the 4th intercostal space, disappear by the 10th week of gestation.
This remaining ridge develops later into the breast. The areola develops by
the 5th month of gestation and the nipple develops shortly after birth.
Ans: A

50
2) Which of the following is true regarding the innervation and blood
supply of the breast?

A) The lateral cutaneous branch of the fourth intercostal nerve is the most
constant nerve to the nipple
B) The breast is innervated by branches of the second to fourth intercostal
nerves
C) The breast is vascularised by two main arteries
D) The internal mammary system accounts for over 80% of total breast
blood flow
E) The subscapular artery arises from the thoracoacromial trunk
Ans: See end of page
Explanation
The breast is innervated by the lateral and anterior cutaneous branches of
the second to sixth intercostals nerves. The anterior cutaneous branches
innervate the medial portion of the breast. The innervation of the nipple and
areola shows frequent variations in number and distribution of anterior and
lateral cutaneous branches of the third, fourth or fifth intercostal nerve. The
fourth lateral cutaneous branch is the most constant, innervating the nipple in
93% of breasts.
The breast is supplied by multiple arterial routes: internal mammary artery,
lateral thoracic artery, thoracoacromial trunk and intercostals arteries. The
internal mammary artery through anterior and posterior perforating branches
provides approximately 60% of the blood flow, mainly to the medial breast.
The lateral thoracic artery supplies up to 30% of blood flow to the lateral
and upper outer breast quadrants. It arises from the axillary artery and
occasionally from the thoracoacromial or subscapular artery. The
subscapular artery is a branch of the axillary artery.
Ans: A

51
3) Which of the following is true regarding lymphatic drainage of the
breast?

A) Fifty percent of the lymphatic drainage of the breast occurs through the
axillary nodes
B) The level of axillary lymph nodes is described in relation to the Pectoralis
major muscle
C) Rotter’s nodes are lymph nodes along the serratus anterior muscle
D) Inadvertent injury to the thoracodorsal nerve during axillary dissection
results in a winged scapula deformity
E) Palpable axillary lymph adenopathy is a contraindication to sentinel lymph
node biopsy
Ans: See end of page
Explanation
About 75% of the lymphatic drainage of the breast is to the axillary basin,
most of the rest is to the internal mammary nodes and some to the
inframammary nodes. Rotter’s nodes are interpectoral nodes between the
pectoralis major and minor muscles. The axilla is divided anatomically into
three levels in relation to the pectoralis minor muscle. Level I nodes are
located lateral to the pectoralis minor muscle, level II behind and level III
medial to it. The thoracodorsal nerve supplies innervation to the latissimus
dorsi muscle. The long thoracic nerve innervates the serratus anterior
muscle, inadvertent damage to which will result in winged scapula. Sentinel
lymph node biopsy (SNLB) identifies the first lymph node receiving drainage
from a primary neoplasm. Contraindications to SNLB include multicentric
carcinoma, locally advanced disease and palpable axillary lymphadenopathy.
A node filled with tumour will not take up mapping agent giving a false-
negative result.
Ans: E

52
4) Which of the following is true regarding breast diseases?

A) Fibroadenoma typically occurs bilaterally during puberty


B) The majority of phylloides tumours degenerate to sarcoma
C) Women with fibrocystic disease in their breasts have an increased risk of
developing breast cancer
D) Accessory nipples may occur outside the milk lines
E) Paget’s disease of the nipple is a benign eczematous condition
Ans: See end of page
Explanation
Accessory nipples develop along the mammary ridge or milk line (line from
the axilla to the groin), most commonly just below the normal breast.
Approximately 50% of the patients and sites outside the milk lines have been
described: scapula, thigh and head and neck.
Fibroadenoma is most common between women aged 20 –50 and is usually
unilateral. It is due to benign fibroepithelial profliferation.
Phylloides tumours (also called cystosarcoma phylloides) are rapidly
growing fibroepithelial periductal tumours. The majority are benign, although
they may recur locally. Malignant degeneration to sarcoma is reported in 6%
of cases. Fibrocystic disease is the most common among all benign breast
conditions, commonly presenting as cyclic, diffuse nodules in both breasts.
Fibrocystic changes alone do not confer an increased risk of developing
breast cancer.
Paget’s disease is a form of intraductal carcinoma that spreads into the
surrounding nipple-areola complex. It is associated with an underlying breast
malignancy with 50-60% of those having a palpable malignant mass. Of the
patients without a mass lesion, 30% will later be found to have an invasive
carcinoma.
Ans: D

53
5) Which of the following is true regarding breast tumours?

A) Her-2/neu amplification is associated with a poorer prognosis in node-


positive breast cancer patients
B) Aromatase inhibitors are only effective in pre-menopausal women
C) Tamoxifen acts mainly on the progesterone receptors
D) p53 is a tumour suppressor gene which can be found on chromosome 13
E) Patients with high concentrations of cyclic AMP binding proteins have
good prognosis
Ans: See end of page
Explanation
Twenty-five to thirty percent of breast cancers overexpress the oncoprotein
Her2/neu, a transmembrane tyrosine kinase receptor, which is associated
with a poorer prognosis in node-positive women. However, it seems to be of
less value in delineating the prognosis in node-negative women. Abnormal
expression of p53, a tumour suppressor gene found on the short arm of
chromosome 17, is often found in patients with Li-Fraumeni syndrome, who
have a greatly increased risk of breast, ovarian and bowel cancer.
Tamoxifen acts both as estrogen agonist and antagonist, which may result in
incomplete blockage of the estrogen receptor thus contributing to its
adverse effects.
High concentrations of cyclic AMP binding proteins, the regulatory
subunits of protein kinase A in a major second messenger system, are
present in 10-15% of breast cancers; this is associated with a very poor
survival rate. The concentration of this can be used to identify a subgroup of
patients who do not have axillary node involvement but yet a poor prognosis.
Ans: A

54
6) Which amongst the following statements is correct regarding the
prevalence of breast cancers?

A) Hereditary breast cancer accounts for 2% of cases


B) Presence of the BRCA gene confers a 25% risk of developing breast
cancer
C) Lobular carcinoma in situ (LCIS) confers a 20% risk of developing breast
cancer
D) LCIS occurs predominantly in women after menopause
E) Tubular carcinoma accounts for 20% of breast cancer
Ans: See end of page
Explanation
Hereditary breast cancer accounts for 5-10% of breast cancer cases and is
mainly caused by BRCA gene mutations. The presence of this gene confers
a 60 –85% risk of developing breast cancer.
Lobular carcinoma-in-situ arises from the epithelium of breast lobules and
confers a twenty percent risk of developing cancer in either breast. LCIS
occurs predominantly in premenopausal women. It is usually an incidental
histological finding and has no typical radiologic or physical manifestations.
Invasive ductal carcinoma accounts for the majority of all breast cancer
cases, followed by invasive lobular carcinoma (5-15%). Other types
(medullary, mucinous, tubular) are rare and accounts for about in 2-6% of
breast cancers.
Ans: C

55
7) Which of the following statements is correct regarding breast
cancers?

A) Lobular Carcinoma In Situ (LCIS) differs radiologically from Ductal


Carcinoma In Situ (DCIS) by typical diffuse long radio-opaque strands on
mammograms
B) Tubular carcinoma of the breast is a poorly differentiated tumour with bad
prognosis
C) Microcalcifications on mammograms are typical signs of intraductal
carcinoma
D) Stage IV (AJCC classification) disease indicates locally advanced breast
cancer
E) Lobular Carcinoma In Situ is confined by the basement membrane of the
breast ducts
Ans: See end of page
Explanation
DCIS, also known as intraductal carcinoma, is confined by the basement
membrane of the ducts and can present with a breast mass, nipple
discharge or Paget’s disease. They may be asymptomatic and detected
through screening programmes as mammographic microcalcifications.
On the contrary, LCIS arise from the epithelium of the breast lobules and
has no radiologic manifestations. Tubular carcinoma is a rare histologic
variant of breast cancer and tends to be highly differentiated with excellent
prognosis.
The American Joint Committee on Cancer (AJCC) TNM staging system is
based on clinical and pathologic information. Stage I and II are considered
early breast cancer; stage III locally advanced breast cancer and stage IV
metastatic breast cancer.
Ans: C

56
8) Which of the following is correct regarding the linguine sign?

A) It indicates capsular contracture of a breast implant seen on T1 weighted


magnetic resonance imaging (MRI)
B) It is typically seen as diffuse long strands of increased signal intensity
within a silicone implant
C) It has high sensitivity but low specificity for implant rupture
D) It is a subcapsular line seen in early capsular contracture around breast
implants
E) It represents multiple enfolding lines of the collapsed implant shell
Ans: See end of page
Explanation
Magnetic resonance imaging is a useful investigation in detecting rupture of
breast implant.
The linguine (derived from Italian – a pasta type) sign is seen at T2-
weighted MRI with Short Tau Inversion Recovery (STIR) sequence. The
sensitivity of MRI in detecting breast implant rupture ranges from 46% to
96%, but with the addition of breast surface coils (specialized coils which
suspend the breast and minimize artefacts) the sensitivity increases to 95%-
100%. The specificity of MRI is approximately 94% for intracapsular rupture
of a silicone breast implant. Multiple curvilinear low-signal intensity lines are
seen within the high-signal intensity silicone gel.
The lines represent layers of collapsed elastomeric shell floating in the
silicone gel within the fibrous capsule.
Ans: E

57
9) Which of the following is true regarding capsular contracture?

A) A palpable and visible implant is classified as Baker grade 2 capsular


contracture
B) Saline implants do not develop capsular contracture
C) Textured silicone elastomer shells are associated with higher capsular
contracture
D) Capsular contracture worsens the quality of mammographic monitoring of
breast cancer
E) Postoperative radiotherapy has no effect on breast implants
Ans: See end of page
Explanation
Baker has classified capsular contracture into four grades:
- Grade I Soft
- Grade II palpable firmness, but no visible deformity
- Grade III palpable firmness and visible deformity
- Grade IV painful, firm capsular contracture
All types of breast implants carry a risk of capsular contracture. The rate of
capsular contracture development is lower in implants with a textured shell.
Radiotherapy, whether delivered pre-operatively or postoperatively,
increases the risk of capsular contracture. Capsular contracture interferes
with the quality of screening mammograms.
Ans: D

58
10) Which of the following is true regarding breast reduction
techniques?

A) Regnault described a technique that is based on an inferior


dermoglandular pedicle
B) The SPAIR technique described by Hammond eliminates incision in the
inframammary fold
C) Marchac described a technique that eliminates the medial component of
the horizontal scar in the inframammary fold
D) Lejour modified Lassus’ vertical scar technique by using a superomedial
pedicle
E) The Skoog technique is a central mound approach
Ans: See end of page
Explanation
The short-scar periareolar inferior pedicle reduction (SPAIR) popularised by
Hammond uses an inferior pedicle and Gore-Tex pursestring suture around
the areola without a horizontal inframammary scar.
Regnault described a ‘B technique’ mammoplasty that eliminates the medial
component of the horizontal scar.
Marchac’s vertical reduction technique has a short horizontal component to
the scar to deal with inferior dogears.
Lejour modified Lassus’ technique and popularised vertical reduction
mammaplasty. Her modifications include suspension of the superior pedicle
to the pectoralis major fascia with sutures, undermining of the edges of the
vertical parenchymal pillars and liposuction. Skoog first described a
technique for superolateral dermoglandular support of the nipple. The central
mound technique was popularised by Hester in 1985
Ans: B

59
11) Which of these statements is true regarding gynaecomastia?

A) The most common cause is drug-induced


B) Gynaecomastia is a recognized side-effect of phenytoin
C) Simon et al. classified gynaecomastia into five grades
D) In patients with Klinefelter syndrome, there is an increased risk of breast
cancer
E) All patients with gynaecomastia have an increased risk of malignant
transformation.
Ans: See end of page
Explanation
There is no increased risk of breast cancer in patients with gynaecomastia
when compared to the unaffected male population, except for patients with
Klinefelter syndrome, who have an approximate 60 times increased risk. The
most common cause for gynaecomastia is idiopathic. Other causes are
obesity, endocrine, neoplastic, systemic disease, drugs [hormones, anti-
androgens, H2-receptor blockers, calcium channel blockers] and drug abuse
(heroine, marijuana, alcohol).
Simon et al. classified gynaecomastia into four grades:
• Grade I: Small enlargement, no skin excess
• Grade IIa: Moderate enlargement, no skin excess
• Grade IIb: Moderate enlargement, with skin excess
• Grade III: Marked enlargement with extra skin.
Ans: D

60
12) Which of the following is a significantly increased risk in a heavy
smoker undergoing immediate DIEP flap reconstruction for breast?

A. Mastectomy flap necrosis


B. Anastamotic thrombosis
C. Partial necrosis of DIEP flap
D. Venous congestion of DIEP flap
E. Fat necrosis
Ans: See end of page
Explanation
Tobacco contains several thousands of chemicals, many of which are
detrimental to wound healing. Nicotine acts via the sympathetic system and
produces vasoconstriction, limiting capillary blood flow. Cigarette smoke
also contains carbon monoxide which shifts the oxygen dissociation curve to
the left and forms carboxyhaemoglobin. Chronic smokers also suffer from
hypoxia and as a result have increased coagulability of blood due to
polycythaemia.
With regards to breast reconstruction using DIEP flap, smokers are at
particular risk of necrosis of mastectomy flaps and abdominal skin flaps.
Ans: A

61
13) The most commonly used classification system for grading breast
ptosis was described by:

A) Baker
B) Hartrampf
C) Benelli
D) Regnault
E) Simon
Ans: See end of page
Explanation
Ptosis is defined as droopiness of the breasts wherein the nipples are lower
than their normal position. This occurs due to weakening of the suspenory
ligaments (Cooper’s ligaments) from involution and gravity.
Regnault’s classification system focuses on the position of the nipple-areola
complex (NAC) in relation to the inframammary fold (IMF) and breast
mound.
In the upright patient:
· Grade I is defined as location of the NAC at or slightly above the IMF
· Grade II is an NAC position below the IMF, but anteriorly projected on the
breast mound
· Grade III is a NAC below the IMF and on the dependent position of the
breast mound, sometimes the nipple pointing directly downward.
Pseudoptosis is a term used to describe the condition where the breast
parenchyma drops drown in relation to the nipple, but the nipple stays in the
normal position.
Ans: D

62
14) Which of the following statements regarding breast reconstruction
is true?

A) Anastomosis to internal mammary artery requires a long vascular pedicle


B) Delayed reconstruction should be performed if radiotherapy is required
C) Smoking is an absolute contraindication for DIEP flap
D) Donor site morbidity for DIEP and free TRAM flap is equal
E) Neoadjuvant chemotherapy is not a contraindication for immediate breast
reconstruction with free flap transfer
Ans: See end of page
Explanation
The internal mammary vessel lies approximately 1.5cm lateral to the lateral
border of the sternum and facilitates anastomosis to a relatively short
pedicle.
Radiotherapy can be administered following immediate breast
reconstruction. However, autologous tissue reconstruction is recommended
to avoid implant-related complications.
Smokers have a higher risk of mastectomy flap necrosis and patients should
be advised to quit smoking preoperatively.
The donor-site morbidity is reduced in DIEP flaps due to preservation of the
rectus abdominis muscle.
Neoadjuvant chemotherapy is administration of chemotherapy prior to
mastectomy. This is indicated in locally advanced breast cancers to reduce
tumour bulk. Side-effects of chemotherapeutic agents have to be borne in
mind while operating on these patients. However, there is no
contraindication to undertake immediate breast reconstruction
Ans: E

63
15) Which statement is true regarding the SGAP (Superior Gluteal
Artery Perforator) flap?

A) It was first described by Rubens


B) The superior gluteal artery has a relatively constant anatomy
C) The preferred recipient vessels for this flap are the thoracodorsal vessels
D) At least two perforators are required to vascularize the entire flap
E) The axis of the skin island must be positioned parallel to the perforators
Ans: See end of page
Explanation
The superior gluteal artery perforator flap was developed by Robert Allen in
1993. It is an ideal choice of breast reconstruction by free tissue transfer in
the patient with no availability of the lower abdomen. The superior gluteal
artery has a relatively constant anatomy with the perforators crossing the
fascia perpendicularly. Where the superior gluteal artery exits the
suprapiriform foramen can be marked on the skin at the proximal 1/3 of a
line between the posterior superior iliac spine and the apex of the greater
trochanter. The piriform muscle is located by the middle of a line between
the posterior superior iliac spine and the coccyx with the superior edge of
greater trochanter. The perforators can be localized by Doppler flowmetry
above the piriform muscle laterodistally to the exit point of the superior
gluteal artery. The axis of the skin island can be positioned in any direction
over the identified perforators and one perforator can vascularize the entire
flap. Pedicle length is often insufficient for anastomosis with the
thoracodorsal vessels
Ans: B

64
16) Who first described the inverted-T scar technique in breast
reduction?

A) Strombeck
B) Lassus
C) Wise
D) Pitanguy
E) McKissock
Ans: See end of page
Explanation

In 1956, RJ Wise described a measured keyhole pattern for skin resection


in breast reduction, resulting in an inverted-T scar. This technique has
remained one of the most common techniques among plastic surgeons.
Skin marking is done preoperatively with the patient in upright position.
Nipple is positioned at the level of the inframammary fold, 19 to 23 cm from
the suprasternal notch or 1 to 2 cm below the midhumeral point (Pitanguy
point). A keyhole pattern is drawn centred on the new nipple position with
vertical limbs diverging to a variable degree. The ends of the vertical limbs
are connected to the inframammary fold with horizontal markings. Closure
should be tension-free to reduce the risk of T-junction necrosis.
Strombeck, McKissock and Pitanguy incorporated Wise’s skin resection
pattern in their reduction techniques with horizontal bipedicle, vertical
bipedicle and superior pedicle respectively.
The disadvantages of the Wise-pattern are ‘bottoming out’ and risk of scar
hypertrophy in the long inferior horizontal scars, which led to the
development of vertical scar techniques (Lassus 1964).
Ans: C

65
17) Which of the following statements regarding periareolar
mammoplasty technique is INCORRECT?

A. Closure around the areola is in a purse-string manner


B. The Goes technique incorporates the use of a Vicryl mesh
C. The breast may flatten in the anteroposterior dimension with time
D. It is combined with liposuction
E. The areola may widen with time
Ans: See end of page
Explanation
In 1990, Bennelli described the ‘round block periareolar technique’ for breast
reduction to minimize scarring. The skin excess is taken up in a purse-string
manner around the areola, which may result in puckering but it tends to
settle with time. Criticisms against this technique include the tendency to
flatten the breast in antero-posterior dimension, resulting in a ‘pancake’
appearance and widening of the areola with time.
Goes‘ technique is based on the principle that skin excision alone does not
prevent ptosis and introduced prolene mesh between the parenchyma and
skin.
Hammond introduced a Gore-Tex purse-string suture and developed the
SPAIR technique (short-scar periareolar inferior reduction).
Lejour introduced liposuction in her vertical reduction mammaplasties.
Ans: D

66
18) To prevent nipple necrosis, Lassus’ vertical mammaplasty
recommends that the upper limit of superior nipple transposition
should be no more than:

A)6 cm
B)7 cm
C)8 cm
D)9 cm
E)10 cm
Ans: See end of page
Explanation
In 1964, Lassus introduced the vertical scar technique in 1964. The key
features of his technique were en-bloc resection of skin, fat and gland, no
undermining of the skin, transposition of the areola on a superior pedicle and
a final vertical scar.
In a review of his 30-year experience, he reported no nipple necrosis when
the nipple transposition is no more than 9 cm. The main complication of this
technique is hypertrophic scarring.
Lejour modified this technique by introducing skin undermining and
liposuction.
Ans: D

67
19) In Regnault’s grading of breast ptosis, grade III corresponds to the
position of the nipple-areolar complex

A) at the level of the inframammary fold


B) slightly above the level of inframammary fold
C) above the level of the inframammary fold but with the major portion of
breast mound below the inframammary fold
D) below the inframammary fold but above the most dependent portion of
the breast
E) below the inframammary fold and the most dependent portion of the
breast
Ans: See end of page

Explanation
Ptosis is characterized by sagging of the breasts where the nipples are
positioned lower than normal in relation to the breast glandular tissue. This
occurs due to weakening of Cooper’s ligaments from involution and/or
gravity. Regnault’s classification of breast ptosis focuses on the nipple-
areolar complex (NAC) in relation to the inframammary fold (IMF) and
breast mound.
With the patient upright:
Grade I is defined as location of the NAC at or slightly above the IMF
Grade II is defined as location of the NAC below the IMF, but anteriorly
projected on the breast mound.
Grade III is an NAC below the IMF and on the dependent position of the
breast mound, sometimes the nipple pointing directly caudally.
Ans: D

68
20) The round block periareolar technique for breast mastopexy was
described by:

A) Wise
B) Lejour
C) Lassus
D) Benelli
E) Regnault
Ans: See end of page

Explanation
In 1990, Bennelli described the round block periareolar technique for
mastopexy. In this method, the breast skin is separated from the breast
parenchyma. Several flaps are created in the breast tissue, which are then
sutured criss-cross to provide a conical shape to the breast. The breast
tissue is then laced together using a permanent suture. The skin excess is
taken up in a purse-string manner around the areola, which may result in
puckering that tends to settle with time.
Ans: D

69
21) A combination of herniation of breast tissue through facial ring
beneath the areola and a narrow base is characteristically seen in

A) Tubular breast
B) Tuberous breast
C) Amastia
D) Hypoplastic breast
E) Synmastia
Ans: See end of page

Explanation
The main characteristic features of tuberous breast are the herniation of the
breast tissue through a constricting facial ring beneath the areola,
hypoplasia of one or more quadrants of the breast with elevated infra
mammary fold and a narrow base. The probable aetiology of a tuberous
breast is the formation of a constricting ring of fascia during development,
which results in herniation of the breast tissue and hypoplasia of part of the
breast.
The above condition is differentiated from tubular breast in which there is no
constricting ring but only a reduction in vertical diameter and no large areola.
Although it presents with hypoplasia of one or more quadrants, only 25 % of
patients have actual reduction in breast volume.
Ans: B

70
22) Nuss procedure treats

A) Pectus excavatum
B) Pectus carinatum
C) Poland syndrome
D) Amniotic band to chest
E) Amastia

Ans: See end of page

Explanation
Nuss procedure is a minimal invasive method for correcting pectus
excavatum. It involves elevation of the sternum by a curved metallic strut. It
is performed under thoracoscopic guidance using two incisions over the 4th
intercostal space. The other method for correction of pectus excavatum is
Ravitch procedure which involves osteotomising the ribs and sternum. A less
invasive technique involves placement of a custom-made prosthesis in a
subcutaneous plane.
Ans: A

71
Breast and Trunk Reconstruction
Extended Matching Questions

1) BREAST REDUCTION TECHNIQUES


Options
A) Strombeck
B) McKissock
C) Dufourmentel
D) Lejour
E) Benelli
F) Skoog
G) Lassus
H) Hall-Findlay
From the list of options above, which name is associated with the following
breast reduction techniques? Each option may be used once, more than
once or not at all.
Questions
1. Verticle bipedicle
2. Verticle scar with medial pedicle
3. Horizontal bipedicle

72
Answers

1-B
2-H
3-A
Explanation
McKissock described a vertical bipedicled dermoglandular flap with Wise-
pattern skin incision.
[McKissock PK. Reduction mammaplasty by the vertical bipedicle flap
technique. Rationale and results. Clin Plast Surg 1976; 3:309].
Hall-Findlay pioneered a variation on the vertical scar using a medial pedicle
with inferolateral gland resection. This technique is particularly suitable in
small to moderate reductions and is targeted to shorten the learning curve in
mastering vertical scar techniques.
[Hall-Findlay E. A simplified vertical reduction mammaplasty: shortening the
learning curve. Plast Reconstr Surg 1999; 104:748-759]
Strombeck developed a horizontal bipedicle technique to ensure survival of
the nipple-areola complex.
[Strombeck JO. Mammaplasty-report of a new technique based on the two
pedicle procedure. Br J Plast Surg 1960; 13:79].

73
2) GRADING AND CLASSIFICATIONS

Options

A) Simon’s grading
B) Heimburg’s classification
C) Regnault’s classification
D) Baker’s grading
E) Ninkovic classification
F) Mathes and Nahai classification
G) Pairolero’s classification
H) Cordeiro’s classification

From the list of options above, choose the correct answer for the
statements below. Each option may be used once, more than once or not
at all.
Questions
1. The system to clinically evaluate tuberous breast deformity
2. The zones of TRAM flap were described using injection studies
3. The system to clinically evaluate gynaecomastias

74
Answers

1-B
2-E
3–A
Explanation
Tuberous breast deformity is characterised by a constricted base and a
large areola which gives an impression of herniation of the gland into areola
(pseudoherniation):
Tuberous deformity was classified by Heimburg classification as follows:
· Type I: Hypoplasia of the lower medial quadrant
· Type II: Hypoplasia of the lower medial and lateral quadrants, sufficient
skin in the subareolar region
· Type III: Hypoplasia of the lower medial and lateral quadrants, deficiency
of skin in the subareolar region
· Type IV: Severe breast constriction, minimal breast base
The zones of TRAM flap were originally described by Hartrampf based on
clinical experience and were studied using injection studies by Ninkovic.
In the Ninkovic classification system the zones are as follows:
- Zone I: Ipsilateral side, towards the midline
- Zone II: Ipsilateral side lateral
- Zone III: Contralateral side, towards the midline
- Zone IV: Contralateral side, lateral

75
3) PIONEERS IN BREAST SURGERY

Options

A.Tansini
B. Hartrampf
C. Rubens
D. Shaw
E. Webster
F. Allen
G. Strombeck
H. Hall Findlay
From the list of options above, choose one correct answer. Each option
may be used once, more than once or not at all.
Questions
1. Latissimus dorsi flap was first described by
2. TRAM flap was first described by
3. Rubens flap, using soft tissues supplied by deep circumflex iliac artery,
was first described by

76
Answers

1-A
2-B
3-B
Explanation
Latissimus dorsi flap was first described by Igino Tansini in 1897. He
reported it as an innovative method of closing chest wall defects following
mastectomy. In 1936 Hutchins described the use of this flap in treating
lymphoedema following mastectomy. He hypothesised that bringing in fresh
tissue from the back will alleviate the lymphoedema. Latissimus dorsi flap
was not much in use until 1970s when Brantigan described his 10-year
experience of using it for breast reconstruction. In 1976, Olivari described
his experience using this flap and ever since it has been in the
armamentarium for breast reconstruction. Emmanuel Delay reported a
modification of this flap by harvesting additional fat pads from the back
(extended latissimus dorsi flap) in 1998.
In 1979, Robbins described a vertically oriented myocutaneous flap using
rectus abdominis muscle for breast reconstruction. In 1982, Hartrampf,
Scheflan, and Black reported the Transverse Rectus Abdominis
Myocutaneous flap wherein abdominal tissue is harvested transversely.
Hartrampf also described Rubens flap based on deep circumflex iliac artery
in 1994. Peter Paul Rubens was a renaissance painter whose famous
painting ‘The Three Graces’ depicted women showing the ‘love handle area’
from which the flap is taken. This technique can be used in women, who
have had a previous abdominoplasty and involve taking skin and fat from the
flanks along with a cuff of the abdominal muscles. This technique failed to
gain popularity due to the donor site morbidity.

77
4) VASCULAR ANATOMY RELATED TO THE THORAX AND BREAST

Options
A) Axillary artery
B) External iliac artery
C) Internal
D) Lateral thoracic artery
E) Thoracoacromial trunk
F) Posterior intercostal artery
G) Internal mammary artery
H) Superior epigastric artery
For the following anatomical descriptions select the artery fromabove list.
Each option can be used once, more than once or not at all.
Questions
1. Deep inferior epigastric artery arises from
2. The lateral mammary arteries are branches of
3. The internal mammary artery continues as

78
Answers

1-B
2-D
3-H
Explanation
Rectus abdominis muscle is supplied by superior epigastric artery and deep
inferior epigastric artery, which form an anastomotic arcade behind the
rectus muscle. Superior epigastric artery is a continuation of the internal
mammary artery. Internal mammary artery arises from subclavian artery
and gives of the musculophrenic artery, continuing as superior epigatric
artery. Deep inferior epigastric artery is a branch of external iliac artery. The
anastomotic pattern between the superior and deep inferior epigastric
arteries is variable and this was elucidated by Moon and Taylor. Lower
abdominal tissue can be harvested based on the superior or deep inferior
epigastric artery.
The blood supply of breast tissue is from the following vessels:
· Lateral thoracic artery: This is a branch of the second part of axillary
artery and supplies branches to the lateral part of breast (Lateral mammary
arteries).
· Thoracoacromial artery: This is also a branch from the second part of
axillary artery. It pierces the clavipectoral fascia and divides into four
branches; clavicular, humeral, acromial and pectoral.
· Internal mammary artery: This artery gives perforating branches, which
supply the medial portion of the breast.
· Posterior intercostal arteries: These are branches from the thoracic aorta
and supply breast tissue.

79
Burns
Single Best Answers
1) The systemic effects of a major burn injury include:
A) immune stimulation
B) decrease in circulating catecholamines
C) hypoglycemia
D) hypervolaemia
E) increased vascular permeability
Ans: See end of page
Explanation
Virtually every organ system in the body is affected after a significant burn.
This is due to release of inflammatory mediators and neural stimulation.
Immunosuppression occurs due to depression of many facets of the immune
mechanism, both cellular and humoral. Burn injury results in a
hypermetabolic state, caused by the secretion of the stress hormones,
including cortisol, catecholamines and glucagon. Blood glucose is often
elevated in major burns. Hypovolaemia is a characteristic feature of major
burns, which is due to loss of protein and fluid into the interstitial space. This
is caused by increased vascular permeabiality. The lungs frequently suffer
from the changes of the post-burn systemic inflammatory response (adult
respiratory distress syndrome) even in the absence of inhalation injury.
Ans: E

80
2) Deep dermal burns:

A) are never associated with blister formation


B) have a blotchy red colouration
C) are more painful than superficial burns
D) are characterised by the presence of eschar
E) heal spontaneously by epithelialisation within 14 days
Ans: See end of page
Explanation
Deep burns may be either deep dermal or full-thickness. Deep dermal burns
may have some blistering, but the base of the blister demonstrates the
character of the deeper, reticular dermis often showing a ‘blotchy red’
colouration. This is due to the extravasation of haemoglobin from destroyed
red cells leaking from ruptured capillaries. The dermal nerve endings are
also situated at the level of the deepdermis (reticular) and, therefore
sensation to pinprick may also be lost. Hence they are less painful than
superficial burns (where thenerve endings are intact). Eschar, the
coagulated dead skin which has a leathery appearance, is a feature of full
thickness burn. Deep dermal burns heal after a prolonged period (more than
three weeks), often with hypertrophic scarring. Therefore, early tangential
excision is advocated in most burn centres.
Ans: B

81
3) Which of the following is true regarding chemical burns?

A) Acids cause more damage to deeper tissues than alkalis


B) The onset of pain may be delayed in alkali burns
C) Neutralising agents should be the first-line of treatment in chemical burns
D) Cement burn is a type of acid burn
E) Systemic magnesium may be required in the treatment of burns due to
hydrofluoric acid
Ans: See end of page
Explanation
Chemical burns can be classified into acids, alkalis, and other specific
agents including petroleum products, nitrates, phosphorous and elemental
metals. The local and systemic effects are influenced by the duration of
contact, concentration of the chemical and amount of the chemical agent.
Alkali burns are generally more serious than acid burns, because of deeper
penetration. Acid burns are generally more painful than alkali burns; in alkali
burns, the onset of pain is delayed, which often delays first aid resulting in
more tissue damage. Both acid and alkali burns should be immediately
flushed away with large amounts of water. Alkali burns require longer
irrigation. If dry powder is still present on the skin, it should be brushed
away before irrigation with water. Cement burns is a type of alkali burns. In
cement burns, neutralizing agents have no advantage over water lavage,
because reaction with the neutralizing agent may itself produce heat and
cause further tissue damage. Hydrofluoric acid penetrates tissues deeply
and even small burns can cause fatal systemic toxicity. The first step in the
management involves copious lavage with water and treatment with topical
calcium gluconate gel. Systemic calcium may be required in some patients
as hydrofluoric acid sequesters calcium within the burn.
Ans: B

82
4) Which statement is correct regarding electrical burns?

A) Low voltage injuries do not cause full thickness burns


B) All patients with electrical burns require cardiac monitoring for at least 24
hours
C) High voltage injuries lead to metabolic alkalosis
D) Cataracts are a delayed complication of electrical burns
E) An urinary output of 0.5ml/kg/hr should be aimed for in adults with
myoglobinuria secondary to electrical burns
Ans: See end of page
Explanation
High voltage electrical injuries are those greater than 1,000 volts. Low
voltage injuries can cause deep dermal or full thickness burns.
Cataract is a recognized long-term complication of electrical injury and
occurs in 1 to 8% of patients. It is more common in those who had
involvement of head and neck. Cardiac damage and arrhythmias can arise
as a complication of electrical burns. Cardiac monitoring is indicated only if
cardiac injury is diagnosed on initial electrocardiogram. High voltage
electrical injuries can lead to deep muscle necrosis and release of toxic
metabolites, hyperkalaemia and metabolic acidosis. This should be
corrected by maintaining adequate perfusion and a urine output 1-1.5
ml/kg/hr is recommended for an adult. Sodium bicarbonate or mannitol can
be added to maintain diuresis.
Ans: D

83
5) Trench foot:

A) is caused by acute exposure to temperatures between -14°C to - 8°C


B) is due to microvascular endothelial damage and vascular occlusion
C) appears white even in the absence of deeper tissue destruction
D) is characterized by pruritic, red-purple lesions
E) is not painful
Ans : See end of page
Explanation
Trench foot or cold immersion foot (or hand) is caused due to a non-freezing
injury of the hands or feet. This is typically seen in soldiers, sailors, or
fishermen, who are exposed to wet conditions and temperatures just above
freezing, i.e., 1°C to 10°C (35°F to 50°F). It occurs due to microvascular
endothelial damage, stasis, and vascular occlusion. Although the entire foot
may appear black, deep tissue destruction may not be present. An
alternating arterial vasospasm and vasodilatation occurs, with the affected
tissue first cold and anaesthetic, progressing to hyperaemia in 24 to 48
hours. This then leads to an intense painful burning and dysaesthesia, as
well as tissue damage characterized by oedema, blistering, redness,
ecchymosis, and ulcerations. Pruritic, red-purple skin lesions (papules,
macules, plaques, or nodules) are a feature of chilblain (also known as
pernio) and not of trench foot. Complications of trench foot include local
infection, cellulitis, lymphangitis, and gangrene. Careful protection from
further exposure and proper attention to foot hygiene can prevent the
occurrence of most such injuries.
Ans: B

84
6) The criteria for transfer of burn victims to a specialised burns unit
include:

A) Partial-thickness burns > 5% of the total body surface area in patients


<10 years or >50 years of age
B) Full-thickness burns involving the hands
C) Full-thickness burns > 2% body surface area in any age group
D) Any form of electrical injury
E) All forms of burns in children
Ans: See end of page
Explanation
The British Burns Association criteria for transfer to a specilaised burns unit
are:
· Partial thickness burns more than 15% body surface area in an adult
· Partial thickness burns more than 10% body surface area in a child less
than 16 years of age
· Dermal or full thickness burn more than 5% body surface area
· Burns involving face, hands, feet, perineum
· Circumferential burns of the limbs, neck of trunk
· Associated systemic illnesses of significance
· Inhalational injury
· Significant chemical burns
· Hydrofluoric acid injury more than 1%
· High voltage electrical injury
· Suspected non-accidental injury in children.
Ans: B

85
7) In the management of a six-month old child with burns:

A) The rule-of-nines accurately estimates the percentage of burns


B) Toxic shock syndrome can occur even with minor burns
C) The child should be fully wrapped in damp dressings until he/she reaches
the burns unit
D) An urine output of 0.5 ml/kg/hr is an indicator of adequate hydration
E) A blood lactate level of up to 6 mmol/L suggests adequate intravascular
fluid volume
Ans: See end of page
Explanation
The Wallace Rule-of-Nines can be used in adults to assess the extent of
burns (A Lund and Browder chart is however more accurate).The child
differs from adult in its relative size of different body parts. In a child, the
head and neck are comparatively larger than in an adult and the lower limbs
are smaller. In a child up to one year of age the head and neck is 18% of
the total body surface area whereas each lower limb is approximately 14%.
Toxic shock syndrome due to staphylococcal infection can occur in minor
burns and is due to the release of exotoxins from the bacteria. Hypothermia
is a much greater risk in children than in adults. Cool water should only be
applied to the burn surface as part of first aid and the rest of the child
should be kept warmly wrapped. Continuous assessment (every 30 minutes)
until the child reaches the burns unit is essential. A urine output of 0.5
ml/kg/hr is adequate in adults; however, in children of this age group, a urine
output of at least 1-2 ml/kg/hr is required. Blood lactate level is a useful
indicator in the management of burns and a rising lactate level associated
with a fall in the pH may suggest poor perfusion of the tissues secondary to
inadequate intravascular volume.
Ans: B

86
8) Regarding frostbite:

A) The underlying pathology is microvascular occlusion


B) First degree frostbite is characterised by hyperaemia and oedema with
partial-thickness skin necrosis
C) The muscle is never affected
D) Reperfusion injury does not occur during rewarming
E) The injured part should be kept in warm water at 50°C for 45-60 minutes
Ans : See end of page
Explanation
Frostbite is due to freezing of tissue from intracellular ice crystal formations
and microvascular occlusion with subsequent tissue anoxia. Frostbite is
classified into first, second, third, and fourth degree according to depth of
involvement. In first degree, there is hyperaemia and oedema without skin
necrosis. In second degree, there are large, clear vesicle accompanying the
hyperaemia and oedema with partial-thickness skin necrosis. Third degree
frostbite is characterised by full-thickness and subcutaneous tissue necrosis,
with hemorrhagic vesicle formation. In fourth degree frostbite, along with full-
thickness skin necrosis, there is involvement of the muscle and bone; and
gangrene may ensue.
Treatment should be immediate to decrease duration of tissue freezing,
although rewarming should be cautiously undertaken if there is a risk of
refreezing. Reperfusion injury can occur during rewarming. The injured part
should be placed in circulating water at a constant 40°C (104°F) until the
pink colour and perfusion return (usually within 20 to 30 minutes).
Ans: A

87
9) Which amongst the following statement is true regarding
metabolism in burns?

A) Glutamine is a non-essential amino acid, synthesis of which occurs mainly


in skeletal muscle
B) The hypermetabolic response to burn injury is characterized by an
increase in nitrogen balance
C) In burn injuries, circulating levels of immunoglobulins are increased
D) After excision and grafting of burn wounds the metabolic rate returns to
normal
E) In acute burn injuries circulating levels of cortisol are decreased
Ans: See end of page
Explanation
Glutamine is the most abundant amino acid in the body and is a non-
essential amino acid. Endogenous production in skeletal muscle from other
amino acids in muscle protein is sufficient under normal conditions. However,
in stress such as trauma and burns, endogenous glutamine production is
inadequate to meet the increased requirements and exogenous glutamine is
essential.
In patients with burns more than 40% body surface area, the metabolic rate
reaches 1.5 – 2 times the predicted resting energy expenditure, resulting in
an increased cardiac output, core body temperature, minute ventilation and
a decreased nitrogen balance.
The metabolic rate does not return to normal until wound remodelling is
complete. The hypermetabolic response turns the body into a catabolic
state resulting in increased levels of glucagon and cortisol. The skin is a
large barrier to invading organisms and breakdown of the immune system
occurs with burn injuries. Circulating levels of immunoglobulins are
depressed in burns.
Ans: A

88
10) Which amongst the following statements is true regarding nutrition
in burns?

A) Normal energy expenditure is about 50 cal/ kg/ day


B) Fat provides most of the fuel for energy
C) Medium chain triglycerides are excellent source of immediate energy
D) The usual estimated energy requirement for a severe burn in an adult is
20kcal/kg/day
E) The greatest nitrogen loss is in the first two days post-burn
Ans: See end of page
Explanation
Normal energy expenditure is about 25 cal/kg/day. Medium chain
triglycerides do not require chylomicron formation for absorption and are
rapidly transported to the liver for oxidation, thus forming excellent source of
immediate energy. Carbohydrates provide most of the energy (52-60%),
followed by fat (25-28%). Protein is not used for fuel except in
hypercatabolic states. The energy and protein requirement in burns can be
calculated using the nutritional formulae:
· Calories: 25 Cal / kg + 40 Cal / % of burns
· Proteins: 1g / kg + 3g / % of burns
The greatest nitrogen loss occurs between days 5 and 10 post-burn. About
20% of kcals should be provided by protein to replace that lost through the
burn wound.
Ans: C

89
11) Which of the following is true regarding immunological changes in
burns?

A) Total parenteral nutrition is associated with enhanced endotoxin


translocation
B) Gastric feeding decreases gut blood flow
C) Humeral immunity is unchanged in burn patients
D) The immunological changes in burns are not seen in children
E) The total T-cell population increases soon after burn injury
Ans: See end of page
Explanation
Total parenteral nutrition is associated with impaired mucosal immunity and
enhanced endotoxin translocation. Aims of enteral feeding are to minimize
net protein loss and to protect the gut from bacterial translocations and
prevent Gram-negative septicaemia. Gastric feeding increases gut blood
flow.
Both cellular and humeral immunity are reduced in burn patients. The total T-
cell population decreases soon after burn injury, both CD4+ and CD8+
lymphocytes are reduced due to glucocorticoid-mediated apoptosis.
Immunological changes are the same in children as in adults.
Ans: A

90
12) Which of the following statements regarding carbon monoxide
toxicity is correct?

A) The elimination half-time of carbon monoxide is 2 hours


B) It causes a rightward shift of the oxyhaemoglobin dissociation curve
C) Carboxyhaemoglobin absorbs light at the same wavelength as
oxyhaemoblobin
D) COHb level of more than 40% is fatal
E) COHb level of 20% are asymptomatic
Ans: See end of page
Explanation
Carbon monoxide is a colourless and odourless gas, which diffuses rapidly
into the blood. It binds very well to haemoglobin with a 240 times greater
affinity, so that oxygen cannot be transported resulting in hypoxaemia. The
elimination half-time is 250 minutes and can be reduced to 40 minutes with
administration of 100% oxygen.
Carboxyhaemoglobin (COHb) absorbs light at same wavelength (660 nm)
as oxyhaemoglobin, therefore pulse oximetry readings are unaffected or
even falsely raised.
COHb level of more than 60% can be fatal. Up to a level of 10%, symptoms
may be absent; this will be followed by headache, confusion with levels of
up to 20%.
Ans: C

91
13) In severe burn injuries:

A) Patients have inappropriately high growth hormone secretion


B) Oxandrolone may enhance outcome by reducing protein catabolism
C) Thrombocytosis is common
D) Endotoxins are released from Gram-positive bacteria
E) Cytokines are depleted during septic episodes
Ans: See end of page
Explanation
Oxandrolone is a synthetic derivative of testosterone, which acts as an
anabolic steroid and may enhance outcome in severe catabolic state such
as starvation and severe burn injuries.
Patients with severe burn injuries have inappropriately low growth hormone
secretion and IGF-1 production. Thrombocytopaenia and depletion or
impaired synthesis of coagulation factors is common.
Endotoxins are components of the wall of Gram-negative bacteria and are
released upon lysis of bacteria. Endotoxaemia stimulate excessive amounts
of cytokines (TNF-á, IL-1â, IL-6, IL-8) secretion leading to widespread
tissue injury and organ failure.
Ans: B

92
14) Jackson’s model depicts:

A) The “Rules of Nines”


B) A fluid resuscitation guideline
C) The pathophysiology of a burn
D) Nutritional requirements in burn injuries
E) The phases of inhalation injury
Ans: See end of page
Explanation
Jackson undertook experimental work on burn wounds in the 1970s in
Birmingham, UK. He created a model that aided the understanding of the
pathophysiology of a burn. The burn wound involves three zones of injury.
The central zone, is the zone of coagulative necrosis. Rapid cell death
occurs due to immediate coagulation of cellular proteins. This zone is
surrounded by an area, where tissue perfusion is compromised due to
impaired microvascular circulation. This area is called the zone of stasis.
The outer zone is the zone of hyperaemia as a result of inflammatory
mediators and vasodilatation. Following the resolution of the inflammatory
and vascular response, the tissues in this area can return to normal.
Ans: C

93
15) The formula used to calculate caloric requirements in adult

burn patients is:


A) Parkland
B) Barclay
C) Evans
D) Curreri
E) Monafo
Ans: See end of page
Explanation
A major burn is a hypermetabolic state, wherein the nutritional and the
metabolic rates can be up to 200% that of normal. The increase in metabolic
rate is proportionat with the percentage of burns up to 60%, after which it
plateaus off. There are several nutritional formulae used for calculating the
caloric and protein requirement in burns.
The Curreri formula is useful for calculating the caloric requirement:
25kcal/day + 40cal / % TBSA / day
A diet, which has a ratio of non-protein calorie: Nitrogen of 100:1 will
promote a positive energy balance. Whenever possible, in patients with
burns, nutrition has to be provided through the enteral route to reduce the
risk of bacterial translocation in the gut. Nasogastric or nasojejunal feeding
is recommended in major burns patients.
Ans: D

94
16) Children with burn injuries are prone to:

A) Hyperglycaemia
B) Hypocalcaemia
C) Hyperkalaemia
D) Hyponatraemia
E) Hyperchloraemia
Ans: See end of page
Explanation
There are several biochemical and metabolic changes in burn patients.
The physiological changes in paediatric burns are different from that of the
adults. Children have limited glycogen stores and higher surface area to
weight and intravascular volume ratios. They are prone to hypoglycaemia,
dilutional hyponatraemia and fluid overload. Free water should be limited and
a source of carbohydrate instituted early. Because of the risk of
hyponatraemia it is recommended to prescribe dextrose/normal saline (4%
dextrose/0.18 % saline) as maintenance fluid. If hyponatraemia develops it
can cause intracellular overhydration leading to serious complications
including convulsions and coma.
Ans: D

95
17) Heterotopic ossification after severe burns:

A) Occurs mainly in the scalp


B) Is due to an increased serum level of calcium and phosphate
C) Has an overall incidence of 75% in burns patients
D) Has an onset between 3 weeks to 3 months
E) Is the same as myositis ossificans
Ans: See end of page
Explanation
Heterotopic bone formation is different from myositis ossificans, which
results from direct muscle trauma. It occurs mainly around joints underlying
areas of full thickness burns, particularly between the olecranon and the
medial supracondylar region of the humerus. It is more common in patients
with burns greater than 20% TBSA with an overall incidence of 1 –3%.
Serum calcium and phosphate remain unchanged, although alkaline
phospatase may be elevated. The usual time of onset is between 3 weeks
to 3 months.
Ans: D

96
18) Hypoxemia, cherry red mucous membranes and mental changes in
a burn patient are characteristic of:

A) Cyanide poisoning
B) Carbon monoxide poisoning
C) Ammonia poisoning
D) Chloride poisoning
E) Hydrocarbon poisoning

Ans: See end of page

Explanation
Inhalational injury can result in injury to the airways by three mechanisms:
1. Direct heat injury to the upper airways. Heat injury to the lower
airways is uncommon because of the protective mechanism. But in case of
pressurised steam inhalation injury, direct thermal injury to the lower airways
can happen.
2. Combustion of household substances can result in release of
toxic gases. These include carbon monoxide, cyanides, hydrocarbons,
ammonia and chloride gases.
3. Chemical tracheo-pneumonitis due to soot particles reaching the
lower airways.
Carbon monoxide does not cause direct chemical injury to the lung.
Compared to oxygen, it has around 250 times affinity to haemoglobin. Thus,
it displaces oxygen from the haemoglobin and binds itself to the
haemoglobin. This reduces the oxygen carrying capacity of blood resulting in
tissue hypoxia, shifting the oxygen dissociation curve to the left. Clinically,
patients with carbon monoxide poisoning present with hypoxia but with
cherry red mucous membranes and skin. They also have altered
consciousness depending on the degree of poisoning. It can result in death
when the carbon monoxide levels exceed 60%.
Ans: B

97
19) In a patient with inhalational burn injury, the half-life of
carboxyhaemoglobin whilst breathing room air is:

A) 40 minutes
B) 90 minutes
C) 120 minutes
D) 250 minutes
E) 360 minutes
Ans:See end of page
Explanation
Carbon monoxide toxicity is one of the leading causes of death in patients
with inhalational burns. Carbon monoxide, released as a by-product of
combustion, is a toxic, colourless and odourless gas, which makes it difficult
to detect. It can be fully oxidized forming carbon dioxide. Its binding capacity
to haemoglobin is much stronger than between haemoglobin and oxygen,
thus decreasing the oxygen-carrying capacity of the blood.
The half-life of carboxyhaemoglobin can be reduced by administration of
100% oxygen to the patient as this expedites dissociation of carbon
monoxide from haemoglobin, reducing its half-life from 250 minutes on room
air to 40 minutes.
Ans: D

98
20) Which of the following statements regarding Biobrane® is true?

A) Impregnated with bovine collagen on a prolene mesh


B) Impermeable to wound fluids
C) Should ideally be applied after 24 hours post-burn
D) Indicated mainly for deep dermal burns
E) Seeded with neonatal fibroblasts

Ans: See end of page

Explanation
Biobrane is made up of nylon mesh impregnated with porcine collagen with
an outer silicone layer, which is permeable to gases but not fluids and acts
like an epidermal layer. It allows rapid re-epithelialisation and decreases
pain in partial thickness burns. It is important to apply the Biobrane? within
24 hours of the burn injury for adherence to the wound bed. It is ideally
indicated in superficial partial thickness burns.
Ans: B

99
21) The appropriate antidote for cyanide toxicity is

A) Sodium thiosulfate
B) Magnesium chloride
C) Calcium gluconate
D) Sodium bicarbonate
E) Potassium permanganate

Ans:See end of page

Explanation
Hydrogen cyanide, from burning plastics, is rapidly absorbed through the
lungs and has an inhibitory effect on cell function due to its binding to the
cytochrome system. The manifestations of Cyanide toxicity are dose
dependent. Altered mental status, convulsions and neurotoxicity occur at
high levels. It is gradually metabolized by the liver enzyme rhodanese,
converting it to thiocyanate, which is then excreted in the urine. Although
pure cyanide poisoning is rare it can occur along with carbon monoxide
poisoning.
Sodium thiosulfate, a chelating agent used to treat cyanide poisoning,
provides sulfur for rhodanese to produce thiocyanate. Alternatively,
intravenous sodium nitrite can be used in severe cases. The nitrite produces
methemoglobin, which binds the cyanide. Hydroxycobalamin, a vitamin B12
precursor, in high doses also binds cyanide and thus a treatment option.
Ans: A

100
22) Hyperbaric oxygen therapy after inhalation injury is best used in
patients with:

A) Positive ventilation-perfusion scan


B) Cyanide poisoning
C) Severe neurologic compromise
D) Acute renal failure
E) Soot particles in the bronchioli on bronchoscopy

Ans: See end of page

Explanation
Following inhalation injury, carbon monoxide can cause intoxication leading to
neurological damage and cardiopulmonary arrest. Carbon monoxide is
rapidly transported across the alveolar membrane and has a higher affinity
to haemoglobin than oxygen. As a result, the hemoglobin-oxygen
dissociation curve shifts to the left, impairing oxygen unloading at the
tissues. With prolonged exposure, carbon monoxide can also saturate the
cell and binds to cytochrome oxidase impairing mitochondrial function and
adenosine triphosphate production. Initial manifestations are neurologic,
such as headache and disorientation due to impaired cerebral oxygenation.
In more severe cases, hallucination and coma can occur. Hyperbaric oxygen
therapy is best used in cases of severe neurologic compromise with high
levels of carboxyhaemoglobin (>50%) and no response to high-flow oxygen.
Ans: C

101
23) A patient with a carboxyhaemoglobin level of 15%

A) has no symptoms
B) may complain of headache
C) is disorientated
D) is comatose
E) soon develops cardiopulmonary arrest

Ans: See end of page

Explanation
Carboxyhaemoglobin level is a measure of the amount of carbon monoxide
absorbed into the blood stream. The severity of the carbon monoxide
toxicity corresponds to peak carboxyhaemoglobin level. The peak level
obtained at the scene may be considerably lower if the patient has been
treated with oxygen at the scene and during transfer. Symptoms of carbon
monoxide toxicity are usually not present until it exceeds 15%. Smokers may
have a baseline level of up to 10%.
Carboxyhaemoglobin level - Symptoms
0 – 5% - Normal value
15-20% - Headache, confusion Nausea, disorientation,visual changes
40 -60% - Hallucination, shock state,coma
60% or above - Cardiopulmonary arrest

Ans: B

102
Burns
Extended Matching Questions
1) BURNS RESUSCITATION
Options
A) 450 mls 0.9% NaCl / hr
B) 315 mls Hartmann’s /hr
C) 460 mls Hartmann’s/ hr
D) 275 mls 0.9% 0.NaCl/ hr
E) 290 mls Hartmann’s /hr
F) 425 mls Hartmann’s/hr
G) 500 mls 0.9% NaCl /hr
H) 350 mls Hartmann’s/hr
From the list, select the most appropriate fluid regime to INITIATE
RESUSCITATION in the following patients with a burn injury. Each option
may be used once, more than once or not at all.
Questions
1. A 17-year-old boy weighing 70kg with flame burns to his whole right lower
limb
2. A 21-year-old student weighing 58 kg with 20% mixed depth burns to the
trunk
3. A 10-week pregnant lady weighing 70kg with deep dermal burn to her
whole anterior trunk

103
Answers

1-B
2-E
3-B
Explanation
Burns patients lose fluids through evaporative loss as well as leakage of
fluid into the interstitium. There are several fluid resuscitation formulae used
for correcting the hypovolaemia.
The recommended (EMSB, ATLS) fluid resuscitation for burn injuries in
adults is Hartmann’s solution using the modified Parkland formula: 3 - 4 ml /
kg body weight / % total burn surface area (TBSA) in the first 24 hours. Fifty
percent of it is given in the first 8 hours post-burn (i.e., 0.25 x weight x TBSA
per hour) and 50% over the remaining 16 hours (0.125 x. weight x TBSA per
hour). As a rule of thumb, if majority of the burn is full thickness, from high-
voltage electrical injury or associated with inhalation injury then 4 m/s is
recommended. In other instances, 3 m/s will be sufficient to start with and
then titrated according to physiological parameters.
1. The whole of each lower limb is 18%. Using the formula 0.25 x 70 x 18=
315 mls/hr of Hartmann’s solution should be given in the first 8 hours.
2. The initial resuscitation fluid should be 0.25 x 58 x 20 = 290 mls of
Hartmann’s solution per hour in the first 8 hours
3. The anterior trunk is estimated to be18% TBSA. This makes 0.25 x 70 x
18 = 315 mls of Hartmann’s per hour in the first 8 Hours

104
2) BURNS RECONSTRUCTION

Options
A) Serial excision
B) Full thickness skin graft
C) Z-plasties
D) ‘Jumping man Z-plasty’
E) Tissue expansion
F) Split thickness skin graft
G) Myocutaneous flap
H) Integra
From the list above, select the most appropriate answer. Each option may
be used once, more than once or not at all.
Questions
1. A 6-year-old child with post-burn alopecia approximately measuring 10 x
6 cm
2. A 10-year-old girl with type I axillary contracture
3. A 21-year-old man with interdigital contracture of the 1st webspace

105
Answers

1-E
2-C
3-D
Explanation
The simplest feasible method often yields the best success in burns
reconstruction.
Tissue expansion is the gold standard for burn alopecia in cases where
excision and direct closure is not achievable. With adequate care, young
children tolerate tissue expanders well. Repeated injections to inflate the
tissue expanders in very young children can pose a practical difficulty. Self-
expanding osmotic tissue expanders and tissue expanders with external
ports have been tried to circumvent this problem.
Type I axillary contractures are linear in nature and involve either the anterior
or posterior fold. Usually sufficient local tissue is available to carry out
release via multiple Z-plasties.
The best results in releasing interdigital contractures are through local flaps
(also called ‘jumping-man’ flap or five-flap plasty), which are in effect two
opposing Z-plasties with a V-Y advancement in the centre. An alternative
technique is the four-flap Z-plasty. Both these techniques work only if
surrounding tissues are healthy. In heavily scarred areas, recruitment of new
tissue in the form of local, regional or distant flap may be necessary.

106
3) MANAGEMENT OF CHEMICAL BURNS

Options
A) Irrigation with water
B) 10% calcium gluconate
C) 1% copper sulphate
D) Hypertonic saline
E) Vegetable oil
F) Petrolatum
G) Paraffin oil
H) Toluene
From the list above, select the most appropriate initial management for the
following chemical burns to dilute and remove the damaging effects. Each
option may be used once, more than once or not at all
Questions
1. Burn caused by a rust remover containing hydrofluoric acid
2. Burn caused by a fertilizer containing phosphorus
3. Burn caused by bleach containing sodium hypochlorite

107
Answers

1-B
2-C
3-A
Explanation
The initial management of most chemical burns is copious lavage with water.
However, there are instances where specific antidotes could prove effective.
Hydrofluoric acid is highly corrosive. Although it is an acid, hydrofluoric acid
causes injuries similar to alkali because it penetrates into deep tissues.
Involvement of more than 2% TBSA can prove fatal. It binds with calcium
and magnesium ions within the cell. Arrhythmias may occur due to
hypocalcaemia and hypomagnesaemia. Treatment consists of copious
lavage for about 20 miniutes. Topical application with 2.5% calcium
gluconate jelly or subcutaneous injection with 10% calcium gluconate into the
involved skin can control the pain and progression of the injury as well as
systemic fluoride poisoning. White phosphorus is used in the manufacture of
pesticides and fertilizers. If it comes in contact with the skin, it causes a
chemical burn with a garlic-like odour. Phosphorus ignites spontaneously
when exposed to air and is rapidly oxidized to phosporus pentoxide. A brief
wash with 1% copper sulphate forms black cupric phosphide and facilitates
removal of phosphorus particles.
Sodium hypochlorite is an oxidizing agent typically causing local
inflammation, which can be reduced by copious water lavage.

108
4) FIRST AID IN BURNS

Options
A) 3° C
B) 5° C
C) 8°C
D) 10°C
E) 15°C
F) 22°C
G) 30°C
H) 40°C
From the list, select the most appropriate answer to the following
questions. Each option may be used once, more than once or not at all.
Questions
1. What is the recommended ideal temperature of water used for cooling
burnt body surface?
2. What is the recommended temperature of a circulating water bath used
for treating frostbite injuries?
3. What is the recommended minimum ambient temperature to prevent
hypothermia in a child with severe injuries?

109
Answers

1-E
2-H
3-G
Explanation
First aid measures in burn injuries include stopping the burning process and
cooling the burnt surface.
The ideal temperature of cold running water to cool the burnt skin surface is
15° C and the range useful is between 8°C and 25°C. This reduces the
inflammatory reaction and can stop progression of necrosis. If the
temperature is too cold, it may cause hypothermia.
The treatment of frostbite should be immediate to decrease tissue injury
from freezing. Patient should be warmed using blankets and hot fluids given
orally. The injured limb should be placed in a circulating hot water bath of
40°C (104 °F).
Children have a higher surface area to bodyweight ratio and are at risk of
greater evaporative loss and subsequently they are very prone to
hypothermia. This can be prevented by raising the ambient temperature to
over 30°C.

110
5) BURN SURFACE AREA
Options
A) 1%
B) 2.5%
C) 4.5%
D) 7%
E) 9%
F) 18%
G) 27%
H) 36%
From the list above select the most correct answer for the following
descriptions. Each option may be used once, more than once or not at all.
Questions
1. Half of head of a 6-month old baby
2. External genitalia
3. One buttock in an adult

111
Answers

1-E
2-A
3-B
Explanation
There are several charts designed for calculating the percentage of burns.
A.F. Wallace designed the Rule of Nines, which divides the adult body into
areas of 9% each or fraction/multiples of 9%.
Lund and Browder designed a chart, which is more accurate and makes
allowances for age:
· The total head of a baby <1 year old is 18%;
· External genitalia is 1%
· One buttock is estimated as 2.5%.

112
6) BURNS DRESSINGS AND SKIN SUBSTITUTES

Options
A) Biobrane®
B) Integra®
C) BioAlcamid®
D) Dermagraft TC®
E) Alloderm®
F) Strattice®
G) Cultured epithelial autograft
H) Dermatix®
From the list above, select the product that matches the following
description. Each option can be used once, more than once or not at all.
Questions
1. It has a layer of cross-linked bovine collagen and chondritin-6-sulphate
2. It should ideally be applied within 24 hours of the burn injury
3. It consists of de-epidermalised and de-cellularized sterile human dermis

113
Answers

1-B
2-A
3-E
Explanation
Integra® is an acellular bilaminar artificial skin substitute, made up of an
upper layer of silastic that acts as a barrier to micro-organisms analogous to
the epidermis and a lower layer of crosslinked matrix of bovine collage and
chondroitin-6-sulphate. This gets incorporated into the wound and becomes
a ‘neodermis’ and is used to replace dermis in full-thickness burns.
Biobrane® is made up of nylon mesh impregnated with porcine collagen with
an outer silicone layer, which is permeable to gases but not fluids and acts
like an epidermal layer. It allows rapid re-epithelialization and decreases
pain in partial thickness burns. It is important to apply the Biobrane® within
24 hours of the burn injury for adherence to the woundbed.
Alloderm® is sterile human dermis without cells used as dermal replacement
for both acute and postburn reconstruction. It can also be applied as a
‘sandwich’ with a thin split skin graft over it.

114
Ethics and Consent

Single Best Answers


1) Gillick competence is a medico-legal term that refers to:
A) Competence of a child below the age of 14 to consent for a medical
procedure
B) Competence of an elderly individual to consent for a medical procedure
C) Competence of a child below the age of 16 to consent for a medical
procedure
D) Competence of an individual with learning disability to consent for a
procedure
E) Competence of a medical practitioner to perform a surgical procedure
after being out of work for three years.
Ans: See end of page
Explanation
Gillick competence is a term used in medical law to decide whether a child
(16 years or younger) is able to consent to his or her own medical
treatment, without the need for parental permission or knowledge.
This ruling came into effect following the case of Gillick vs West Norfolk and
Wisbech health authority (1985) wherein Mrs V Gillick ran an active
campaign against prescribing contraceptives to under 16s without their
parent’s consent. The decision of the House of Lords was that a child < 16
years of age can consent for treatment if he or she has sufficient
understanding and intelligence to appreciate the proposed treatment.
Ans: C

115
2) Which is a true statement regarding consent?

A) The next of kin can give consent for an individual with severe dementia
B) Fathers can always give consent for a medical procedure for children
C) A child less than 16 years can legally give consent for a medical
procedure
D) Parents always have a right to deny medical treatment for their children
E) A consent is not legally valid if it is not signed by the patient
Ans: See end of page
Explanation
A child can give consent if he/she is Gillick competent (able to understand
the treatment proposed and its implications).
In the case of a demented person family members cannot give consent,
though they are usually involved in the decision making. These patients
require two doctors to make a combined decision and document it. Although
mothers can always consent on behalf of children, fathers can give consent
only if they are married to the child’s mother at the time or have parental
rights.
Parents do not have a right to deny a medical treatment for their child.
Consent can be implied, verbal or written. For most significant
medical/surgical procedures a written consent is obtained. If a patient is
unable to sign, it can be documented that the patient has agreed to
treatment in the presence of a witness.
Ans: C

116
3) An 8-year-old boy is scheduled to have a pinnaplasty. In the
anaesthetic room, he gets agitated and refuses to have the operation.
His parents state that he always gets ‘worked up’ before visiting a
doctor and he will be fine after the procedure. They thus want you to
proceed with the procedure. The best practice in this situation is:

A) Proceed with the procedure since parents have already signed the
consent
B) Cancel the operation and review the child in out-patient clinic
C) Call the duty manager to obtain legal advice
D) Request the anaesthetist to sedate the child before giving the general
anaesthetic
E) Fill an incident form and write in the notes but proceed with the
operation
Ans: See end of page
Explanation
As legal custodians of the child, parents can give consent for operations.
However, when surgery is undertaken for non-urgent procedures, it is
essential to involve the child in the decision making process. With regards to
pinnaplasty, which is aimed at improving the appearance of the ears, it is
important that the child understands the risks and benefits of the procedure
and he requests the operation.
Ans:B

117
4) You are about to undertake a nail bed repair on a 4-year-old boy. In
the waiting area of the theatre, you realise that the consent form was
signed by the child’s step-father who is currently married to his
mother. On questioning, he informs you that the mother lives far away
and she does not know about the injury. The best practice in this
instance would be to:

A) proceed with the procedure since the consent is valid


B) get a colleague or your senior to sign a two-doctor consent (consent
form 4)
C) call the social services to obtain advice
D) defer the operation and try to contact the child’s mother
E) fill an incident form and write in the notes but proceed with the
operation in the best interest of the child
Ans:See end of page
Explanation
The consent is not valid in this scenario since the man is not the child’s
biological father although he is currently married to his mother. The best
practice in this instance would be to defer the operation and attempt to
contact the mother to obtain her consent. If she is unable to reach the
hospital and sign the form, then a verbal consent over the phone is
acceptable. If the mother in not contactable, procedure are usually
undertaken only if it is a life- nor a limb-threatening emergency.
Ans:D

118
5) You are operating on a 32-year-old computer engineer with a
traumatic hand injury. He has consented for repair of the flexor
tendons and the neurovascular bundle of his middle finger. On
exploring the wound, you note that he has an unstable fracture of the
proximal phalanx that in your clinical judgement requires a K-wire
fixation. You request the ODP to get the patient’s X-ray but are
informed that the patient has not had an X-ray. The scrub nurse and
the anaesthetist bring to your attention that he is not consented for
bony fixation. What would be the best practice in this situation?

A) Proceed with the K-wire fixation since it is in the patient’s best interest
B) Ask the patient’s wife to come to theatre to explain the situation and
obtain her consent with a view to proceed
C) Repair the tendon and neurovascular bundle but not fix the bone
D) Request the anaesthetist to wake the patient, wait for 45 mins in the
anaesthetic room and then proceed with the fixation
E) Fix the fracture with lag screws and decide not to inform the patient
Ans: See end of page
Explanation
This is not an uncommon predicament, which should be avoided by ensuring
that relevant investigations are reviewed and consent form checked before
the patient is anesthetised. However, at this point, as a surgeon you have to
act in the best interest of the patient. Repairing the flexor tendons without
stabilising the fracture will compromise the repair. Deferring the operation to
fix the bone will result in the patient requiring a second general anaesthetic
with its inherent risks. The best practice would be to proceed with fracture
fixation and tendon repair. There is little point in informing the patient’s wife
intra-operatively to obtain her consent since it is not legally valid. Once the
patient has recovered from the anaesthetic, you have to explain to the
patient about the procedure and also document the intra-operative findings
in the case notes. It is also important to fill an incident form.
Ans:A

119
6) You are doing a fellowship in a foreign country. In a charity camp,
you see an 11-year-old girl along with both her parents who are
requesting circumcision for cultural reasons. The girl is willing and
the sister in charge of the clinic states that this is a common request.
You are a bit hesitant and hence you seek the advice of the organiser
of the camp who also confirms that other surgeons do the procedure.
What would you do in this instance?
A) Accept the girl’s and parent’s request, and list her for the procedure
B) State that you are not happy to do the procedure but one of your
colleagues will do it the following week
C) Refuse to carry out the procedure
D) Ask your senior colleague’s opinion before embarking on the procedure
E) Advise the parents to wait for the procedure until the child is old enough
to consent
Ans: See end of page
Explanation
The GMC guidelines explicitly state that female circumcision or female
genital mutilation should not be carried out under any circumstances either in
the UK or abroad. Prohibition of female circumcision act (1985) outlawed the
procedure in the UK and female genital mutilation act (2003) makes it illegal
to perform the above procedure even outside the borders of the UK. It is
punishable by law with imprisonment of up to 14 years.
Ans:C

120
7) Bolam principle refers to the:

A) competence of a child to consent for a surgical procedure


B) competence of an underage mother to consent for her child’s treatment
C) standard of care by a clinician as judged by his peers
D) ability of a 16-year-old child to consent for a cosmetic breast implant
E) freedom of a clinician in acting in the best interests of a patient who
lacks capacity
Ans: See end of page
Explanation
The test to determine the standard of care expected of a doctor is often
referred to as the Bolam principle, which states that ‘a doctor is not
considered negligent if he reaches the standard of a responsible body of
medical opinion’. This followed the case (1957) Bolam vs Friern hospital
management committee, wherein the patient John Bolam received an
uncontrolled electroconvulsive therapy and sustained serious injuries
including acetabular fractures. He sued the hospital for compensation
reasoning that they were negligent in not giving him muscle relaxants, not
restraining and not warning him about the risks. The judge McNair noted that
the medical opinion at the time was not to use muscle relaxants and it was
considered that restraints may increase the risk of fractures. It was also
common practice at the time not to warn patients about the risks of
procedures unless asked. The jury delivered a verdict in favour of the
hospital.
Ans:C

121
8) Which of the following is NOT one of the four ethical principles
popularised by Beauchamp and Childress?

A) Justice
B) Non-maleficence
C) Autonomy
D) Empathy
E) Beneficiance
Ans: See end of page
Explanation
Beauchamp and Childress proposed four cardinal principles in medical
ethics, which are: (i) Justice (ii) Non-maleficence (iii) Autonomy, and (iv)
beneficiance. Justice is fair, equitable and appropriate treatment. Non-
maleficence is an obligation not to harm intentionally or impose the risks of
harm through negligence. Autonomy is the individual’s freedom and capacity
for intentional action, Beneficiance refers to acts performed for the welfare
of others.
Ans:D

122
9) Which of the following statements is true regarding Gillick
competence?

A) Refers to consenting capacity of a patient who is <18 years old


B) A child <16 years cannot refuse treatment even if Gillick competent
C) Is named after the judge who passed the judgement
D) Assessment for Gillick competence has to be carried out by a child
psychiatrist
E) Applies only in England and Wales
Ans: See end of page
Explanation
Gillick competence is applicable to children <16 years of age. However, a
child <16 years cannot legally refuse treatment even if they are Gillick
competent. It is named after Mrs V Gillick in the popular Gillick vs West
Norfolk and Wisbech health authority (1985) case. The assessment for
competence is done by the doctor undertaking the treatment. This standard
has subsequently been approved in all of the United Kingdom as well as
Australia, Canada and New Zealand.
Ans: B

123
10) You are consenting a 9-year-old boy with bilateral prominent ears
for pinnaplasty in a morning list. The anaesthetist, after reviewing the
child in the ward, informs you that the patient has got a late diastolic
(pre-systolic) cardiac murmer that may signify either a mitral/tricuspid
stenosis or a heart block. He feels that it is not prudent to give a
general anaesthetic at the present time. The parents understand the
issue but the boy insists on having the procedure since he gets
teased a lot in school. He is getting distressed and agitated when you
suggest cancelling the operation. The parents are happy for him to
have the procedure under LA although the boy is not willing.

What do you think is the most appropriate course of action in this


instance?
A) Carry out the procedure but only after booking a post-operative bed in
the paediatric intensive care unit
B) As the boy is insisting, obtain his consent and proceed with the operation
under general anaesthetic
C) Cancel the operation since the risks outweigh the benefits
D) Perform the operation under local anaesthesia with parental consent
E) Arrange an urgent ECG and Echocardiogram, and place his name in the
afternoon list or the NCEPOD list
Ans: See end of page
Explanation
Although relevant past medical history should have been explored at initial
consultation, it is good practice to evaluate the risks again before
undertaking any elective procedure. In this scenario, the child is having a
surgical procedure that is neither life-threatening nor functional. It can be
classified as an aesthetic procedure but indeed the psychological and
emotional issues of the child need to be taken into consideration.
Nevertheless there is no indication for this procedure to be carried out on
the day itself – it can wait until appropriate investigations have been carried
out and opined by a specialist (i.e., cardiologist in this case).
Children <16 years are presumed not to have capacity to consent unless

124
they satisfy health professionals that they do have such capacity. It is
unlikely that court would consider children <13 years to be Gillick competent
in most situations. The child getting distressed or agitated is not a good
enough reason to concede to his request since it is not in his best interest.
You cannot perform the procedure under LA in this child even if the parents
are willing, since, firstly, the child won’t agree to have the injection, and,
secondly, he should not forced or coerced. Although an ECG and
Echocardiogram are the correct investigations to carry out, the procedure
should be not rushed and carried out on the same day.
In essence, in this instance, it is best to cancel the procedure, investigate
him appropriately, obtain input from a cardiologist and then list him when it is
absolutely safe for him to have a general anaesthetic. If he is deemed
unsafe, then the procedure can be undertaken under local anaesthetic when
he is older and willing to consent.
Ans:C

125
11) Which amongst the following statements is true regarding legal
principles relating to capacity?

A) A person is globally competent or incompetent


B) Capacity is, ultimately, a legal and not a medical decision
C) The usual standard for proof in civil cases is ‘beyond reasonable doubt’
D) Incapacity has to be established for every single event
E) An imprudent decision is by itself sufficient grounds for incapacity
Ans: See end of page
Explanation
Competent adults have a legal right to refuse medical treatment or demand
withdrawal of treatment. In the legal and ethical analysis of treating people
against their will, the patient needs to be competent, i.e., has legal capacity.
Capacity is ‘function specific’ - a patient may be competent to consent to
one type of treatment but not to another. Ultimately, the court decides on
the question of capacity, although it takes medical assessments of capacity
into consideration. In assessing capacity, the court will consider whether ‘the
balance of probabilities’ favours capacity or lack of it, and uses this as the
standard for proof. Once incapacity is established, it is presumed to
continue until the contrary is proven by acceptable evidence. An imprudent
decision is not by itself sufficient grounds for incapacity. An unwise decision
might however alert a need for assessment of capacity.
Further reading:
Buchanan and Brock described three central elements in defining capacities
needed for competence:
1. Understanding and communication
2. Reasoning and deliberation
3. The person must have a set of values or conception of the good.
These imply cognitive (intellectual) abilities, sufficient short-term memory to
retain relevant information for decision-making and ability to assign different
relative weight or importance to the individual himself, against his set of
values, which must be at least minimally consistent, stable and affirmed as
his own. A patient is considered competent if the above elements are
present.

126
Ans: B

127
Ethics and Consent
Extended Matching Questions
1) Legal Age

Options
A) 11 years
B) 12 years
C) 13 years
D) 14 years
E) 15 years
F) 16 years
G) 17 years
H) 18 years
From the list above, select the most appropriate answer for the following
statements. Each option may be used once, more than once or not at all.
Questions
1. Legal age that a child can consent to treatment, with the consent having
same validity as an adult
2. Legal age at which a child can consent participating in a longitudinal
research study
3. Minimal age that most courts would consider a child to be Gillick
competent

128
Answers

1-F
2-F
3-C
Explanation
Section 1 of the Family Law Reform Act (1969) states that a child is an
individual who is under the age of 12. However, in section 8 it states that a
child aged 16 or above can consent to treatment and such consent is to be
treated in the same way as an adult’s consent although it may not cover
every medical procedure, e.g., cosmetic surgery, tissue donation or
research. The child has to be at least 16 years to participate in any form of
medical research.
Children from the age of 13 can consent to treatment (but can’t refuse
treatment) if they are considered to be Gillick competent. To be Gillick
competent the child must have ‘sufficient understanding and intelligence to
enable him or her to understand fully what is proposed’.

129
Fundamental Principles in Plastic Surgery
Single Best Answers
1) Which among the following statements regarding local flaps is
TRUE?
A) Limberg flap is a type of rotation flap
B) Dufourmentel flap is a variant of transposition flap
C) Rotation flaps are very useful in lower limb reconstruction
D) Bilobed flaps are commonly used in lip reconstruction
E) Langenbeck palatoplasty is an example of an uni-pedicled advancement
flap
Ans: See end of page
Explanation
Limberg flap (also referred to as rhomboid flap) is an example of a
transposition flap. The defect is converted into a rhomboid with angles of
60° and 120°, and a flap of similar dimensions is designed and transposed
into the defect. Dufourmentel flap is a variant of Limberg flap but with
narrower angles (typically 30° and 150°). Rotation flaps are commonly used
in scalp reconstruction and can be combined with scoring of the galea to
enhance the reach of the flap. Bilobed flaps, described by Zimany, can be
used in reconstruction of defects over nasal tip and alar regions.
Langenbeck palatoplasty is an example of a bipedicled advancement flap.
Ans: B

130
2) The main function of the Pectoralis major muscle is to:

A) extend the arm


B) laterally rotate the arm
C) abduct and flex the arm
D) adduct and medially rotate the arm
E) abduct and laterally rotate the arm
Ans: See end of page
Explanation
Pectoralis major along with the lattisimus dorsi forms a powerful adductor of
the arm. It is also a medial rotator of the arm. The chief adductors are the
sternocostal fibres of the muscle whereas the clavicular head helps in medial
rotation of the arm. The muscle is tested by adducting the arm against
resistance with the arm abducted at 60 degrees.
Ans: D

131
3) Which amongst the following nerves arise from the trunk of the
brachial plexus?

A) Upper subscapular nerve


B) Suprascapular nerve
C) Lower subscapular nerve
D) Thoracodorsal nerve
E) Nerve to Rhomboids
Ans: See end of page

Explanation
A sound knowledge of the anatomy of the brachial plexus is important for
understanding the various pathological lesions. The brachial plexus is formed
by the anterior rami of C5 to T1 nerve roots. They unite to form three trunks
(upper, middle and lower) that divide into anterior and posterior divisions.
The divisions unite to form cords as follows:
· Anterior divisions of upper and middle trunks form the lateral cord
· Anterior division of the lower trunk forms the medial cord
· All the posterior divisions form the posterior cord
Branches arise from the roots, trunks and cords of brachial plexus.
The branches from the roots are:
· Dorsal scapular nerve/Nerve to rhomboids(C5)
· Nerve to subclavius (C5, C6)
· Long thoracic nerve (C5, C6, C7)
Suprascapular nerve is the only branch that arises from the trunk (upper
trunk) of the brachial plexus. It supplies the supraspinatus and infraspinatus
muscles.
Ans: B

132
4) Foetal wound healing is characterised by:

A) prolonged inflammatory response


B) rapid migration of fibroblasts
C) a decreased synthesis of total collagen
D) decreased hyaluronic acid synthesis
E) a decreased expression of transforming growth factor beta3
Ans: See end of page

Explanation
Cutaneous wounds in the foetus heal without scarring and with complete
restitution of the normal skin architecture. A number of unique properties of
foetal skin repair contribute to this phenomenon. Foetal wounds are
characterised by reduced proinflammatory signals that results in
suppression of the inflammatory infiltrate during the early phase of healing.
Foetal fibroblasts migrate at a faster rate than adult fibroblasts. Their
increased migration velocity during repair affects collagen deposition and
cross-linking. Foetal fibroblasts also synthesise more total collagen.
Hyaluronic acid, a major component of the extracellular matrix, stimulates
migration of fibroblasts. Foetal fibroblasts synthesise more hyaluronic acid
than their adult counterparts. The expression of transforming growth factor
beta3, a growth factor associated with reduced scarring, is higher in foetal
wounds. The other important differences in foetal and adult wound healing
include an altered balance between matrix metalloproteinases and their
inhibitors, and altered gene expression profiles.
Ans: B

133
5) Choose the correct statement regarding management of wounds:

A) Hydrogel dressings are most useful in clean granulating wounds


B) Alginate dressings are useful in dry necrotic wounds
C) Occlusive dressings are useful in infected wounds
D) Iodine based dressings are not effective in infected wounds
E) Sharp debridement is useful to stimulate healing of chronic wounds
Ans: See end of page
Explanation
Hydrogels retain moisture or provide hydration, and are thus useful in dry,
necrotic wounds. Alginate dressings are highly absorbent, have haemostatic
properties and are useful in packing cavities. They are thus useful in the
management of exudative wounds. Occlusive dressings should be avoided in
infected wounds as they can worsen the infection by retaining the exudate
and causing maceration of the surrounding skin. Iodine based dressings are
effective in the management of infected and exuding wounds, particularly
diabetic foot ulcers. Sharp debridement, either using a curette or a scalpel,
is useful in stimulating healing of inert and static wounds. Sharp debridement
removes excess exudate, necrotic material, bacterial colonies (biofilms) and
other foreign bodies, which are detrimental to healing, besides stimulating an
acute wound healing response.
Ans: E

134
6) Select the correct regarding embryology of the upper limb:

A) The limb bud appears by the 26 day of gestation


th

B) Hand develops by 12 week of gestation


th

C) The embryology of hand development was elucidated by avian grafting


experiments by Swanson
D) Hox d13 mutations are seen in acrosyndactyly
E) The thickened tip of the limb bud is called Zone of Polarising Activity
Ans: See end of page
Explanation
The embryology of upper limb was elucidated through the avian grafting
experiments of Wolpart.
The limb buds appears by the 26th day and develops from the lateral plate
mesoderm on the flank of the developing embryo. The limb bud consists of
undifferentiated mesenchym covered by ectoderm. The tissue at the most
distal part of the developing limb is called the apical ectodermal ridge, which
secretes fibroblast growth factors that influence the proximal-distal
development of the limb. Zone of polarising activity (ZPA) is an area of
mesenchym in the posterior aspect of the developing limb, which secretes
an enzyme called sonic hedgehog. This enzyme is important in the
development of the anteroposterior axis of the limb.
Hand develops during the 33rd day and the rays develop through apoptosis
and radial growth. Hox d13 and Gli3 mutations are seen in polydactyly.
Swanson was responsible for the classification of congenital hand
anomalies.
Ans: A

135
7) Regarding Staphylococci:

A) They are never seen as a skin commensal


B) They typically appear as clusters of red, rod shaped organisms on Gram
staining
C) Staphylococcus epidermidis is the most common cause of post-operative
wound infections
D) They can develop resistance to antibiotics through mutation in the gene
coding for penicillin-binding protein
E) They can cause erythema and blistering of the epidermis through release
of endotoxins
Ans: See end of page
Explanation
Staphylococci, including Staphylococcus aureus and staphylococcus
epidermidis, are common skin commensals. Staphylococci are Gram -
positive cocci and thus stain purple on Gram staining. They characteristically
form clusters on microscopy (staphyle meaning ‘bunch of grapes’ and
coccos meaning ‘granule’ in Greek). Staphylococcus aureus is the most
common cause of post-operative wound infections. Resistance to commonly
used beta-lactam antibiotics, such as penicillins and cephalosporins,
develops through acquisition of MecA gene, which codes for a mutated form
of the penicillin-binding protein. This protein is required for the binding of all
beta-lactam based drugs and leads to infections. They are difficult to treat,
requiring treatment with antibiotics such as teicoplanin and vancomycin.
Scalded Skin Syndrome is caused by release of exotoxins produced by
Staphylococcus aureus that leads to erythema and blistering of the
epidermis.
Ans: D

136
8) Keloids:

A) are caused by wound haematoma and infection


B) are less common in children and young adults
C) usually develop within weeks of initial injury
D) are characterised by an increased collagen degradation
E) extend beyond the margins of the original scars
Ans: See end of page
Explanation
Keloids are dermo-proliferative disorders unique to humans. The aetiology of
keloids is unclear although various theories have been purported including
familial tendency such as an autosomal dominant or recessive inheritance,
hormonal influence, altered immunological response, enhanced role of
transforming growth factor–â, abnormality of keratinocyte control over
fibroblasts, and down-regulation of apoptosis related genes. Factors such
as haematoma, infection and wound dehiscence predispose to hypertrophic
scar formation (not keloids). Keloids extend beyond the original scar
margins whilst hypertrophic scars are confined to the borders of the original
wound. Keloids are more common in wounds that cross tension lines and in
areas such as the earlobe, presternal and deltoid regions. They commonly
affect children and young adults, and they undergo rapid growth during
puberty and increase in size during pregnancy. Hypertrophic scars generally
develop within weeks of injury, whereas keloids can develop up to one year
later. Collagen synthesis is three times higher in keloids than in hypertrophic
scars and 20 times higher in keloids than in normal skin. The absolute
amount of collagen is also increased in keloids, indicative of increased
collagen synthesis or decreased collagen degradation.
Ans: E

137
9) Following tissue expansion the:

A) epidermis becomes thinner


B) elastic fibers remain unchanged
C) collagen bundles become thinner
D) dermal appendages proliferate
E) expanded skin becomes hypervascular
Ans: See end of page
Explanation
Tissue expansion works on the principles of ‘biological creep’ (disruption of
gap junctions and increased tissue surface area that results in cell
proliferation) and ‘mechanical creep’ (morphological changes in tissues due
to stress). Approximately, 70% of tissue gain is obtained from mechanical
creep and the rest from biological creep.
Tissue expansion causes an initial mechanical elongation of skin under a
constant load over time. The epidermis becomes thicker whilst the dermis
becomes thinner. Collagen bundles thicken whereas elastic fibers may
rupture and undergo fibrotic changes due to microfragmentation of elastin.
Expanded skin manifest similar changes associated with the ‘delay
phenomenon’ and Become hypervascular. Angiogeneic factors such as
VEGF are expressed on the surface of the expanded tissue at a higher level
that results in augmentation in blood flow. The muscle may atrophy
significantly during the process of expansion.
Ans: E

138
10) In which amongst the following areas is healing by secondary
intention an acceptable option whilst managing a surgical defect?

A) Medial canthal area


B) Lateral canthal area
C) Upper eyelid
D) Lower eyelid
E) Upper lip
Ans: See end of page
Explanation
Concave areas such as the medial canthus will heal by secondary intention
and thus is an acceptable way of managing defects in this anatomical area
(laissez-faire technique – French: non-interference). Defects in the other
regions may require a local flap or a full thickness skin graft. If defects in the
other regions are left to heal by secondary intention, it may result in scar
contracture leading to a poor cosmetic and functional outcome. For
example, an ectropion may result if defect in the lower eyelid is left to heal
by secondary intention and upper eyelid defects left to heal by secondary
intention may lead to lagophthalmos.
Ans: A

139
11) Which among the following statements regarding skin grafting is
true?

A) Primary contraction is greater in split thickness grafts as compared to full


thickness grafts
B) The dermis does not regenerate during healing of split skin graft donor
sites
C) The organism that is most detrimental to skin graft ‘take’ is
Staphylococcus
D) Contraction of a split thickness graft is due to transformation of the
dermal elements into myofibroblasts
E) Sensory recovery is better in split thickness grafts
Ans: See end of page
Explanation
The elastic recoil of the skin graft after it is harvested is called primary
contraction and is a function of the amount of elastin in the tissues. Elastic
recoil is seen more with full thickness skin grafts. During healing of a split
skin graft donor site, only the epidermis re-epithelialises. This limits the
ability to harvest repeated split skin grafts from one area.
The bacteria most detrimental to graft take is beta-haemolytic Streptococci.
They produce enzymes, which can break down fibrin threads that normally
anchor the graft. The secondary contraction occurring in a split skin grafted
area is due to wound contraction that takes place underneath the graft and
caused by myofibroblasts in the wound (not in the graft). The sensory
recovery following skin grafting is variable and is better with full thickness
skin grafts.
Ans: B

140
12) Which micro-organism is associated with infections following leech
therapy?

A) Aeromonas hydrophila
B) Pasteurella multocida
C) Streptococcus epidermidis
D) Mycobacterium marinum
E) Pseudomonas aeruginosa
Ans: See end of page
Explanation
Medicinal leeches are used to salvage failing flaps or replanted parts when
there is venous congestion. They secrete a local anticoagulant, hirudin,
which allows bleeding for 8 – 12 hours. Patients on leech therapy can
develop infection with Aeromonas hydrophila, a Gram-negatvie anaerobic
rod. These bacteria are endosymbiotic within the leech and they inhibit
growth of other bacteria. They aid the nutrition of the leeches by producing
digestive enzymes to break down red cells and haemoglobin.
Aeromonas hydrophila is commonly sensitive to quinolones such as
ciprofloxacin and patients on leech therapy should ideally be commenced on
this antibiotic.
Ans: A

141
13) Which of the following is true regarding alloplastic implantation?

A) Hyaluronic acid preparations do not get absorbed


B) Silicone is associated with an increased risk of developing connective
tissue diseases
C) Zyderm® is made of porcine collagen
D) Methylmethacrylate elicits an exothermic reaction during drying
E) Cyanoacrylate is non bio-degradable
Ans: See end of page
Explanation
Methylmethacrylate is used in craniofacial bone augmentation, fabrication of
gentamycin-beads and securing artificial joint components. When used for
calvarial remodelling to avoid soft tissue burns, it is important to cool it and
allow the exothermic reaction to settle down.
Average absorption rates of hyaluronic acid (used in lip augmentation or to
treat rhytides) are 20-50% of the original volume by six months. Extensive
reviews of the safety of silicone performed by the UK Independent Review
Group concluded that there was no evidence that silicone implants can
cause systemic diseases. Zyderm® is made from sterilized bovine collagen
and is used to treat rhytides (wrinkles). Cyanoacrylate is the main
constituent of tissue adhesive. It is strong and biodegradable.
Ans: D

142
14) Choose the statement that is true regarding bones:

A) Long bones and iliac crest are formed by intramembranous ossification


B) In primary bone healing callus is minimal
C) Nutrient arteries supply the bones at the sites of muscle attachments
D) Osseoconduction is controlled by bone morphogenic proteins (BMPs)
E) Bone grafts without periosteum undergo less absorption
Ans: See end of page
Explanation
Intramembranous ossification occurs by direct deposition of bone within a
vascularized membranous template and occurs in the flat bones of the face,
calvarium and ribs. Long bones and iliac crest are developed by enchondral
ossification from a cartilage precursor.
The bone derives its blood supply by the following routes: periosteal,
apophyseal, epiphyseal and nutrient arteries; the latter supplies via the
medullary cavity. Primary bone healing occurs if bone ends are directly
apposed rigidly. The inflammatory and proliferative phases are less marked
and callus formation is minimal.
Bone graft healing occurs through incorporation (adherence of graft),
osseoconduction (bone graft as a scaffold along which vessels and
progenitor cells travel), osseoinduction (differentiation of mesenchymal cells
into osteocytes – this process is controlled by BMPs) and osteogenesis
(formation of new bone by surviving cells within the bone graft). Bone grafts
with intact periosteum undergo less absorption.
Ans: B

143
15) During the angiogenesis phase of wound healing, there is:

A) epithelial cell migration towards the angiogenic stimulus


B) maturation of fibroblasts
C) proliferation of keratinocytes
D) proteolytic degradation of the basement membrane of the parent vessel
E) altered MMP and TIMP profiles
Ans: See end of page
Explanation
Angiogenesis refers to the formation of new blood vessels from pre-existing
vessels at the site of injury. Four steps are recognised during this process;
these include:(i) migration of endothelial cells (not epithelial cells) towards
the angiogenic stimulus; (ii) proteolytic degradation of the parent vessel
basement membrane; (iii) proliferation of endothelial cells behind the leading
front of migrating cells; and (iv) maturation of endothelial cells. Fibroblasts
and keratinocytes play crucial roles in other stages of the wound healing
process such as granulation tissue formation and epithelialisation. Alteration
in MMP and TIMP profiles are seen during the reorganization phase of the
wound healing process and not during angiogenesis.
Ans: D

144
16) Myofibroblasts:

A) are present in the healing wound from about 72 hours after wounding
B) reach a maximum level within 4-5 days of initial injury
C) have structural property similar to that of collagens
D) help in the proliferation of endothelial cells
E) are responsible for healing of the wound in a scarless fashion as seen in
foetal wounds
Ans: See end of page
Explanation
Myofibroblasts appear in the wound approximately three days after
wounding and increase in number to a maximal level between the 10th and
21st days. Myofibroblasts are characterised, among others, by the
presence of stress fibres that contain alpha-smooth muscle actin and
indented nuclei, and thus have structural properties between those of a
fibroblast and a smooth muscle cell. Their main function is to contract the
granulation tissue and deposit new ECM. Although they promote wound
closure, myofibroblasts are also responsible for subsequent wound
contracture and scarring.
Ans: A

145
17) Which of the following statements regarding Rheumatoid Arthritis
is true?

A) Men are affected more commonly than women


B) Prevalence is highest in young adults
C) It has been associated with HLA-DR4
D) Garrod’s pads are part of the clinical presentation
E) The American College of Rheumatology criteria does not include
radiographic changes

Ans: See end of page

Explanation
Rheumatoid arthritis is a chronic systemic inflammatory disease that
predominantly affects synovial tissues. Women are more commonly
affected with a ratio of 3:1. The peak age of onset of the symptoms is
between 35 and 45 years. The HLA-D allele DR4 is associated with RA
patients.
The diagnosis of rheumatoid arthritis is often made in the presence of four
out of the seven of the American College of Rheumatology criteria. These
are: (i) Morning stiffness >1 hour located around joints and present for >6
weeks (ii) Arthritis in three or more joints present for >6 weeks (iii) Arthritis
of hand joints present for >6 weeks (iv) Symmetric arthritis for >6 weeks (v)
Physician observed rheumatoid nodules (vi) Serum rheumatoid factor
positive, and (vii) Typical radiographic changes.
Garrod’s pads are thickening of the dorsum of proximal interphalangeal
joints and are seen in Dupuytren’s disease.
Ans: C

146
18) Which medication should be avoided in the treatment of acute
attack of gout?

A) Ibuprofen
B) Indomethacin
C) Colchicine
D) Steroids
E) Allopurinol

Ans: See end of page

Explanation
Gout is caused by an inflammatory response to the formation of
monosodium urate crystals in joints secondary to hyperuricemia. Symptoms
manifest in the skin as tophi, which are painful masses that represent
precipitations of urate crystals. Acute gout presents with painful joints, which
are red, shiny and tender. The clinical features can mimic septic arthritis and
may result in failed attempts at drainage. A diagnosis is made with a history
of gout, chronic disease in other joints or elevated serum uric acid levels. If
the presentation is ambiguous, joint aspiration and microscopy to look for
negatively birefringent needle-like uric acid crystals will clinch the diagnosis.
In chronic gout, a secondary infection of an established tophus may be
present.
Medical management is the mainstay of treatment for an acute attack of
gout. Non-steroidal anti-inflammatory drugs are recommended as an initial
treatment to relieve acute symptoms. Colchicine, which works by interfering
with the uric acid crystals to reduce inflammation, can also be used as an
alternative. In a severe attack of gout, where NSAIDs and colchicine failed
to relieve the symptoms, corticosteroids can be used. Allopurinol is useful to
prevent gouty attacks. It is a xanthine oxidase inhibitor that disrupts the
enzyme responsible for converting purines into uric acid and hence will lower
uric acid levels. However, during an acute attack, allopurinol should be
avoided because sudden decrease in uric acid levels can prolong an existing
acute attack.
Ans: E

147
20) Which of these structures has the highest antigenicity and hence
higher propensity for rejection in a patient considered for hand
transplantation?

A) Skin
B) Muscle
C) Tendon
D) Nerves
E) Blood vessels
Ans: See end of page
Explanation
In composite tissue transplantation, highest antigenicity is for skin due to the
presence of antigen presenting cells, Langerhans cells, Keratinocytes
expressing MHC I and MHC II, Intercellular adhesion molecule 1 and pre
inflammatory cytokines. The leukocytes in the transplanted allograft -
especially the dendritic cells - form an important stimulus for the recipient
cells T lymphocytes to develop immune sensitivity. This could result in graft
rejection if no immunosuppression is used..
Ans: A

148
21) Choose the correct statement regarding musculocutaneous flaps:

A) These flaps are based on vessels running either within or near the fascia
B) Scapular and parascapular flaps are examples of musculocutaneous
flaps
C) Type III flaps are supplied by a single dominant pedicle
D) Sartorius and the tibialis anterior are examples of a type IV flap
E) Type V flaps have multiple segmental pedicles
Ans: See end of page
Explanation
Musculocutaneous flaps are based on perforators that reach the skin
through the muscle (fasciocutaneous flaps are based on vessels running
either within or near the fascia). Scapular and parascapular flaps are
examples of fasciocutaneous flaps. Based on their vascular supply, the
musculocutaneous flaps were classified by Mathes and Nahai in 1981. Type
I musculocutaneous flaps are supplied by a single dominant pedicle, which
enters the muscle near its origin or insertion. Type III flaps are supplied by
two vascular pedicles, each arising from a separate regional artery.
Sartorius, tibialis anterior, and the long flexors and extensors of the toes are
examples of type IV flap (supplied by multiple segmental pedicles). Type V
flaps have one dominant vascular pedicle and secondary smaller segmental
pedicles. Examples of this include latissimus dorsi and pectoralis major.
Ans: D

149
22) In the classification of fasciocutaneous flaps, the vascular basis of
a Mathes and Nahai Type A flap:

A) Has musculocutaneous perforators


B) May be identical to Cormack-Lamberty type B
C) Courses along an intercompartmental or intermuscular septum
D) Is unsuitable for free tissue transfer
E) Has multiple perforators
Ans: See end of page
Explanation
Both the Cormack-Lamberty and Mathes-Nahai classification categorize
fasciocutaneous flaps according to the origin of their fascial perforators:
Cormack-Lamberty
• Type A - Multiple perforators (with no discrete origin)
• Type B - Solitary perforator (single, usually direct perforator)
• Type C - Septocutaneous perforators (courses along an
intercompartmental or intermuscular septum)
• Type DO - steofasciocutaneous flap
Mathes-Nahai
• Type A - Direct cutaneous perforator (similar to the older term ‘axial
pattern’)
• Type B - Septocutaneous perforator
• Type C - Musculocutaneous perforators (indirect)
Ans: B

150
23) The blood supply of tensor fasciae latae flap comes from the:

A) Lateral circumflex femoral artery


B) Medial circumflex femoral artery
C) Superficial epigastric artery
D) Superficial external pudendal artery
E) Superficial external iliac artery
Ans: See end of page
Explanation
Tensor fasciae latae is a Type I muscle (Mathes and Nahai
classification) and is supplied by the transverse branch of the lateral
circumflex femoral artery, which is a branch of profunda femoris artery
(arises from common femoral artery). The tensor fasciae latae flap can be
raised either as a pedicled flap to cover trochanteric defects or as a free
flap.
The descending branch of the lateral circumflex femoral artery supplies the
antero-lateral aspect of the thigh and forms the vascular basis for the
anterolateral thigh (ALT) flap.
Ans: A

151
24) While raising an omental flap, one must consider that:

A) it can only be used reliably as a pedicled flap


B) it is based on the superior mesenteric artery
C) only part of the omentum can be used for reconstruction
D) it is relatively avascular
E) it is commonly pedicled on vessles on the right side
Ans: See end of page
Explanation
The greater omentum is a double-layered extension of peritoneum that
hangs down from the greater curvature of the stomach and proximal part of
the duodenum. It then folds back and attaches to the anterior surface of the
transverse colon. The entire or part of the omentum can be isolated on
either the right or left gastroepiploic vessels, which are branches of the
gastroduodenal artery and splenic artery respectively. Due to its excellent
microcirculation and phagocytic activity with in the lymphatics, the omental
flap can be employed to provide wound cover over contaminated or
irradiated wounds. For transfer as a pedicled flap, it is usually pedicled on
the right (the larger) gastro-epiploic vessels. The looped vascular pattern
allows it to be lengthened without losing its vascularity. Used as a free flap,
it can cover a large area such as the scalp.
Ans: E

152
25) The main blood supply of pectoralis major flap is from the:

A) Internal mammary artery perforators


B) Thoracoacromial artery
C) Lateral thoracic artery
D) Suprascapular artery
E) Thoracodorsal artery
Ans: See end of page
Explanation
The main blood supply for pectoralis major flap comes from the
thoracoacromial artery, which is a branch of the axillary artery (second
part). It is a type V muscle and the minor pedicles are from the internal
mammary perforators and the lateral thoracic artery. The thoracoacromial
artery pierces the clavipectoral fascia and divides into four branches –
clavicular, humeral, acromial and pectoral.
Pectoralis major muscle is commonly raised as a pedicled myocutaneous
flap for head and neck reconstruction, based on the thoracoacromial axis.
Based on the internal mammary perforators, it can be used as a turnover
muscle flap for covering sternal defects.
Ans: B

153
26) The Trapezius muscle is mainly supplied by:

A) Cranial part of the accessory nerve


B) Cervical plexus
C) Spinal part of the accessory nerve
D) Dorsal scapular nerve
E) Suprascapular nerve
Ans: See end of page
Explanation
The main nerve supply of the trapezius is from the spinal part of the
accessory nerve (C1-4). It also receives proprioceptive signals from the
cervical plexus (C3 and C4). Intra-operatively, the accessory nerve is
differentiated from the cervical plexus by the fact that it passes through the
substance of the sternomastoid muscle compared to the cervical plexus
which passes behind the sternomastoid. The main function of the trapezius
is to retract the scapula toward the midline. The upper fibres elevate the
whole scapula. The muscle can be tested by asking the patient to shrug the
shoulder against resistance.
Ans: C

154
27) While raising a groin flap, one must consider that:

A) it has a relatively short arterial pedicle


B) the vascular pedicle is a musculocutaneous perforator
C) it is based on the deep circumflex iliac artery
D) The lateral cutaneous femoral nerve courses along the vascular axis
E) it has a constant arterial anatomy
Ans: See end of page
Explanation
The vascular basis of the groin flap was described by McGregor and
Jackson in 1972, who introduced the concept of axial blood supply. Daniel
and Tayor reported the first microvascular transfer of the groin flap in 1973,
based on the superficial circumflex iliac artery. The superficial circumflex iliac
artery (SCIA) is a direct cutaneous artery, arising from the femoral artery in
close proximity to the superficial inferior epigastric artery, about 1-2cm
inferior to the inguinal ligament deep to the deep fascia of the thigh.
It is commonly used as a pedicled flap for resurfacing defects of the hand. It
is also used as a free fasciocutaneous flap, although the arterial anatomy is
variable and the arterial pedicle is relatively short. No sensory nerve
accompanies the vascular axis, but the lateral cutaneous femoral nerve
crosses the deep branch of the SCIA at the lateral border of the sartorius
muscle.
Ans: A

155
28) Hypertrophic scars:

A) are usually seen over the extensor surfaces


B) may result from excessive wound tension
C) commonly extend beyond the margins of the original scar
D) have a strong genetic predisposition
E) never improve with conservative treatment
Ans: See end of page
Explanation
Hypertrophic scar is a form of excessive healing resulting from
overproduction of several components of the healing process, namely
fibroblasts, collagen, elastin and proteoglycans. The incidence of
hypertrophic scars is highest in: wounds crossing flexor surfaces; wounds
crossing tension lines; areas of excessive wound tension and movement;
deep dermal burns and wounds left to heal by secondary intention (more
than 3 weeks). Hypertrophic scars are confined to the margins of the
original scar whilst keloids outgrow the wound (scar) area. Hypertrophic
scars do not have a genetic predisposition; keloids are thought to have a
genetic link. Hypertrophic scars respond to appropriate conservative
treatment, including topical steroids and compression therapy, and subside
with time.
Ans: B

156
29) All of the following are recognised treatment of CRPS (Complex
Regional Pain Syndrome) EXCEPT:

A) Sympathetic block
B) Pregabalin
C) Early passive joint movements
D) Pressure garments
E) Mirror therapy
Ans: See end of page
Explanation
Complex Regional Pain Syndrome is a term that has replaced the previously
used descriptions such as Reflex Sympathetic Dystrophy and Causalgia. It
is now divided into CRPS I (Reflex Sympathetic Dystrophy) and CRPS II
(Causalgia). CRPS I is characterised by pain, oedema, stiffness, and
discoloration out of proportion to the degree of initial injury. CRPS II has a
definable nerve injury in addition to the above signs and symptoms. There
are several treatments that have been tried for CRPS. These include
sympathetic blocks, drugs such as amytriptiline, gabapentin or pregabalin,
physiotherapy and mirror therapy. Pressure garments are useful in treating
hypertrophic scars and have no role in the treatment of CRPS.
Ans: D

157
30) Which amongst the following is an example of Type III muscle
(Mathes and Nahai classification)?

A) Rectus femoris
B) Rectus abdominis
C) Pectoralis major
D) Gastrocnemius
E) Soleus
Ans: See end of page
Explanation
Type III muscles have two dominant pedicles and the rectus abdominis
muscle is a classic example. It is supplied by both the superior and inferior
epigastric arteries, and a flap can be raised based on one of these vessels.
Based on the superior epigastic artery (continuation of the internal mammary
artery), abdominal tissue can be transferred as a pedicled TRAM flap
whereas the deep inferior epigastic artery (arises from the external iliac
artery) forms the vascular basis for free TRAM and DIEP flaps.
Gluteus maximus is another example of a Type III muscle. Rectus femoris
and gastrocnemius are examples of Type I muscles; pectoralis major an
example of Type V muscle and soleus is an example of Type II muscle.
Ans: B

158
31) While raising temporal fascia flaps, one must consider that:

A) The superficial temporal fascia is continuous with the SMAS posteriorly


B) The superficial temporal artery is a terminal branch of the internal carotid
artery
C) The auriculotemporal nerve lies posterior to the superficial temporal
vessels
D) The temporoparietal scalp has three layers
E) Superficial temporal fascia cannot be harvested as a free flap
Ans: See end of page
Explanation
The temporoparietal scalp is a five-layered structure comprising of
hairbearing skin, subdermal fibrofatty layer, superficial temporal fascia,
loose areolar layer and deep temporal fascia. The Superficial temporal
fascia is continuous with the SMAS inferiorly, the galea aponeurotica
superiorly, the frontalis muscle anteriorly and the occipitalis muscle
posteriorly. Both the superficial temporal fascia and deep temporal fascia
can be raised as separate flaps. The superficial temporal flap is supplied by
the superficial temporal artery, which is a terminal branch of the external
carotid artery, and the deep temporal flap is supplied by a proximal branch
of the superficial temporal artery (middle temporal artery). The
Auriculotemporal nerve accompanies the superficial temporal vessels
posteriorly and perforates the fascia superiorly to supply the overlying scalp.
Superficial temporal fascia can be harvested as a free flap for defects in the
hand where a gliding surface can be provided for the tendons. It can also be
used as a pedicle flap for ear reconstructions and filling orbital defects.
Ans: C

159
32) A 60° Z-plasty has a theoretical gain in length of approximately:

A) 25%
B) 50%
C) 75%
D) 100%
E) 120%
Ans: See end of page
Explanation
Z-Plasty is a technique widely used to lengthen or reorient a scar by
transposing two triangular flaps. They have several applications in plastic
surgical reconstruction and the theoretical gain in length depends on the
angles of the ‘Z’. In theory, for each 15° increase in angle a 25% length can
be gained. Therefore:
· A 30° Z-plasty: Will provide a 25% gain in length
· A 45° Z-plasty: Will provide a 50% gain in length
· A 60° Z-plasty: Will provide a 75% gain in length
· A 75° Z-plasty: Will provide a 100% gain in length
· A 90° Z-plasty: Will provide a 125% gain in length.
However, these figures are theoretical and the actual gain in length is
determined by the amount of laxity of the skin laterally. The commonly used
angle for a Z-plasty in surgical practice is 60°.
Ans: C

160
33) While raising a rectus abdominis flap, one must consider that:

A) It is a type V muscle flap


B) It is segmentally innervated
C) The Scarpa’s fascia is the fatty layer of the abdominal wall
D) The anterior rectus sheath above the arcuate line has contributions from
the aponeurosis of three muscles of the abdominal wall
E) Deep inferior epigastric artery enters the anterior rectus sheath posterior
to the arcuate line
Ans: See end of page
Explanation
The rectus abdominis is a type III muscle with dual blood supply from the
inferior and superior epigastric arteries. It is segmentally innervated with
short fibre length making it unsuitable for most functional transplants. The
superficial fatty layer of the superficial fascia of the anterior abdominal wall
is called Camper’s fascia whereas the deep layer is called Scarpa’s fascia;
the latter is attached to the deep fascia of the thigh, pubic arch and linea
alba. Below the arcuate line, the anterior rectus sheath has contributions
from the aponeuroses of external oblique, internal oblique and transversus
abdominis. The deep inferior epigastric artery is usually the dominant artery
of the anterior abdominal wall, arising from the external iliac artery
approximately 1cm above the inguinal ligament. It enters the posterior rectus
sheath above the arcuate line.
Ans: B

161
34) While raising a gastrocnemius flap, one must consider that:

A) It is a type IV muscle flap


B) The sural nerve courses over the lateral head and muscle bellies
C) It is supplied by branches of the medial and lateral sural arteries
D) The long saphenous vein courses within the furrow between the two
muscle bellies
E) The gastrocnemius muscle dorsiflexes the foot
Ans: See end of page
Explanation
Gastrocnemius is a type I muscle flap (one vascular pedicle, Mathes and
Nahai classification) that is commonly used to cover defects around the knee
joint. The medial sural artery arises from the popliteal artery approximately
1-cm proximal to the femoro-tibial articulation and enters the muscle hilum
after traversing 4 –5 cm. All the major branching occurs in the proximal one -
third to one - half of the muscle. The lateral sural artery arises in a more
distal location form the popliteal artery and has a similar branching pattern.
Both the short saphenous vein and sural nerve course within the furrow
between the two muscle bellies. The gastrocnemius muscle flexes the knee
and plantar-flexes the foot.
Ans: C

162
35) While raising a gracilis muscle flap, one must consider that

A) Is a type III muscle flap


B) Is supplied by the femoral nerve
C) Has its vascular supply arising mainly from the deep pudendal artery
D) Lies medial to the adductor longus
E) Inserts into the medial surface of the tibia posterior to the Sartorius
muscle
Ans: See end of page
Explanation
The gracilis is a type II muscle flap (Mathes and Nahai classification), i.e., it
has one dominant vascular pedicle (branch of medial femoral circumflex
artery) and minor pedicles (branches from superficial femoral artery and
profunda femoris artery). The adductor muscles of the thigh have their origin
from the pubic tubercle. The aponeurotic origin of the gracilis muscle is
located below the origin of the adductor longus muscle. Distally, it inserts
between the sartorius and semitendinous muscle onto the medial aspect of
the tibia. The gracilis muscle is supplied by the anterior branch of the
obturator nerve (L2-L4).
Ans: E

163
36) While raising a radial forearm free flap, one must consider

that:
A) It is a type C fasciocutaneous flap (Cormack-Lamberty classification)
B) It can only extend to 1/3 of the circumference of the forearm
C) The vascular pedicle is short if the skin paddle is placed distally
D) The radial artery is neighboured by the palmaris longus along its course
E) The basilic vein courses on the radial side of the flap
Ans: See end of page
Explanation
The radial forearm flap is a type C fasciocutaneous flap whose blood supply
is derived from multiple septocutaneous perforators following the course of
the radial artery. When an osseous segment is included, the flap is
considered a type D flap, in which the septocutaneous perforators supply
both the bone and the skin.
It is a commonly used free flap due to its ease of dissection and consistent
anatomy. The radial artery and its system of perforating vessels can supply
an extensive area of skin extending from the wrist crease to 3-4 cm above
the elbow occupying up to 2/3 of the circumference of the arm. If the skin
paddle is placed proximally, the vascular pedicle is short but donor site
closure is more easily accomplished. It is neighboured by flexor carpi
radialis on the ulnar side distally and the cephalic vein on its radial side.
Ans: A

164
37) The source vessel of the antero lateral thigh flap lies between

A) Vastus lateralis and rectus femoris


B) Vastus lateralis and tensor fascia lata
C) Tensor fascia lata and rectus femoris
D) Vastus intermedius and rectus femoris
E) Vastus intermedius and tensor fasia lata
Ans: See end of page
Explanation
The main arterial supply to the antero lateral thigh flap arises from the
descending branch of the lateral circumflex femoral artery. This artery lies
between vastus lateralis and rectus femoris muscles. The perforators from
the source vessel travel to skin either through the septum (septocutaneous
perforator) or vastus lateralis muscle (Musculocutaneous perforator). The
pedicle length is usually between 7 or 8 cm. Depending on the clinical need,
it can be raised either as supra facial or sub facial flap.
Ans: A

165
38) Whilst performing a sartorius switch, one must consider that

A) the Sartorius is a type II muscle


B) the entire muscle has to be mobilised to provide adequate coverage of
the vessels
C) the procedure is contraindicated in patients who have had radiotherapy
D) the muscle should ideally be harvested with a cuff of periosteum from
the anterior superior iliac spine
E) there is a risk of patellar instability following this procedure

Ans: See end of page

Explanation
Sartorius arises from the anterior superior iliac spine and inserts on the
upper part of the medial tibia in front of the gracilis and semitendinosus. It is
a type IV muscle flap according to the Mathes and Nahai classification,
supplied by segmental branches from the femoral artery.
The sartorius switch is an accepted surgical technique that is performed
during inguinal lymph node dissection. This procedure provides protection to
the femoral vessels in the adverse event of wound dehiscence. It is
especially useful in patients who have had previous radiotherapy. It is
important not to mobilise more than 1/3rd of the muscle to avoid
devascularisation. A small cuff of periosteum is often harvested from the
muscle origin to aid suturing to the pubic bone.
Ans: D

166
39) Which of the following statements is true regarding gluteus
maximus muscle?

A) It is a type V muscle
B) It is supplied by the internal pudendal artery
C) It is innervated by the superior gluteal nerve
D) Extends the knee as part of its action
E) Inserts into the greater femoral tuberosity
Ans: See end of page
Explanation
Gluteus maximus, the largest muscle in the body, arises from the ilium,
lumbar fascia and the sacrotuberous ligament, to insert onto the gluteal
tuberosity of the femur and upper end of iliotibial tract. It is supplied by both
the superior and inferior gluteal arteries, making it a type III muscle and is
supplied by the inferior gluteal nerve. It extends and externally rotates the
hip and also supports the extended knee through iliotibial tract.
Ans: D

167
40) Which amongst the following statements is true regarding acute
transplant rejection?

A) Occurs with minutes to hours after reperfusion


B) Mediated by the pre-formed antibodies in the recipient
C) Results in complement activation that causes damage to the vascular
D) Mediated by T-cells
E) The process is irreversible

Ans: See end of page

Explanation
Hyperacute rejection occurs immediately (usually within minutes to hours)
after reperfusion of the graft, mediated by the pre-formed antibodies to
ABO antigens or the MHC molecules present in the transplanted allograft.
This process results in complement Activation that causes damage to the
vascular endothelium of the allograft resulting in rapid thrombus formation
resulting in irreversible graft loss. Acute rejection happens days to weeks
after the transplant, mediated by T-cells. This process is reversible with
appropriate immunosuppressant treatment. Chronic rejection starts few
months to years after transplantation. This results in progressive fibrosis
because of inflammation and infiltration of mononuclear cells. This can be
controlled by immunosuppressants.
Ans: D

168
41) A 39-year-old machine operator presents with degloving of the
penile shaft following his trouser being caught in a rolling machine.
The main stay for reconstruction of penile shaft skin is

A) Split Skin Graft


B) Full Thickness Skin Graft
C) Pedicled groin flap
D) Peforator based lower abdominal flap
E) Islanded scrotal flap
Ans: See end of page
Explanation
Penile skin reconstruction requires a robust, thin pliable skin. In a
completely degloved penile shaft with healthy bed, a thick split skin graft
forms the main stay of treatment. A thorough debridement of all necrotic
skin and tissue is required in the first instance followed by application of a
skin graft.
Scortal flap if used is usually bipedicled, with staged division of the pedicles.
Groin and other pedicled flaps though have been tried can make the penile
skin bulky and may not be pliable. The colour match is not good compared
to SSG.
Ans: A

169
42) Polyglecaprone 25 is the main component of

A) PDS
B) Vicryl
C) Monocryl
D) Polysorb
E) Dexon
Ans: See end of page
Explanation

Monocryl is an absorbable monofilament suture made of a polymer of


polyglecaprone 25, which is a co polymer of glycolide and e-caprolactone.
The main advantage of monocryl over other sutures is its pliability that aides
superior handling. It gets absorbed completely in 90 to 120 days but loses
its tensile strength within 21 days.
PDS is the longest lasting absorbable monofilament suture and is made up
of polydioxanone. Vicryl is a braided absorbable suture made up of
polyglactin 910. Polysorb is a synthetic absorbable suture composed
glycolide/lactide copolymer. Dexon is a synthetic absorbable suture
composed of glycolic acid.
Ans: C

170
43) Synthetic sutures such as monocryl are absorbed by

A) Enzymatic degradation
B) Hydrolysis
C) Phagocytosis
D) Chemical degradation
E) Dissolution
Ans: See end of page
Explanation
There are essentially two processes involved in the absorption of
absorbable sutures. They are enzymatic degradation and hydrolysis. The
process depends on the type of absorbable sutures. Natural absorbable
sutures derived from mammalian intestine such as catgut are absorbed by
enzymatic degradation. Synthetic absorbable sutures such as vicryl,
monocryl and PDS are absorbed by hydrolysis. The process of absorption
occurs at a linear rate that results in loss of suture mass. The host
leukocytes remove the cellular debris and the suture material.
Ans: B

171
44) The incision for harvesting a sural nerve graft is best placed:

A) Medial to tendoachilles
B) Anterior to the medial malleolus
C) Over the lateral malleolus
D) Over the dorsum of the foot
E) Lateral to tendoachilles
Ans: See end of page

Explanation
Sural nerve is formed by the union of the branches of tibial nerve (medial
sural nerve and common peroneal nerve) through a sural communicating
branch. It passes down the posterolateral aspect of the leg, lying close to
short saphenous vein. It pierces the deep fascia near the middle of the leg
and lies subcutaneously. It continues along the lateral aspect of the foot and
the little toe. It is useful as a donor nerve graft and also as a site for nerve
biopsies site. It lies lateral to tendoachilles and can be rolled against the
tendon using a palpating finger.
The incision medial to tendoachilles is used for harvesting plantaris
tendon graft and an incision anterior to medial malleolus is placed to harvest
the saphenous vein.
Ans: E

172
45) The skin paddle of a free fibula flap can be made more robust by
harvesting part of which muscle along with the flap?

A) Gastrocnemius
B) Soleus
C) Peroneus longus
D) Peroneus brevis
E) Tibialis anterior
Ans: See end of page
Explanation
Free fibula flap was first introduced in 1970s and is used mostly in head and
neck reconstruction as a replacement for the mandible. It is supplied by
peroneal artery, which provides a nutrient artery to the bone. After its intial
description, it was modified to include a skin paddle. However, there was
reluctance amongst surgeons to use it as an osteocutaneous flap due to the
less reliable skin paddle. Better knowledge of the anatomy of this flap has
led to improved harvesting techniques and survival. The cutaneous
perforators can be septocutaneous or musculocutaneous. In the case of
musculocutaneous perforators, circulation to the skin paddle can be
improved by harvesting a portion of soleus muscle along with the fascial
septum to include more perforators.
Ans: B

173
Fundamental Principles In Plastic Surgery
Extended Matching Questions
1) ALLOPLASTIC MATERIALS
Options
A) Medpor®
B) Hydroxyapatite
C) Vitallium
D) Stainless steel
E) Poytetrafluoroethylene
F) Methyl methacrylate
G) Butyl cyanoacrylate
H) Polypropylene
For the questions given below, select the most appropriate answer from the
list of options above. Each option may be used once, more than once or
not at all.
Questions
1. The common name of porous polyethylene
2. An alloy of Cobalt and Chromium
3. Useful as bone cement in orthopaedic surgery

174
Answers

1-A
2-C
3-F

Explanation
Polyethylene is a synthetic polymer with a simple carbon chain backbone. It
is commonly used in the porous form and is commercially known as
Medpor®. It is easy to carve in the operating room and is used in nasal
augmentation and ear reconstruction.
Vitallium is an alloy of Cobalt and Chromium, and is more resistant to
corrosion than stainless steel. It also has a lower incidence of
hypersensitivity.
Methylmethacrylate is commonly used as bone cement in orthopaedics. It
produces an exothermic reaction when it hardens.
Polytetrafluoroethylene (PTFE) is used as a synthetic graft in vascular
surgery. Butyl cyanoacrylate is used as a tissue adhesive to seal skin
wounds. Polypropylene is a synthetic polymer that has a variety of uses
such as a suture material (prolene®) and as a mesh in hernia surgery.

175
2) LAYERS AND COMPONENTS OF THE SKIN

Options
A) Stratum lucidum
B) Stratum granulosum
C) Langerhans cells
D) Merkel cells
E) Melanocytes
F) Keratinocyte
G) Stratum spinosum
H) Glomus cells
For the questions given below, select the most appropriate answer from the
list of options above. Each option may be used once, more than once or
not at all.
Questions
1. This layer is present in palms and soles
2. Skin macrophages responsible for immunological function in the skin
3. This layer contains granules of keratohyalin

176
Answers

1-A
2-C
3-B
Explanation
The epidermis of the skin is composed of stratified squamous epithelium
with several distinct layers. The bottom most layer is the stratum basale or
stratum germinativum (basal layer), which contains the suprabasal
keratinocytes. The next layer of cells immediately above the basal layer is
the stratum spinosum (squamous or prickle cell layer). Above this layer is
the stratum granulosum (granular layer) which consists of one to three
layers of flattened cells containing keratohyalin granules. Stratum lucidum is
a clear layer which is present in glabrous skin of palms and soles. The
outermost layer of the epidermis is called the stratum corneum (cornified
layer; horny layer), composed of multiple layers of polyhedral cells. These
cells are anucleated and keratin-rich, forming the tough outer protective
layer of the skin. This layer is an effective barrier to most micro-organisms,
chemicals and fluids, although it is permeable to some substances (hence
topical treatments).
Keratinocytes are the principal cell type within the epidermis. In addition,
cells such as melanocytes, Langerhans cells and Merkel cells are also found
in the epidermis. Langerhans cells are dendritic cells which are located in
the upper part of the squamous layer. They detect and collect exogenous
antigen, process it and present it non-specifically to T-lymphocytes in the
skin or lymph nodes. Merkel cells are non-dendritic cells that lie near hair
follicles and unmyelinated nerve endings. They attach to adjacent
keratinocytes via the action of desmosomes, and when they come into
contact with nerve endings, they function as slow-adapting
mechanoreceptors and transducers for fine touch.

177
3) VASCULAR BASIS OF FLAPS

Options
A) Medial circumflex femoral artery
B) Ascending branch of Lateral circumflex femoral artery
C) Transverse branch of Lateral circumflex femoral artery
D) Descending branch of Lateral circumflex femoral artery
E) Posterior radial collateral artery
F) Radial recurrent artery
G) Perforators from internal mammary artery
H) Thoracoacromial artery
For the flaps given below, select the vascular supply for the flaps from the
list of options above. Each option may be used once, more than once or
not at all.
Questions
1. Tensor fasciae latae flap
2. Lateral arm flap
3. Deltopectoral flap

178
Answers

1-C
2-E
3-G
Explanation
Tensor fascia lata flap is a Type I muscle flap and it derives its blood supply
from the transverse branch of lateral circumflex femoral artery, a branch of
the profunda femoris artery. The descending branch of the lateral circumflex
femoral artery forms the basis of the anterolateral thigh flap.
Lateral arm flap is a Type C fasciocutaneous flap (Cormack-Lamberty
classification), supplied by the posterior radial collateral artery, which arises
from the profunda brachii artery. The perforators lie in the lateral
intermuscular septum between the biceps and triceps muscles. This flap can
also be raised as a osteofasciocutaneous flap that includes a small segment
of the postero-lateral aspect of the humerus, thus making it a Type D flap
(Cormack-Lamberty classification).
Deltopectoral flap, also known as Bakamjian flap, is perfused by three
perforators from the internal mammary artery of which the second
perforator forms the main vascular supply. With the advent of free tissue
transfer, this flap is used less commonly in head and neck reconstruction.
However, it can serve as a ‘life boat’ in the event of a failed reconstruction.

179
4) LOCAL ANAESTHETIC AGENTS

Options
A) Lidocaine
B) Bupivacaine
C) Cocaine
D) Amethocaine
E) Marcaine
F) Prilocaine
G)Tetracaine
H) Ropivacaine
From the list above, select the local anaesthetic agent that is most
appropriate for the statements below. Each option may be used once, more
than once or not at all.
Questions
1. The maximum recommended dose of this drug is 2.0 mg/kg
2. It blocks the reuptake of norepinephrine
3. May result in methaemoglobinaemia if used in high doses

180
Answers

1-B
2-C
3-F
Explanation
Local anaesthetics block the conduction of nerve impulses by preventing
sodium passage through the nerve sodium channels. They vary, in their
ability to cause cardiac toxicity. Lidocaine is ‘fast-in, fast-out sodium channel
blocker, whereas bupivaciane leaves the tissue slowly. Hence, bupivacaine
is more likely to precipitate cardiac arrhythmias. The maximum
recommended dose for plain lidocaine and bupivacaine are 4-5mg/kg and
2mg/kg respectively. When used with adrenaline, the maximum dosage of
lidocaine can be increased to up to 7mg/kg; however, the dosage of
bupivacaine remains the same.
Cocaine is used as topical anaesthetic commonly used in anaesthetising
mucous membranes of the nose and throat. It blocks the re-uptake of
norepnephrine and therefore has a sympathomimetic effect (all other local
anaesthetics cause sympathetic blockage). The maximum recommended
dose of cocanine is 1 mg/kg. At doses above this, it results in tachycardia,
hypertension, hyperthermia and seizures result.
A mixture of prilocaine (2.5%) and lidocaine (2.5%) is used as topical
anaesthetic (EMLA®) before minor skin procedures, venepuncture or to
anaesthetise split skin graft donor sites. In high doses, prilocaine can cause
methaemoglobinaemia.

181
5) TERMINOLOGY IN FLAP TRANSFER:

Options
A) Reverse flow
B) Prelamination
C) Ischaemia-reperfusion
D) Delay
E) Cantilever principle
F) Chimerism
G) Crane principle
H) Waltzing
From the above list, select the term used for the following descriptions.
Each option may be used once, more than once or not at all.
Questions
1. Free radical activation is a key component in this process
2. Distally-based flaps often carry this term
3. Temporary placement of a flap to convert a non-graftable wound into a
graftable wound bed

182
Answers

1-C
2-A
3-G
Explanation
During ischaemia-reperfusion injury, free radicals are generated and
activated, which induces apoptosis of cells. The no-reflow phenomenon
describes failure of a free tissue to reperfuse after re-establishing blood
supply. Endothelial injury, platelet aggregation and leakage of intravascular
fluid are thought to contribute to this phenomenon.
Distally-based flaps are often based on reverse (retrograde) flow, e.g.,
through deep palmar arch in distally based radial forearm fasciocutaneous
flap or the sural fasciocutaneous flap based on perforators from the
peroneal artery.
The ‘delay’ principle is aimed at enhancing the survival of a flap, usually
performed a week prior to flap transfer. This can be achieved by partially
elevating a flap (random delay) or dividing one of the two vascular sources
(axial delay). For example, the DIEA can be divided to increase the survival
of a pedicled TRAM flap. ‘Delay’ induces a level of ischaemia in the flap
without causing necrosis but ‘conditions’ the tissue so that the flap will
survive after later elevation.
In wound beds that are not suitable for skin grafting, the Crane principle can
be employed. This involves transferring a flap on a wound for a short period
of time, then returning it to its location so as to leave behind a layer of tissue
at the recipient site that is suitable for skin grafting.
Flap prelamination (aka: prefabrication), mainly used in H&N reconstructions,
involves introduction of additional layers into the flap prior to transfer. This is
done before partial/complete elevation of the flap and later suturing the flap
to form structures at the site of reconstruction.

183
6) MUSCLES OF THE THIGH

Options

A) Rectus femoris
B) Vastus lateralis
C) Vastus intermedius
D) Vastus medialis
E) Sartorius
F) Adductor longus
G) Adductor magnus
H) Gracilis
From the list of options above, select the most appropriate answer. Each
option can be used more than once or not at all:

Questions
1. Most superficial muscle of the medial side of the thigh
2. The hamstring part arises from the ischial tuberosity
3. Innervated by anterior division of obturator nerve and rotates flexed knee
internally

184
Answers

1–H
2–G
3–H
Explanation
Gracilis is the most superficial muscle of the medial side of the thigh and
arises from the inferior pubic ramus and adjoining ischial ramus. It inserts as
a cylindrical tendon into the upper part of the medial surface of the tibial
shaft just behind sartorius. It is innervated by the anterior division of
obturator nerve and rotates the flexed knee internally.
The adductor magnus is a composite muscle formed by the fusion of
adductor and hamstring muscles. The hamstring part arises from the ischial
tuberosity, passing downwards to the adductor tubercle of the femur with an
expansion to the medial supracondylar line. The adductor part arises from
the ischiopubic ramus, and inserts higher along the medial suprachondylar
line and linea aspera up to the gluteal tuberosity. The hamstring part is
supplied by the sciatic nerve and the rest by the posterior division of the
obturator nerve.

185
7) REGARDING SUTURE MATERIALS

Options
A) Vicryl
B) Silk
C) Catgut
D) Monocryl
E) Nylon
F) Prolene
G) PDS
H) Ticron
For the questions given below, select the most appropriate answer from the
list of options above. Each option may be used once, more than once or not
at all.
Questions
1. This suture material gets absorbed by proteolysis
2. This suture gets absorbed by hydrolysis in 180 to 210 days
3. This suture material has the lowest tissue reactivity and the lowest co-
efficient of friction compared to others.

186
Answer

1–C
2–G
3–F
Explanation
Catgut is the only natural absorbable suture that is in current clinical use (not
used in the UK). It is one of the earliest forms of suture material available
and is taken from the submucosa of sheep intestine. It is absorbed by
proteolysis. There are three sub-types of catgut depending on the time
taken for absorption. (i) The fast absorbing catgut gets absorbed in 2 to 4
weeks but maintains its tensile strength for only 5 to 7 days. (ii) Plain catgut
gets absorbed in 70 days but maintains tensile strength for 7 to 10 days. (iii)
Chromic catgut gets absorbed by around 90 days but maintains tensile
strength for 10 to 21 days. This property is achieved by treating plain catgut
with chromic salts.
PDS is a synthetic absorbable suture made from Polydioxanone. The
advantage of PDS is that it maintains its strength longer than other
absorbable sutures. It gets absorbed completely by hydrolysis in 180 to 210
days. Because of this nature, it is useful in providing prolonged dermal
support and hence minimizes scar stretching.
Prolene is a non-absorbable suture made from polypropylene. It has a lower
tensile strength and higher memory; hence more throws are required to
secure knots. It has the lowest tissue reaction compared to other non-
absorbable sutures.

187
8) SUTURES

Options
A) 5 days
B) 7 days
C) 14 days
D) 21 days
E) 28 days
F) 42 days
G) 90 days
H) 180 days
From the list above, select the most appropriate answer for the following
questions. Each option may be used once, more than once or not at all.
Questions

1. Number of days it takes for polyglactin sutures (Vicryl®) to lose 50% of


the original tensile strength?
2. Approximately 50% of the original tensile strength in Vicryl rapide®
(polyglactin) is lost by how many days?
3. Number of days it taken for Polydioxanone (PDS®) to be completely
absorbed?

188
Answers

1-D
2-A
3-H
Explanation

189
Vicryl® is a synthetic absorbable suture composed of copolymer of glycolide
and L-lactide. Progressive loss of tensile strength and eventual absorption
occur through hydrolysis. Approximately 50% of the original tensile strength
is remaining at 21 days and all original tensile strength is lost by about 35
days. Absorption is complete between 56 and 70 days (8-10 weeks).
Vicryl rapide® has more rapid loss of strength due to polymer material with
a lower molecular weight. Approximately 50% of the original tensile strength
is lost by 5 days and all original tensile strength is completely lost by 10 to
14 days. Absorption is complete by 42 days.
Polydioxanone (PDS®) is a delayed absorbable suture and elicits only a
slight tissue reaction during absorption. Approximately 50% of tensile
strength remains after 28 to 42 days. Absorption is minimal until about 90
days but becomes essentially complete between 180 and 210 days..

190
Hand and Upper Extremity
Single Best Answers
1) The time taken for the complete development of the upper limb in a
developing foetus is:
A) 28 days
B) 42 days
C) 56 days
D) 78 days
E) 96 days
Ans: See end of page
Explanation
At four weeks after conception, the cranio-caudal axis of the embryo is
established. The arm buds appear as a result of mitotic activity of the lateral
body walls. The blood vessels to the arm develop by 30 days and the
nerves by 36 days. Finger separation is completed by 47 days by the
process of apoptosis. The muscles, carpals and cartiliagenous elements
appear around 44 to 47 days. Every muscle is completely identifiable in a
foetus by seven weeks (49 days). The phalangeal development is complete
by 50 days and the entire upper limb is developed by 56 days.
Ans: C

191
2) Which amongst the following statements is correct regarding

the management of de Quervain’s tenosynovitis?


A) Analgesics and reassurance are sufficient since most cases are self –
limiting
B) Steroid injections can be used as initial treatment
C) Surgery involves release of the second extensor compartment
D) Splintage is as effective as surgery
E) The provocative test includes radial deviation of the wrist over flexed
thumb.
Ans: See end of page
Explanation
De Quervain’s disease is tenosynovitis of the first extensor compartment
(abductor pollicis longus and extensor pollicis brevis) at the wrist. Patients
present with pain over the dorsolateral aspect of the wrist and the
provocative test includes ulnar deviation of the wrist with the thumb fully
flexed (Finkelstein’s test).
The initial treatment of de Quervain’s tenosynovitis is steroid injections into
the first extensor compartment. Cases which do not respond to this will
require release of the first extensor compartment. While releasing the
compartment one has to careful not to injure the branches of superficial
radial nerve.
Ans: B

192
3) State the muscle behind which the roots of brachial plexus lie:

A) Longus colli
B) Semispinalis capitis
C) Scalenus anterior
D) Scalenus medius
E) Scalenus posterior
Ans: See end of page
Explanation
The roots of the brachial plexus arise from the anterior rami of C5, C6, C7,
C8 and T1. These are fibres that remain after contributing to the nerve
supply of the paravertebral and scalene muscles. All the roots emerge from
the spinal canal and lie behind the scalenus anterior muscle and emerge
between the scalenus anterior muscle and the scalenus medius muscle to
form the trunks. The trunks cross the lower part of the posterior triangle of
the neck to form the divisions behind the clavicle.
The structures which pass anterior to the scalenus anterior muscle are
phrenic nerve, the transverse cervical vessels, suprascapular artery and
subclavian vein.
Ans: C

193
4) Which amongst the following is muscles supplied by all the nerve
roots in the brachial plexus (C5-T1)?

A) Triceps
B) Latissimus dorsi
C) Deltoid
D) Pectoralis major
E) Biceps
Ans: See end of page
Explanation
The nerve supply to the pectoralis major is from the brachial plexus through
its medial and lateral pectoral nerves. The medial pectoral nerve arises from
the medial cord of the brachial plexus and the lateral pectoral nerve
originates from the lateral cord of the brachial plexus. Pectoralis major is the
only muscle in the upper limb supplied by all the segments of the brachial
plexus (C5, C6, C7, C8 and T1). The clavicular head of pectoralis major is
supplied by C5 and C6 and the sternal head is supplied by C7, C8 and T1.
Ans:D

194
5) The nerve roots that contribute to the formation of radial nerve are:

A) C5, C6
B) C5, C6, C7
C) C6, C7, C8
D) C8, T1
E) C5, C6, C7, C8, T1
Ans: See end of page
Explanation
The radial nerve is a continuation of the posterior cord, which is formed by
the union of all the posterior divisions of the trunks (C5, C6, C7, C8 and T1).
It crosses the posterior wall of the axilla, crosses the lattissimus dorsi
muslce and passes through the triangular space along with the profunda
brachii artery (formed by the teres major, long head of TRICEPS and
humerus). In the arm, it gives a branch to the long head of triceps and
supplies sensation to the posterior aspect of the arm by its branch called the
posterior cutaneous nerve of the arm. It then continues to supply all the
extensor muscles in the forearm (extensors of the wrist and fingers) both
directly and through its posterior interosseous nerve (motor) branch; the
other division, the superficial cutaneous branch, runs along the lateral aspect
of the forearm overlying the brachioradialis muscle and supplies sensation
over the 1st webspace.
Ans:E

195
6) The biceps muscle is supplied by:

A) Axillary nerve
B) Median nerve
C) Ulnar nerve
D) Musculocutaneous nerve
E) Radial nerve
Ans: See end of page
Explanation
The biceps brachii muscle is supplied by the musculocutaneous nerve (C5,
C6 & C7), the nerve of the flexor compartment of the arm. The
musculocutaneous nerve passes between the parts of the coracobrachialis
and lies between the biceps and brachailis muscles. It is accompanied by
the brachial artery and vein. At this level, it gives branches to both the
muscles (biceps and brachialis). After this level, it becomes a purely
cutaneous nerve and emerges at the lateral margin of the biceps tendon to
become the lateral cutaneous nerve of the forearm. The musculocutaneous
nerve also supplies the shoulder joint and the coracobrachialis muscles.
Ans: D

196
7) The nerve that crosses the brachial artery from the lateral to medial
side in the arm is the:

A) Ulnar nerve
B) Musculocutaneous nerve
C) Radial nerve
D) Axillary nerve
E) Median nerve
Ans: See end of page
Explanation
The median nerve arises from the medial and lateral cords of the brachial
plexus (medial root of median nerve and lateral root of median nerve) at the
lower border of the axilla. At its commencement, the nerve lies lateral to the
brachial artery. As it passes distally through the arm it lies anterior to the
brachial artery, thus crossing from lateral to medial side. At the level of the
elbow, it lies medial to the brachial artery. In the arm, the median nerve
does not give any branches to the muscles. It provides sympathetic supply
to the brachial artery and at a lower level, it gives a branch to the elbow
joint. Along with the ulnar nerve, it supplies muscles in the flexor
compartment of the forearm and the hand.
Ans: E

197
8) The nerve that passes between the two heads of the pronator teres
is the:

A) Musculocutaneous nerve
B) Superficial branch of the radial nerve
C) Median nerve
D) Ulnar nerve
E) Posterior interosseus nerve
Ans: See end of page
Explanation
The median nerve accompanies the brachial artery lying medial to it at the
elbow. It passes between the two heads of the pronator teres at the level of
the origin of the muscles. It is the main nerve of the flexor compartment of
the forearm. The other nerves that pass between the muscle heads in the
forearm are: (i) Ulnar nerve - passes between the humeral and ulnar heads
of the flexor carpi ulnaris, and (ii) Deep branch of the radial nerve (posterior
interosseous nerve) - passes between the two heads of the supinator after
providing branches to the supinator and the extensor carpi radialis brevis
(ECRB).
Ans: C

198
9) Which amongst the following tendons form the ulnar border of the
anatomical snuffbox?

A) Extensor carpi radialis brevis


B) Abductor pollicis brevis
C) Extensor pollicis longus
D) Abductor pollicis longus
E) Extensor pollicis brevis
Ans: See end of page
Explanation
When the thumb is fully extended there is a concavity, which appears on the
radial side of the wrist. This depression, called the anatomical snuffbox, lies
between the tendons of abductor pollicis longus and extensor pollicis brevis
on the radial side and the extensor pollicis longus tendon on the ulnar side.
The branches of the superficial radial nerve can be rolled over the tendon of
the extensor pollicis longus. The cephalic vein lies on the roof of the snuffbox
and radial artery on the floor. Four bony points are palpable in the snuffbox
and from proximal to distal they are radial styloid, scaphoid, trapezium and
base of the first metacarpal.
Ans: C

199
10) Which amongst the following structures pass through the carpal
tunnel?

A) Tendon of flexor carpi radialis


B) Tendon of flexor pollicis longus
C) Ulnar nerve
D) Tendon of flexor carpi ulnaris
E) Tendon of palmaris longus
Ans: See end of page
Explanation
The carpal bones are concave on the anterior aspect and a fibro-osseous
canal, called the carpal tunnel, lies between the flexor retinaculum and the
carpal bones. The flexor retinaculum is attached to the tubercle of scaphoid
and the ridge on trapezium laterally and pisiform and the hook of hamate
medially.
Ten structures pass through the carpal tunnel: all four tendons of flexor
digitorum superficialis, all four tendons of flexor digitorum profundus, the
tendon of flexor pollicis longus, and the median nerve.
Ans: B

200
11) The nerve that passes through the triangular space before
entering the arm is the

A) Axillary nerve
B) Ulnar nerve
C) Radial nerve
D) Dorsal Scapular nerve
E) Musculocutaneous nerve
Ans: See end of page
Explanation
The boundaries of the triangular spaces are as follows
Superiorly : Teres Major
Laterally : Humerus
Medially : Long head of the triceps
The structures that pass through the triangular spaces are the Radial nerve
(C5, C6, C7, C8 and T1) and the Profunda brachii vessels. The radial nerve
arises as a continuation of the posterior cord of the brachial plexus. It
crosses the lower border of the posterior axillary fold in front of latissimus
dorsi muscle and passes through the triangular space to enter the arm.
There is also a quadrangular space in the shoulder which is superior to the
triangular space. It is bounded superiorly by subscapularis, inferiorly by
teres major, medially by long head of triceps and laterally by long head of
triceps. Axillary nerve and posterior circumflex humeral vessels pass through
this space.
Ans: C

201
12) The pectoralis major muscle is supplied by:

A) Upper and lower subscapular nerves


B) Medial and lateral pectoral nerves
C) Dorsal scapular nerve
D) Thoracodorsal nerve
E) Long thoracic nerve
Ans: See end of page
Explanation
The nerve supply of the pectoralis major is from the brachial plexus through
its medial and lateral pectoral nerves. The medial pectoral nerve arises
from the medial cord of the brachial plexus and the lateral pectoral nerve
from the lateral cord of the brachial plexus.
Pectoralis major is the only muscle in the upper limb which is supplied by all
the roots of the brachial plexus (C5, C6, C7, C8 and T1). The clavicular
head of the pectoralis major is supplied by C5 and C6 and the sternocostal
head is supplied by C7, C8 and T1.
Upper lower subscapular nerves supply subscapularis muscle. Dorsal
scapular nerve supplies the rhomboids major and minor. Thoracodrosal
nerve supplies latissmus dorsi muscle and long thoracic nerve supplies
serratus anterior muscle.
Ans: B

202
13) The posterior interosseous nerve

A) supplies extensor carpi radialis longus muscle


B) provides innervation to the dorsal wrist skin
C) terminates in the third extensor comparmen
D) is a suitable donor nerve in digital nerve grafts
E) is harvested in a posterior interosseous flap to make it sensate
Ans: See end of page
Explanation
The radial nerve divides into a superficial sensory branch and a deep motor
branch named the posterior interosseous nerve at the level of the lateral
epicondyle. It travels between the two layers of the supinator to enter the
extensor compartment of the forearm. The extensor carpi radialis longus
muscle is supplied by the radial nerve before its division into the
aforementioned two branches. This is clinically relevant since in posterior
interosseous nerve palsy wrist extension is preserved albeit with radial
deviation of the wrist. It does not have any sensory branches except for
proprioception from the wrist joint. It terminates in the fourth extensor
compartment of the wrist. Due to the size match, it is a suitable donor nerve
for grafting digital nerves. While harvesting posterior interosseous flap, it is
important to preserve the nerve to retain the motor function of the extensor
muscles.
Ans: D

203
14) The presence of Horner’s syndrome in a patient with brachial
plexus injury strongly suggests:

A) C5 and C6 root avulsion


B) C6 and C7 root avulsion
C) C7 and C8 root avulsion
D) C8 and T1 root avulsion
E) T1 and T2 root avulsion
Ans: See end of page
Explanation
Horner’s syndrome (oculosympathetic palsy) is caused by damage to the
post-ganglionic fibres of the sympathetic trunk. The features of Horner’s
syndrome include miosis (constricted pupil), enopthalmos (recession of
eyeball within the orbit), upper eyelid ptosis and anhidrosis (loss of
sweating) on the ipsilateral side of the face. The presence of Horner’s
syndrome in a patient with brachial plexus injury strongly suggests the
avulsion of C8 and T1 roots. The sympathetic pre-ganglionic fibres exit the
spinal cord through the T1 root and ascend cranially through the sympathetic
chain to supply the head and neck region. Therefore, interruption of the
sympathetic supply through avulsion of the T1 root results in Horner’s
syndrome. The presence of Horner’s syndrome is a more reliable sign in
adult brachial plexus injury than in obstetric traction injury.
Ans: D

204
15) Which statement is true regarding compression neuropathies of
the upper limb?

A) The commonest compression neuropathy of the upper limb is cubital


tunnel syndrome
B) Wartenberg syndrome involves dorsal sensory branch of ulnar nerve
C) Guyon’s canal syndrome is mostly idiopathic in aetiology
D) The initial treatment for Guyon’s canal syndrome is steroid injection
E) Radial nerve compression is tested using resisted middle finger extension
test
Ans: See end of page
Explanation
The commonest compression neuropathy of the upper limb is carpal tunnel
syndrome, which is often idiopathic in aetiology.
However, Guyon’s canal syndrome, or compression of ulnar nerve in the
wrist, often has an extrinsic cause of compression or is due to external
trauma. The treatment of Guyon’s canal syndrome is surgical
decompression.
Wartenberg syndrome is a compression neuropathy of the superficial radial
nerve, when it lies under the brachioradialis muscle.
Radial nerve compression is tested using resisted middle finger extension
test.
Ans: E

205
16) An essential first step in the examination of a patient with brachial
plexus injury is to:

A) Test the motor function of small muscles in the hand


B) Ascertain localised tenderness in the nerve root exits along the cervical
spine
C) Test sensory loss
D) Evaluate active and passive movements of joints in the upper limb
E) Evaluate Tinel’s sign
Ans: See end of page
Explanation
It is important to examine the active and passive range of movements of all
joints before evaluating upper extremity musculature and testing for sensory
loss. This is because in a patient with established joint contracture or joint
stiffness, or who has had insufficient physiotherapy, the stiffness and
decreased range of movements could mislead the examiner. Furthermore,
(in patients with established joint contractures), undertaking complex surgery
to repair the nerves may be futile since poor range of joint movement may
compromise the benefits of surgery.
Ans:D

206
17) Which of the following statements is true regarding nerve palsy?

A) Tendon transfers in low median nerve palsies is aimed at creating


thumb flexion
B) In high median nerve palsies there is no additional motor deficit as
compared to low median nerve palsy
C) Low ulnar nerve palsies have more clawing than high palsies
D) Only muscles of MRC grade 3 or 4 are suitable for tendon transfers
E) Huber transfer uses flexor digiti mini brevis transfer for thumb opposition.
Ans: See end of page
Explanation
In low ulnar nerve palsies there is more clawing since the long flexors of the
ring and little fingers are functional. This is called ‘Ulnar Paradox’. In low
median nerve palsies a variety of tendon transfers have been described to
create thumb opposition, which is the main motor deficit. In high median
nerve palsies, there is additional paralysis of FPL and FDPs to index and
middle fingers. Muscles of MRC grade 4 or 5 should be used for tendon
transfers. The other criteria for suitability of a muscle for transfer include
adequate amplitude of excursion, suitable anatomical location, synergism
and expendability. Huber transfer uses abductor digiti minimi for thumb
opposition.
Ans:C

207
18) Which amongst the following muscles is supplied by the recurrent
(motor) branch of the median nerve?

A) 1st palmar interossei


B) Abductor pollicis longus
C) Extensor pollicis brevis
D) Adductor pollicis
E) Abductor pollicis brevis
Ans: See end of page
Explanation
The median nerve divides at the level of the carpal tunnel into lateral and
medial branches. The recurrent motor branch of the median nerve (T1)
arises from the lateral branch. It loops over the distal border of the flexor
retinaculum (transverse carpal ligament) to supply three thenar muscles -
abductor pollicis brevis, flexor pollicis brevis and opponens pollicis. The
lateral branch then divides into three digital branches – two to the thumb and
one to the index finger (radial border). The medial branch terminates into
two common digital nerves to the second and third web spaces (supplying
the ulnar border of the index finger, both borders of the middle finger and
radial border of the ring finger).
Ans:E

208
19) Camitz transfer for opponensplasty uses:

A) Abductor digiti minimi muscle


B) Flexor digitorum superficialis to ring finger
C) Extensor indices tendon
D) Palmaris longus
E) Extensor digiti mini tendon
Ans: See end of page
Explanation
The main motor deficit in low median nerve palsy is loss of thumb
opposition, which normally is brought about by combined action of the
opponens pollicis, abductor pollicis brevis and flexor pollicis brevis. There
are several techniques described for opponensplasty.
Camitz (1929) described the use of Palmaris longus for opponensplasty.
The other techniques described are abductor digiti minimi muscle (Huber
transfer), flexor digitorum superficialis to ring finger (Bunnell), and extensor
indices proprius (Burkhalter).
Ans: D

209
20) Which amongst the following nerves is used for nerve transfer in
brachial plexus reconstruction?

A) Trigeminal nerve
B) Spinal accessory nerve
C) Vagus
D) Glossopharyngeal nerve
E) Facial nerve
Ans: See end of page
Explanation
Nerve transfers are used in brachial plexus injuries when there are no
proximal stumps available for repair or for nerve grafting as in avulsion
injuries. Extra-plexal nerves (nerves outside the brachial plexus) are often
used as donor nerves. Examples of such nerves include:
· Spinal accessory nerve
· Phrenic nerve
· Intercostal nerves
Spinal accessory nerve (XI Cranial nerve) runs along the posterior triangle of
neck and supplies the sternocleidomastoid and trapezius muscles. For nerve
transfers, the terminal branch of the nerve distal to its connection with the
cervical plexus is used. This preserves all of the innervation to
sternocleidomastoid muscle and some of the innervation to trapezius
muscle. Spinal accessory nerve is usually transferred to neurotize
suprascapular nerve to restore shoulder function/movements.
Ans: B

210
21) The majority of Salter-Harris fractures belong to type:

A) I
B) II
C) III
D) IV
E) V
Ans: See end of page
Explanation
Salter-Harris classification is used to describe epiphyseal injuries (1963).
Salter-Harris fractures are unique to the paediatric population.
· The Type I fracture is a transverse fracture through the hypertrophic zone
of the physis, the growing zone is not injured.
· Type II is a fracture through the physis and the metaphysis. The epiphysis
is not involved. This is the commonest type (80%).
· Type III fractures are through the epiphysis and physis. These are intra-
articular fractures.
· In type IV the fracture passes through the epiphysis, physis and
metaphysis and therefore also intra-articular.
· Type V is a compression injury of the growth plate. This is a group difficult
to diagnose and have a poor functional outcome.
Ans: B

211
22) Which amongst the following statement is true regarding Bennett’s
fracture?

A) It affects the head of 1st metacarpal


B) The fracture occurs in dorso-volar axis
C) The anterior fragment remains unattached to ligaments
D) It is commonly treated conservatively
E) It is usually a stable fracture
Ans: See end of page
Explanation
The most common fracture of the thumb is an intra-articular fracture -
dislocation through the base of the first metacarpal.
Bennett’s fracture, described in 1882, is an eponym for fracture of the base
of the first carpometacarpal joint with dislocation of the thumb over the small
volar fragment (‘palmar beak’ fragment), which is attached to the ligament.
The mechanism of injury is an axial force through a partially flexed thumb
metacarpal. The strong pull of the abductor pollicis longus causes
displacement of the fragments.
Closed reduction and plaster cast may be effective in the treatment, if the
reduction can be maintained. Articular incongruity (>1mm) and recurrent
subluxation are indications for surgical treatment. This could be in the form
of percutaneous K-wires or fixation using mini-screws
Ans: B

212
23) Kienböck’s disease:

A) affects the scaphoid bone


B) affects mainly older people
C) is usually bilateral
D) has a strong association with ulnar minus variant
E) can be treated by ulnar shortening
Ans: See end of page
Explanation
Kienböck’s disease is avascular necrosis of lunate, affecting mainly young
and active people. It is usually unilateral and affects the dominant hand.
The exact aetiology of this disease is unknown. One of the theories by
Linscheid is a ‘nutcracker effect’ on the lunate from the capitate and the
radius in patients with a short ulna (ulnar minus wrist variant). The other
theory is that it is due to repeated minor trauma.
Ulnar minus variant is present in 23% of normal wrists and 78% of
Kienbock’s disease. Radial shortening is sometimes carried out to correct
the ulnar minus variant.
Ans:D

213
24) Steindler’s procedure in brachial plexus reconstructive surgery
involves:

A) transferring latissimus dorsi muscle for elbow flexion


B) fusion of elbow in functional position and release of triceps function
C) transfer of pectoralis major muscle for shoulder abduction
D) proximal advancement of the flexor muscles of the forearm to obtain
elbow flexion
E) transfer of trapezius muscle for shoulder abduction
Ans: See end of page
Explanation
In 1918, Arthur Steindler described a procedure to restore elbow flexion by
transposing the flexor-pronator muscles arising from the medial epicondyle
to a more proximal point on the humerus. This increases the movement arm
for elbow flexion sufficient to permit active flexion. This procedure however
provides only a moderate elbow flexion (ability to lift about 2kg). One of the
limitations of this procedure is the risk of flexion contractures developing in
the elbow.
The other procedures that can be considered for restoring elbow flexion are:
· Transfer of pectoralis major muscle
· Transfer of triceps muscle
· Transfer of latissimus dorsi muscle
· Free functioning muscle transfer..
Ans: D

214
25) The Sauve-Kapandje procedure in surgery for Rheumatoid Arthritis
involves:

A) hemi-resection of distal radio-ulnar joint with interposition of tendon graft


B) distal ulnar excision and synovectomy of the distal radio-ulnar joint
C) fusion of the radio-lunate joint
D) fusion of distal radio-ulnar joint and segmental resection of proximal ulna
E) total wrist fusion
Ans: See end of page
Explanation
There are several surgical procedures described for correction of wrist
deformities in Rheumatoid Arthritis. In patients with well-preserved mid-
carpal joints, a radiolunate fusion may be useful. In more advanced arthritis,
total wrist fusion may be attempted.
The Darrach procedure (distal ulnar excision) is often performed in
conjunction with synovectomy of the distal radio-ulnar joint (DRUJ).
This helps to correct DRUJ deformities and prevents attrition ruptures of the
extensor tendons by the prominent ulnar head. The distal ulnar resection
must be limited to less than 2 cm, to prevent instability of the remaining ulna.
The other surgical options in DRUJ arthritis include the Sauve-Kapandji
procedure (distal radioulnar fusion with segmental ulnar resection) and the
Bowers procedure (hemiresection of the DRUJ with interposition of a tendon
graft).
Ans: D

215
26) Dupuytren’s disease:

A) is more prevalent among the Oriental than the Viking race


B) most commonly affects the little finger
C) affects the radial side of the hand more commonly
D) may be associated with retroperitoneal fibrosis
E) does not involve Cleland’s ligaments
Ans: See end of page
Explanation
Dupuytren’s disease is a condition of unknown aetiology characterised by
contraction of the palmar or digital fascia. It affects various fascial
structures in the hand, but Cleland’s ligaments are not involved. It affects 1-
3% of the population of North-Europe and the USA. Although rare, it can
affect non-Caucasians as well. It is more prevalent in the Viking race. It is
three times more common in males and there is a strong hereditary
disposition. The ringfinger is the most commonly affected finger followed by
the little finger.
The following conditions are associated with Dupuytren’s disease:
Knuckle pads (Garrod’s pads); penile fibrous plaques (Peyronie’s disease);
and plantar fibromatosis (Lederhosen’s disease).
Retroperitoneal fibrosis, however, is not associated with Dupuytren’s
disease.
Ans: E

216
27) A 60-year-old lady sustained a Colle’s fracture of the right wrist,
which was treated in a Plaster of Paris cast. A few weeks after removal
of the plaster she noticed difficulty in extending the right thumb. The
most likely cause is:

A) compression neuropathy of posterior interosseous nerve


B) transection of posterior interosseous nerve by fracture fragment
C) adhesion of extensor pollicis longus tendon to fracture callus
D) rupture of extensor pollicis longus tendon
E) joint stiffness

Ans: See end of page


Explanation
Colle’s fracture was first described by Abraham Colle in 1814. Originally
described as low-energy extra-articular fracture of distal radius, it results
from a fall on an outstretched hand with an impact on the palm. The fracture
typically occurs through the distal metaphysis approximately 4 cm proximal
to the distal end of the radius. There is dorsal displacement of the distal
fragment, described as the dinner fork deformity. These fractures can be
treated conservatively in Plaster of Paris cast following reduction. One of
the complications is rupture of the extensor pollicis longus tendon, which
winds around the dorsal tubercle of Lister. This is believed by many to be an
attrition rupture over the fracture callus. However, some hold the view that it
is due to an avascular necrosis of the tendon due to disruption of its
periosteal blood supply.
Ans: D

217
28) The cause of joint deformity in arthritis due to Systemic Lupus
Erythematosis is:

A) Synovial proliferation
B) Cartilage destruction
C) Ligamentous laxity
D) Osteophyte formation
E) Deposition of amyloid in the joint cavity
Ans: See end of page
Explanation
Systemic lupus erythematosis (SLE) is an autoimmune disorder
characterized by involvement of various organ systems in the body including
skin, kidneys, central nervous system, musculoskeletal and cardiovascular
system. A malar rash (butterfly rash) is one of the pathognomic clinical sign
of this condition.
These patients also develop polyarthritis, mainly of the proximal and
interphalangeal joints (Jaccoud’s arthritis). The primary pathology is the
laxity of ligaments, which causes secondary joint deformity. Wrist and small
joints of the hand are affected and the involvement is usually symmetric. The
articular cartilage is unaffected in the initial stages.
Ans: C

218
29) Flexion and radial deviation of the distal interphalangeal joint of
the little finger is characteristic of :

A) Kirner’s anomaly
B) Camptodactyly
C) Madelung’s deformity
D) Symphalangism
E) Arthrogryposis
Ans: See end of page
Explanation
Kirner’s deformity is a skeletal deformity characterised by progressive
palmar and radial curvature of the distal interphalangeal joint of the little
finger. It commonly appears in adolescence and is more common in girls. It
can sometimes be inherited as autosomal dominant. This deformity does not
cause functional problems and treatment is aimed at improvement of the
appearance. Various treatment modalities include splintage, epiphyseodesis
or wedge osteotomy. The osteotomies can be dorsal closed wedge or volar
open wedge.
Ans: A

219
30) The function of the glomus body in the skin is:

A) Mechanoreception
B) Tactile perception
C) Temperature regulation
D) Pain and temperature perception
E) Vibration
Ans: See end of page
Explanation
The glomus body is an arteriovenous anastomotic apparatus situated in the
reticular layer of the dermis and is responsible for temperature regulation.
The arteriovenous anastomosis in the glomus body is called sucquet-hoyer
canal. Glomus bodies are numerous in the fingers and toes. The role of the
glomus body is to shunt blood away from the cutaneous circulation, thus
minimizing heat loss in cold weather. Glomus tumours are known to arise
from glomus bodies and are commonly seen in fingertips. However these
tumours can also occur in sites where glomus bodies are not seen.
Merkel cells are responsible for mechanoreception; tactile perception is
carried by Meissner corpuscles whereas pain and temperature are carried
by free nerve endings. Pacinian corpuscles are the receptors for vibration
sense.
Ans: C

220
31) The most sensitive diagnostic test for glomus tumours in the
finger is:
A) CT scan
B) MRI scan
C) PET scan
D) Ultrasound scan
E) Plain X-ray
Ans: See end of page
Explanation
Glomus tumours are hamartomas arising from the glomus apparatus in the
skin. They are usually only a few mm in diameter and present with pain,
point tenderness and cold sensitivity. Clinically, the lesion is often not visible
and the only evidence may be a faint blue mark over the nail.
Transillumination can be a useful clinical test to detect these lesions. They
commonly occur under the nail, but can also occur on the volar aspect of the
digit.
MRI can detect a glomus tumour as small as 5mm and is bright on T2
weighted images. These lesions are treated by surgical excision, which may
involve removing the nail and splitting the nailbed.
Ans: B

221
32) Which amongst the following structures is commonly used in
correcting claw hand deformity in ulnar nerve palsy?

A) Palmaris longus
B) Pronator teres
C) Extensor indices proprius
D) Brachioradialis
E) Extensor carpi radialis longus
Ans: See end of page
Explanation
There are several tendon transfers described for correcting claw hand
deformity. The aim of these operations is to recreate the action of intrinsic
muscles, which is flexion at the metacarpophalangeal joints and extension at
the interphalangeal joints. One such procedure is the Brand transfer using
Extensor Carpi Radialis Longus, lengthened using plantaris tendon grafts.
The tendon is routed from the palmar to the dorsal aspect of the fingers and
is sutured to the extensor apparatus. Another useful technique is the
Zancolli-Lasso procedure, which uses the slips of flexor digitorum
superficialis tendon to the ring finger and routing it through the A2 pulley
suturing it on itself.
Ans: E

222
33) Which of the following is a site for ulnar nerve compression?

A) Arcade of Frohse
B) Ligament of Struthers
C) Band of Osborne
D) Lacertus fibrosus
E) Flexor Digitorum Superficialis arch of origin
Ans: See end of page
Explanation
There are several possible sites of compression of the ulnar nerve in the
upper limb. The commonest site is in and around the elbow resulting in
cubital tunnel syndrome. Band of Osborne is the fascial condensation over
the cubital tunnel at the elbow, which is released during a cubital tunnel
decompression.
Another site is the medial intermuscular septum between biceps and triceps
muscle (Arcade of Struthers).
Arcade of Frohse is the origin of the supinator muscle and is a site of radial
nerve compression.
Ligament of Struthers is a ligamentous structure, which extends from a bony
spur above the medial epicondyle of the humerus to the medial condyle. It is
regarded as the accessory third head of coracobrachialis muscle and is a
site of median nerve compression. Lacertus fibrosus is the aponeurotic part
of the biceps tendon, which blends with the deep fascia of the forearm. This
structure as well as the arch of the origin of flexor digitorum superficialis
muscle are sites of compression of the median nerve.
Ans: C

223
34) Wartenberg sign is seen in:

A) Radial nerve compression at the wrist


B) Ulnar nerve injury at the wrist
C) Median nerve compression by pronator muscle
D) Thoracic outlet syndrome
E) Tenosynovitis of the first extensor compartment
Ans: See end of page
Explanation
There are several clinical signs described in ulnar nerve palsy. Clawing of
the ring and little fingers occur due to paralysis of the intrinsic muscles
(interossei and lumbricals), which normally aid in flexing the
metacarpophalangeal joints and extending the interphalangeal joints.
Paralysis of the adductor pollicis muscle results inability to adduct the thumb
against the index finger and on attempting adduction, flexion of the
interphalangeal joint ensues (Froment’s sign).
Abducted posture of the little finger due to paralysis of third palmar
interosseus is known as Wartenberg sign. This is due to paralysis of the
palmar interosseus, which normally adducts the little finger. The unopposed
action of extensor digiti minimi (which has a pull to the ulnar side) results in
abduction. This is not to be confused with Wartenberg syndrome, which is
compression of Superficial Radial nerve at the wrist due to tight wrist
watches or bracelets.
Ans: B

224
35) The main structure causing metacarpophalangeal joint contracture
in Dupuytren’s disease is:

A) Central cord
B) Spiral cord
C) Natatory ligaments
D) Lateral cord
E) Pretendinous cord
Ans: See end of page
Explanation
The normal fascial structures in the hand are called bands and are arranged
longitudinally, transversely and vertically. The palmar aponeurosis splits into
four or five slips and continue as the pretendinous bands. Each pretendinous
band has three different insertions. The first (superficial layer) terminates in
the dermis distal to the metacarpophalangeal joints, the second
(intermediate layer) passes deep to the natatory ligament and merges with
the lateral digital sheet. These are called the spiral bands of Gosset. The
third (deep layer) passes vertically on the side of the flexor sheath near the
A1 pulley. Bands, which are normal fascila structures, are known as cords
when they are pathological changes of Dupuytren’s disease. The contraction
of a pretendinous cord results in MCP joint contracture.
Central cord, lateral cord and spiral cord, all cause contracture of Proximal
interphalangeal joint. Natatory cords limit the abduction of the fingers and
decrease the span.
Ans: E

225
36) Hand dominance in children develops by the age of:
A) 6 months
B) 12 months
C) 18 months
D) 3 months
E) 36 months
Ans: See end of page
Explanation
Hand dominance is an attribute defined by the unequal distribution of fine
motor skills between the left and right hands. Individuals who have more
dexterity with the right hand are called right-handed. There are several
theories about hand dominance including biological and environmental
True hand dominance in children develops between 18 to 24 months and is
well established by 3 to 4 years. Any hand preference before the age of 18
months may indicate a unilateral functional impairment.
Ans: C

226
37) A Nalebuff Type I deformity of the thumb in Rheumatoid Arthritis is
characterised by:

A) Boutonierre deformity with adduction at carpometacarpal joint


B) Swan neck deformity
C) Boutonierre deformity
D) Swan neck with adduction at carpometacarpal joint
E) Game- keeper’s thumb
Ans: See end of page
Explanation
Classification system of rheumatoid arthritis of the thumb was first proposed
by Nalebuff and subsequently modified by Ratliff:
Type I: Boutonniere deformity
Type II: Boutonniere deformity with adduction at carpometacarpal (CMC)
joint
Type III: Swan neck with adduction at CMC joint
Type IV: Gamekeeper’s thumb
Type V: Swan neck with no CMC joint involvement
Type VI: Arthritis Mutilans
Ans: C

227
38) The normal conduction velocity of nerves in the upper limb is:

A) 75m/s
B) 100m/s
C) 120m/s
D) 50m/s
E) 25m/s
Ans: See end of page
Explanation
The normal conduction velocity of the nerves in the upper limb is between 50
-60m/s. It can be calculated by measuring the motor latency at two different
points and knowing the distance between them. Conduction velocity =
Distance between the points(cm) X 10 Time difference (ms) Conduction
velocity for motor nerves is influenced by temperature, age and myelin
sheath thickness
Ans: D

228
39) Martin-Gruber anastamosis is:

A) Abnormal communication between ulnar nerve and median nerve in the


forearm
B) Abnormal communication between posterior interosseous nerve and ulnar
nerve in the forearm
C) Abnormal communication between median nerve and ulnar nerve in the
forearm
D) Abnormal communication between ulnar nerve and median nerve in the
palm
E) Abnormal communication between radial nerve and median nerve in the
forearm
Ans: See end of page
Explanation

The Martin-Gruber anastomosis is an abnormal communication from the


median nerve (or the anterior interosseus nerve) to the ulnar nerve in the
forearm. In patients with Martin Gruber anastomosis, high ulnar nerve palsy
will result only in a sensory deficit and the observation that the intrinsic
muscles in the hand are innervated by the median nerve can be
demonstrated by blocking median nerve with a high median nerve block.
The Martin-Gruber anastomosis is present in 10 -17% of the population.
Riche-Cannieu anastomoses are motor communications between the median
and ulnar nerve in the palm and are present up to 70% of people.
Ans: C

229
40) Regarding Volkmann’s ischemic contracture

A) Results from untreated compartment syndrome in the hand


B) Flexor pollicis longus is never affected
C) Extensors digitorum communis are never involved
D) If mild, can be treated with dynamic splinting and tendon lengthening
E) Flexion of metacarpophalangeal joints is one of the features
Ans: See end of page
Explanation
Volkmann’s ischaemic contracture may result following untreated forearm
compartment syndrome and leads to an insensate hand with contractures:
forearm pronation, wrist flexion, hyperextended metacarphophalangeal joints
and interphalangeal joint flexion. Tendons with the longest excursion are
most severely affected, i.e. flexor digitorum profundi and flexor pollicis
longus. It can be graded in three levels of severity: Mild levels (wrist flexors)
can be treated with dynamic splinting and tendon lengthening procedures;
moderate grade (wrist and digital flexors) require tendon transfers after
excision of necrotic muscle; severe forms (wrist and digital flexors and
extensors) may require neurolysis as well.
Ans: D

230
41) Which of the following is NOT a cardinal sign as described by
Kanavel in flexor tenosynovitis?

A) Fusiform swelling
B) Ascending cellulitis
C) Flexed posture of digit
D) Tenderness over entire flexor tendon sheath
E) Disproportionate pain on passive extension
Ans: See end of page
Explanation
Tenosynovitis is an infection of the tendon sheath, which almost always
affects the flexor tendons in the hand. Most cases result from penetrating
injuries and the commonest micro-organism is Staphylococcus Aureus.
Alan B Kanavel described the four cardinal signs of flexor tenosynovitis.
They are: fusiform swelling of the digit, partially flexed posture of the digit,
tenderness over the flexor tendon sheath and pain on passive extension.
Although a frequent feature in infected fingers, cellulitis is not included in the
four cardinal signs described by Kanavel.
Ans: B

231
42) A felon is:

A) Often a result of chronic paronychia in nailbiters


B) Most commonly caused by Group A, beta-haemolytic streptococcus
C) A viral infection of the fingertip
D) Associated with juvenile rheumatoid arthritis
E) An abscess of the distal pulp of the thumb or finger
Ans: See end of page
Explanation
A felon is an abscess of the distal pulp of a thumb or finger. The pulp has
multiple closed compartments and abscess formation within them results in
rapid swelling and pain. Delayed presentation may lead to necrosis or
extension of the infection into distal phalanx or flexor sheath. It often
presents following a puncture wound. The commonest pathogen is S.
Aureus.
Herpes simplex virus infection of the fingers presents as herpetic whitlow,
for which, incision and drainage of which is generally contraindicated. The
pulp of the affected finger is not as tense as in a felon.
Ans: E

232
43) What is the hand anomaly seen in Poland’s syndrome?

A) Symbrachydactyly
B) Camptodactyly
C) Clinodactyly
D) Symphalangism
E) Phocomelia
Ans: See end of page
Explanation
Symbrachydactyly is the current nomenclature for atypical cleft hand. It is
characterised by a ‘U’shaped cleft and are unilateral and sporadic. Feet are
rarely involved. They can be seen in Poland’s syndrome. This syndrome
consists of absence of the sterno-costal head of pectoralis major along with
congenital hand anomalies, the most common type is symbrachydactyly.
Ans: A

233
44) Wartenberg’s syndrome is a compression of

A) the superficial radial nerve at the wrist


B) the ulnar nerve at the wrist
C) the posterior interosseous nerve under the arcade of Frohse
D) the anterior interosseous nerve at the wrist
E) the ulnar nerve under the arcade of Struthers
Ans: See end of page
Explanation
Wartenberg syndrome, named after a German neurologist who described
cases of inflammation of the superficial radial nerve in 1932, arises as a
result of compression of the superficial radial nerve at the wrist. It often
occurs in patients who wear tight watches or bracelets. It is also seen in
patients who use the arm in a pronated position for extended periods and in
individuals who do repetitive flexion and pronation of the forearms. During
pronation, the superficial radial nerve is compressed between the tendons of
the brachioradialis and the extensor carpi radialis longus. This condition can
mimic DeQuervain’s disease and even Finklestein’s test can be falsely
positive.
Non-operative treatment for Wartenberg’s syndrome is to maintain the
affected arm in a supinated position using appropriate splintage. Operative
treatment involves incising the fascia between these two tendons or dividing
the tendon slip where the nerve exits through the brachioradialis tendon.
Ans: A

234
45) Inability to make an ‘O’ sign with the thumb and index finger could
indicate:

A) Wartenberg’s syndrome
B) Posterior interosseous syndrome
C) Anterior interosseous syndrome
D) Radial tunnel syndrome
E) Pronator syndrome

Ans: See end of page

Explanation
Inability to make an ‘O’ sign indicates paralysis of flexor pollicis longus and
flexor digitorum profundus of the index finger. Both these muscles are
innervated by the anterior interosseous nerve, the deep branch of the
median nerve. This is a classical sign of patients presenting with anterior
interosseous nerve syndrome.
The anterior interosseous nerve is a branch of the median nerve in the
forearm. It descends on the interosseous membrane to the wrist and
supplies muscles of the deep flexor compartment: radial half of flexor
digitorum profundus, flexor pollicis longus and pronator quadratus. It is
sensory to the wrist and carpal joints.
Ans: C

235
46) A 35-year-old woman presents with a six-month history of pain in
her right forearm. On examination, there is weakness of flexion of the
interphalangeal joint of thumb and the distal interphalangeal joint of
index finger. Sensation to the hand is preserved. What is the most
likely diagnosis?

A) Carpal tunnel syndrome


B) Pronator syndrome
C) Anterior interosseous nerve syndrome
D) Cubital tunnel syndrome
E) Radial tunnel syndrome

Ans: See end of page

Explanation
The anterior interosseous nerve, which is the deep branch of the median
nerve, supplies the flexor digitorum profundus to the index and middle
fingers, flexor pollicis longus and pronator quadratus. Compression of the
anterior interosseous nerve (anterior interosseous nerve syndrome) can be
due to tendinous bands, accessory muscles (e.g., Gantzer’s muscle, which
is an accessory FPL) or due to an aberrant radial artery. This can be
differentiated from pronator syndrome that results from compression of the
median nerve before its division and is associated with paraesthesia along
the median nerve distribution.
The onset of symptoms in anterior interosseous nerve syndrome is usually
insidious and slowly progressive. However, acute onset of anterior
interosseous nerve palsy is a result of viral mononeuritis, also known as
Parsonage-Turner syndrome. This usually resolves spontaneously but in
cases that fail to improve tendon transfers may be indicated.
Ans: C

236
47) Which of the following statements is true regarding the treatment
of Parsonage-Turner syndrome?

A) Urgent decompression is indicated


B) Early tendon transfers provide better functional results
C) Corticosteroids are contra-indicated
D) Antiviral medication may benefit
E) Gabapentin has been proven to be effective if started early
Ans: See end of page
Explanation
Parsonage–Turner syndrome, also known as brachial plexitis, is a non-
specific inflammation of the brachial plexus. It presents with flu like
symptoms with sudden onset of pain and brachial plexus palsy. However, in
a mononeuritis form it can present as a peripheral nerve palsy of which the
commonest is anterior interosseous nerve palsy.
Usually, the symptoms resolve spontaneously. Antiviral treatment may be
beneficial if started early. High doses of corticosteroids and antiviral
medications may be beneficial if started early. Patients whose symptoms fail
to resolve may need tendon transfers but these are usually performed after
a year of conservative treatment.
Ans: D

237
48) Volkmann’s test is used in Cerebral palsy to assess:

A) tightness of digital flexor tendons


B) tightness of wrist flexors
C) tightness of intrinsic muscles
D) tightness of pronator muscle
E) tightness of biceps muscle
Ans: See end of page
Explanation
Tightness of flexor tendons in the digits can be tested using Volkmann’s test.
This test is carried out by holding the fingers in extension with the wrist
flexed, and then the wrist is gradually extended. In normal hand, full wrist
extension can be achieved with the fingers in extension. However, when
there is tightness of FDS or FDP tendons, full wrist extension is not
possible.
FDS and FDP can be separately tested in this test. If PIP joint cannot be
passively straightened with the wrist in extension, it is due to tightness of
FDS and if PIP joints can achieve full passive extension but DIP joints cannot
be fully extended, then it is due to tightness of FDP.
Tightness of flexor tendons of the fingers is an indication for surgical
intervention.
Ans: A

238
49) Which of the following is NOT a deformity typically seen in
cerebral palsy?

A) Pronation of forearm
B) Wrist flexion
C) Finger flexion
D) External rotation of the shoulder
E) Thumb in palm deformity
Ans: See end of page
Explanation
Cerebral palsy is sequelae of irreversible perinatal brain injury and there is
variable involvement of motor and sensory systems and intelligence.
There are several deformities typical of cerebral palsy. These can be based
on the anatomical region in the limb:
Shoulder: Internal rotation, spasticity and possible contracture
Elbow: Flexion deformity with spasticity
Forearm: Pronation due to tightness of pronator teres and pronator
quadratus
Wrist: Flexion deformity often due to tightness of FCU
Fingers: Flexion deformity due to tightness of FDS and FDP, intrinsic
tightness
Thumb: Thumb in palm deformity.
A systematic evaluation of the deformities is important to plan treatment in
these patients which include Botox injections, tendon release, tendon
transfers and bony fusions.
Ans: D

239
50) Classic Boyes transfer for radial nerve paralysis uses

A) flexor carpi ulnaris tendon to provide finger extension


B) flexor carpi radialis tendon to provide wrist extension
C) flexor digitorum superficialis of the middle finger to provide finger
extension
D) pronator teres to provide finger extension
E) palmaris longus tendon to provide thumb extension
Ans: See end of page
Explanation
In 1960, Boyes described a set of tendon transfers for radial nerve palsy.
The combination of transfers he described is as follows:
1. PT to ECRL and ECRB
2. FDS (middle finger) to EDC
3. FDS (ring finger) to EIP and EPL
4. FCR to APL and EPB
His reasoning was that the FCU is a more important tendon to be preserved
for wrist flexion than the FCR and that the greater excursion (70 mms) of the
superficialis tendons would make it the ideal motor for finger extensors. It
allows simultaneous wrist and finger extension, independent thumb and index
finger extension, and does not require the tenodesis effect of wrist flexion to
achieve full extension. The disadvantages include weakness of grip because
of loss of FDS to middle and ring fingers, and potential swan-neck deformity
or PIPJ contracture..
Ans: C

240
51) Radial club hand

A) is bilateral in less than 10%


B) has autosomal dominant inheritance
C) is associated with cardiac defects in more than 50% of patients
D) is bilateral with hypoplastic thumb in TAR (Thromobocytopenia Absent
Radius) syndrome
E) has an elbow that is held in a fixed flexion deformity
Ans: See end of page
Explanation
Radial club hand, a type of longitudinal deficiency, is bilateral in about 50%
of cases. The aetiology is not clearly understood and could be either genetic
or environmental. It is associated with cardiac defects in about 10-13% of
cases, the commonest of which is Holt-Oram syndrome (septal defects). If
present bilaterally with hypoplastic thumbs, one should suspect TAR
(Thromobocytopenia Absent Radius) syndrome. The other associated
anomalies include Fanconi’s anaemia (progressive pan-cytopenia) and
VATER/VACTERL (Vertebral, Anal, Cardiac, Tracheal, Oesophageal, Renal
and Limb anomalies) syndrome. The elbow is usually held in extension and is
often a fixed deformity.
Ans: D

241
52) The classification system that describes the different types of
constriction ring syndrome is

A) Swanson
B) Bayne and Klug
C) Stellings
D) Buck-Gramcko
E) Patterson
Ans: See end of page
Explanation
Congenital constriction ring syndrome presents with tight rings across either
the digits or whole limb, resulting in varying degrees of vascular or lymphatic
compromise. This can even result in auto amputation of the finger tips in-
utero and subsequent fusion of the tips resulting in what is known as
acrosyndactyly.
Patterson classified this into four types based on the degree of severity:
• Type 1 is a mild groove which can be either transverse or oblique
• Type 2 is a deeper groove with an abnormal distal part
• Type 3 presents with acrosyndactyly
• Type 4 is complete amputation of the part distal to the constriction.
Ans: E

242
53) Camptodactyly

A) does not affect multiple digits


B) is commonly unilateral
C) in the infantile type is associated with a female preponderance
D) does not usually affect function
E) can be treated surgically with good outcomes
Ans: See end of page
Explanation
Camptodactyly is a congenital flexion deformity of the fingers most
commonly affecting the little finger. It can also affect the ring, middle and
index fingers in decreasing order of frequency. It is commonly bilateral and
can be inherited or sporadic.
There are two types of camptodactyly; one which affects infants with equal
incidence in boys and girls, and the other occurring in adolescence and
common in girls. Patients seek treatment mainly for cosmetic reasons as
function is rarely impaired. The results of surgical release are not often
satisfactory.
Ans: B

243
54) Which amongst the following statements is true regarding Juvenile
Rheumatoid Arthritis?
A) Rarely occurs in a pauci-articular form
B) Rheumatoid factor is positive in about 90% of patients
C) Metacarpals show radial deviation at carpometacarpal joints
D) The fingers are radially deviated at the metacarpophalangeal joints
E) Swan neck deformities are common
Ans: See end of page
Explanation
Juvenile rheumatoid arthritis (JRA) may present in a polyarticular or pauci-
articular form. About 80% of children are seronegative (Rheumatoid factor
negative) as opposed to 30% in classic Rheumatoid Arthritis. Deformities in
JRA are different to that seen in the adult form. In JRA, there is ulnar
deviation of the metacarpals at the carpometacarpal joints whereas in
classic Rheumatoid Arthritis it is radially deviated. Likewise, in JRA there is
radial deviation of the fingers at the metacarpophalangeal joints whereas it
is deviated towards the ulnar side in adult Rheumatoid Arthritis. Swan-neck
deformities, tendon rupture and nerve compression occur rarely in JRA
Ans: D

244
Hand and Upper Extremity
Extended Matching Questions
1) NERVES IN THE UPPER LIMB
Options
A) Median nerve at elbow
B) Superficial branch of radial nerve
C) Posterior interosseous nerve
D) Radial nerve at elbow
E) Median nerve at wrist
F) Ulnar nerve at wrist
G) Ulnar nerve at forearm
H) Musculocutaneous nerve
For each of the cases described below, select the nerve most likely
injured. Each option can be used once, more than once or not at all.
Questions
1. A 26-year-old bartender with multiple stab wounds to his right upper limb.
He complains of numbness over his dorsal first web space and has difficulty
in extending his thumb and fingers.
2. A 35-year-old computer engineer with multiple lacerations over his left
forearm and wrist from punching a glass window. He has mild clawing and
impaired sensation the volar aspect of the ring and little fingers. Sensation
over the dorsum of the hand is intact. There is also weakness of abduction
and adduction of the fingers.
3. A 20-year-old man with laceration over the right upper arm and elbow. He
has difficulty flexing the elbow and also has numbness along the radial side
of right forearm.

245
Answers

1-D
2-F
3-H
Explanation
Radial nerve divides at the elbow into superficial radial nerve and posterior
interosseous nerve. Superficial radial nerve supplies sensations to dorsum of
the first web and posterior interosseous nerve supplies the extensor
muscles. In this case it has to be the main trunk of the radial nerve that is
divided.
The main branch of ulnar nerve supplies the intrinsic muscles of the hand
and also gives sensations to the volar aspect of ring and little fingers. The
dorsal aspect of the ulnar side of the hand and the ulnar two fingers are
supplied by the dorsal branch of ulnar nerve which is given off 5cm proximal
to wrist. Therefore in this case it should be an injury of the ulnar nerve at the
wrist.
Musculocutaneous nerve supplies biceps, brachialis and coracobrachialis
muscles and continues as lateral cutaneous nerve of forearm.

246
2) CONGENITAL HAND ANOMALIES

Options
A) Clinodactyly
B) Kirner’s deformity
C) Madelung’s deformity
D) Camptodactyly
E) Symphalangism
F) Arthrogryposis
G) Brachysyndactyly
H) Symbrachydactyly
For the descriptions given below, select the most appropriate term from the
list of options above. Each option can be used once, more than once or not
at all.
Questions
1. Ulnar head is characteristically prominent in this deformity
2. This deformity is characterised by palmar and radial deviation of the distal
interphalangeal joint.
3. Absent joint crease and stiffness of fingers are characteristic of this
Deformity

247
Answers

1-C
2-B
3-E
Explanation
Madelung’s deformity is characterised by inadequate growth of the distal
radial physis in its anterior ulnar segment. It usually presents between 8 to
12 years and is characterised by dorsal subluxation and prominence of ulnar
head.
Kirner’s deformity is characterised by palmar and radial deviation of the
distal interphalangeal joint of the little finger. It usually appears between 7-
14 years of age and is more common in girls. Treatment is by splintage,
epiphyseodesis or wedge osteotomy.
Symphalangism is characterised by congenital stiffness of the joints and
affects the proximal interphalangeal joints. It can occur as a hereditary form
where the ulnar digits are more affected or as a non-hereditary form.

248
3) COMPRESSION NEUROPATHIES OF THE UPPER LIMB

Options
A) Band of Osborne
B) Ligament of Struthers
C) Arcade of Struthers
D) Vascular leash of Henry
E) Arcade of Frohse
F) Lacertus fibrosus
G) Pronator teres
H) Arch of flexor digitorum superficialis origin
For each of the description below, select the most correct answer from the
list of options above. Each option may be used once, more than once or
not at all.
Questions
1. This structure forms the roof of the cubital tunnel and is a potential site for
compression of ulnar nerve
2. This structure is a thickening of the medial intermuscular septum
between the biceps and triceps, and a site for ulnar nerve compression
3. This structure is the proximal margin of supinator muscle and a site of
compression of posterior interosseous nerve

249
Answers

1-A
2-C
3-E
Explanation
There are several potential sites for nerve compression in the upper Limb
(from distal to proximal).
Median nerve can be compressed at the following sites:
a) Carpal tunnel
b) Between the two heads of pronator teres
c) Underneath Lacertus fibrosus or the aponeurotic portion of the biceps
tendon
d) Fibrous arch of origin of flexor digitorum superficialis
e) Ligament of Struthers (a ligamenous structure spanning the medial
epicondyle and a supra-trochlear spur; it is considered a vestigial third head
of coracobrachialis)
Ulnar nerve can similarly be compressed at the following sites:
a) Guyon’s canal in the wrist
b) Cubital tunnel which is roofed by a ligamentous structure called Band of
Osborne
c) A thickening of the medial intermuscular septum called arcade of
Struthers
Radial nerve can be compressed at the following sites:
a) Extensor carpi radialis brevis muscle
b) Arcade of Frohse or the proximal edge of supinator origin
c) Vascular leash of Henry or recurrent radial vessels

250
4) UPPER LIMB EXAMINATION

Options
A) Finkelstein test
B) Phalen’s test
C) Kirkwood-Watson test
D) Bouvier manoeuver
E) Spurling’s test
F) Roo’s test
G) Lichtman test
H) Allen’s test
For the following descriptions, select the most correct answer from the list
of tests above. Each option may be used once, more than once or not at
all.
Questions
1. This test is positive in ulnar mid-carpal instability
2. This test is used to detect cervical root compression
3. This test is used to assess ulnar claw hand

251
Answers
1-G
2-E
3-D
Explanation
Lichtman test is used to evaluate ulnar midcarpal instability. With the wrist in
pronation and palmar flexion, the examiner palpates the triquetrum 2cm
distal to the ulnar head. On radial deviation of the wrist the triquetrum
becomes prominent and on ulnar deviation it disappears under the
examiner’s finger. If it initially becomes more prominent, Lichtman test is
positive, suggesting attrition of the triquetro-hamato-capitate ligament.
Spurling’s test is a test for cervical root compression. The head is tilted
towards the involved side and pressure applied to the top of the head. In
positive cases paresthesia along the involved dermatomes will be
experienced.
Bouvier manoeuver is used for assessment of ulnar claw hand.
Examiner’s fingers are used to block hyperextension of the
metacarpophalangeal joints and the patient is asked to extend the inter-
phalangeal joints.
Bouvier positive: Extension is possible at interphalangeal joints. Anti-claw
procedures should give good results.
Bouvier passive positive: Fingers cannot be extended actively but can be
passively straightened. This is due to attenuation of the central slip caused
by prolonged contracture.
Bouvier passive negative: Fingers are in fixed flexion deformity as a
consequence of joint contractures.

252
5) FINGERTIP RECONSTRUCTION

Options
A) Littler flap
B) Moberg flap
C) Atasoy-Kleinert flap
D) Venkataswamy flap
E) Kutler flap
F) Tranquilli-Leali flap
G) Foucher flap
H) Kite flap
From the list above, select the flap name for the following descriptions.
Each option may be used once, more than once or not at all.
Questions
1. Bilateral V to Y advancement
2. Raised on the ulnar side of the middle finger and transferred to the thumb
3. Triangular oblique island flap advancement from the less damaged side

253
Answers

1-E
2-A
3-D
Explanation
Kutler (1947) described a method of V to Y advancement based on two
triangular flaps from the sides of the finger. The dorsal incision leaves a
millimetre of skin at the margin of the nailfold and extends proximally in the
midlateral line to the interphalangeal joint crease or just proximal to it. The
volar incision is only through skin. This technique is useful for clean
transverse amputations of the fingertip exposing bone.
Littler (1960) described a heterodigital neurovascular island flap for thumb or
index fingertip reconstruction. The flap is taken from the pulp of the middle
finger based on the ulnar side. It can also be taken from the radial side of
the ring finger. For the flap to reach the thumb, the digital artery to the
adjacent finger will have to be sacrificed and the nerve dissected proximally.
Venkataswamy (1980) described an oblique triangular flap to reconstruct a
lateral or medial oblique tip loss. The base of the flap is as wide as the
defects and the sides are 2 – 2 ½ times as long. One side follows the mid-
lateral line, while the other (longer) side crosses the volar side of the digit.

254
6) FLEXOR TENDON INJURY ZONES

Options
A) Zone 1
B) Zone 2a
C) Zone 2b
D) Zone 2c
E) Zone 3
F) Zone 4
G) Zone 5
H) Zone 6
For each of the following descriptions, select the zone of injury from the list
of options above. Each option may be used once, more than once or not at
all.
Questions
1. The flexor pollicis longus is rarely injured in this zone
2. This zone is involved in Leddy-Packer type II injuries
3. Laceration in this zone is also termed ‘spaghetti’ injury

255
Answers

1-E
2-A
3-G
Explanation
The flexor pollicis longus is covered by thenar muscles in zone 3 and
therefore rarely injured at this level.
Leddy and Packer classification describes flexor digitorum profundus tendon
avulsion injuries in zone 1. The types are described according to the level of
tendon retraction:
Type I: Palm
Type II: PIP joint (held by vinculum longum)
Type III: A4 pulley (held by bone avulsion fragment)
Type IV: As in type III but the stump itself avulses off fracture fragment
Zone 5 is the region of the distal forearm from the proximal edge of the
transverse carpal ligament to the musculotendinous junction of the flexors.
Lacerations in this zone involving multiple tendon injuries is termed ‘spaghetti’
wrist due to the resemblance of the pasta dish. Tang has subclassified zone
2 flexor injuries into zone 2a and 2b in relation to A2 pulley

256
7) EXTENSOR TENDON INJURIES

Options
A) Zone 1
B) Zone 2
C) Zone 3
D) Zone 4
E) Zone 5
F) Zone 6
G) Zone 7
H) Zone 8
For each of the following descriptions, select the commonest zone that is
injured from the list of options above. Each option may be used once, more
than once or not at all.
Questions
The zones on the extensor side of the hand where the following injuries
commonly occur:
1. Human ‘fight’ bite
2. Boutonnière deformity
3. Mallet injury

257
Answers
1-E
2-C
3-A
Explanation
Kleinert and Verdan described a classification for extensor tendon
lacerations according to eight zones. An easy mnemonic is “odd joints, even
bones”. During fights, a clenched fist may hit the teeth of the opponent at
the level of metacarpophalangeal joint. This corresponds to zone 5.
Boutonnière deformity results from untreated central slip injuries (PIP joint
flexion and DIP joint extension), which overlies the PIP joint (Zone 3).
Extensor tendon injuries in zone I result in a flexion deformity of the distal
interphalangeal joint, known as mallet deformity. These can an isolated
tendon injuries or an associated fracture of the tendon bearing bony
segment (Zone 7).

258
8) ANATOMICAL VARIATIONS

Options
A) 30%
B) 80%
C) 75%
D) 5%
E) 50%
F) 15%
G) 95%
H) 60%
From the list above, select the most appropriate answer for the following
questions. Each option may be used once, more than once or not at all.
Questions
1. In what percentage of people is the palmaris longus tendon absent?
2. Up to what percentage of people may have absence of an independent
slip of the extensor digitorum communis tendon to the little finger?
3. In what percentage of people is the flexor digitorum superficialis of the
little finger conjoined with the ring finger?

259
Answers
1-F
2-E
3-F
Explanation
The palmaris longus tendon is absent in approximately 15% of the
population. (Reimann AF, et al.1944). Therefore, it is important to clinically
test for the tendon or clinically confirm the presence of the tendon prior to
planning for tendon grafting procedures.
The extensor digitorum communis to the little finger has been found to be
absent in up to 50% in a cadaver-based study. If absent, the tendon is
replaced by juncturae from the ring finger (Tan V and Daluiski A in: Review
of hand surgery)
In approximately 15% of people with normal hands, the flexor digitorum
superficialis to the little finger is conjoined with the superficialis tendon to the
ring finger. These individuals are unable to flex the proximal interphalangeal
joint of the little finger independently.

260
9) EPONYMS IN UPPER LIMB

Options

A) Watson
B) Adson
C) Tinel
D) Phalen
E) Allen
F) Wartenberg
G) Guyon
H) Finkelstein
From the list above, please select the correct answer to match the
following descriptions. Each option can be used once, more than once or
not at all.
Questions
1. Subclavian vascular compression
2. Little finger abduction
3. Tenosynovitis of the first extensor compartment

261
Answers

1–B
2–F
3–H
Explanation
Adson’s manoeuvre is a provocative test for thoracic outlet syndrome. Whilst
doing this manoeuvre, the patient remains in a sitting position with the arm
externally rotated and forearm supinated. The patient is then asked to turn
the head towards the affected side, lift the chin and hold their breath in
inspiration. Obliteration of the radial pulse is a positive sign.
Wartenberg sign is seen in ulnar nerve palsy where there is ulnar deviation
of the little finger as a result of the unopposed action of the extensor
digitorum minimi tendon due to the paralysis of palmar interossei.
Finkelstein’s test is a provocative test for diagnostic of deQuervain’s
tenosynovitis (tenosynovitis of the 1st entensor compartment). In this test,
the patient is asked to grasp the thumb in the palm and the examiner ulnar
deviates the wrist. Pain along the first dorsal compartment of the wrist is a
positive sign.

262
10) TENDON TRANSERS

Options
A) Royle-Thompson transfer
B) Boyes transfer
C) Camitz Transfer
D) Huber Transfer
E) Brand Transfer
F) Zancolli Lasso procedure
G) Stiles–Bunnell procedure
H) Riordan procedure
For the questions given below, select the most appropriate answer from the
list of options above. Each option may be used once, more than once or
not at all.
Questions
1. This procedure is used to correct clawing following ulnar nerve palsy by
way of looping FDS through a slit in the A1 pulley
2. This method provides opposition by rotating the abductor digiti minimi
across palm and inserting in into the tendon of abductor pollicis brevis
3. This procedure is used to provide MCP joint flexion and IP joint extension
by attaching slips of FDS to the lateral bands of ring and little fingers.

263
Answer

1-F
2-D
3–G
Explanation
In Zancolli Lasso procedure, the FDS tendon slips (usually middle or ring
fingers) are looped through the A1 pulley and sutured to themselves to
prevent hyperextension of the MCP joints in ulnar claw hand. Each FDS
tendon can be divided into two slips and the division is usually done proximal
to the first cruciate pulley. With this procedure, there is a risk of Swan-neck
deformity following loss of FDS tendon.
Huber transfer is preferred in children for reconstruction of a hypoplastic
thumb. This procedure involves transfer of abductor digiti minimi across the
skin of palm and suturing it to the tendon of abductor pollicis brevis. It also
improves the appearance of the child’s hand by increasing the muscle bulk
over the thenar eminence.
Sir Harold Stiles and Forrester-Brown described the first tendon transfer for
intrinsic deficiency in ulnar claw hand using one slip of the superficialis
tendon and inserting it into the corresponding extensor digitorum tendon.
Bunnell modified Stiles technique by re-routing both slips of all superficialis
tendons and anchoring them to both sides of the lateral bands.

264
11) THUMB ANOMALIES - I

Options:
A) Wassel type I
B) Wassel type II
C) Wassel type III
D) Wassel type IV
E) Wassel type V
F) Wassel type VI
G) Wassel type VII
For the questions given below, select the most appropriate answer from the
list of options above. Each option may be used once, more than once or
not at all.
Questions

1. A 2-month-old child is brought to the paediatric hand clinic with an extra


digit. On examination, the child has a double thumb. X-ray shows duplication
of the phalynges and a bifid metacarpal. Which type is this?
2. A 3-month-old child is brought to the paediatric hand clinic with an
abnormal appearing thumb. X-rays reveal a duplicated distal phalanx with a
bifid proximal phalanx. Which type is this?
3. A 6-month-old child is brought to the paediatric hand clinic with a
complete duplication of the right thumb. X-rays reveal a completely
duplicated metacarpal with three phalanges in the duplicated thumb?

265
Answer

1–F
2–C
3–G
Explanation
Thumb duplication can occur at any level and it has been classified by
Wassel into seven types.
WASSEL classification of duplication of thumb
Type - Description
I - Bifid distal phalanx
II - Duplication at the interphalangeal joint
III - Bifid proximal phalanx
IV - Duplication at the metacarpophalangeal joint
V - Bifid metacarpal
VI - Duplication at the carpometacarpal joint
VII - Triphalangeal thumb
The commonest type if Wassel VI (40%) followed by type VII (20%). The
rarest type is type I (2%). For types I and II, the treatment is a sharing
procedure using components from both thumbs (Bilhaut-Cloquet) should be
performed. In the proximal duplications, decision has to be made as to
which of the two thumbs should be retained. However, reconstruction is
performed using components from both thumbs.

266
12) THUMB ANOMALIES - II

Options
A) Blauth type I
B) Blauth type II
C) Blauth type IIIa
D) Blauth type IIIb
E) Blauth type IV
F) Blauth type V
G) Blauth type VI
For the questions given below, select the most appropriate answer from the
list of options above. Each option may be used once, more than once or
not at all.
Questions
1. A 2-month-old child is brought to the paediatric hand clinic with a
complete absence of thumb and a normal forearm.
2. A 3-week-old child is referred to the paediatric hand clinic with an
anamoly of right thumb. On examination, the child has small sized thumb with
narrow first web space, hypoplasia of thenar muscles, unstable MCP and
CMC joints.
3. A 2-year-old child is brought to the paediatric hand clinic with slightly
diminished size of the thumb on right side with stable MCP and CMC joints,
and a good web span

267
Answers

1–F
2–D
3–A
Explanation
Hypoplasia of the thumb was classified by Blauth in 1967. Type III was sub-
classified by Manske in 1995 into IIIa and IIIb.
Blauth classification of hypoplastic thumb
Type - Clinical Features
I - Minor hypoplasia with normal skeleton
II - Narrow first web space; hypoplastic thenar muscles; MCP joint instability
III - Narrow first web space; hypoplastic thenar muscles; MCP joint
instability; abnormal extrinsic tendons; hypoplastic metacarpal
IIIA - Stable CMC joint
IIIB - Unstable CMC joint
IV - Pouce flottant (floating thumb); rudimentary phalanges; skin bridge with
neurovascular pedicle
V - Absent thumb
Types I, II and IIIa are amenable to reconstruction, whereas IIIb, IV and VI
are often treated by pollicisation of the index finger

268
Head and Neck
Single Best Answers
1) The facial artery is a branch of the:
A) External carotid artery
B) Brachiocephalic artery
C) Ascending pharyngeal artery
D) Internal carotid artery
E) Transverse cervical artery
Ans: See end of page
Explanation
The facial artery is the main artery of the face. It arises from the external
carotid artery in the carotid triangle. It enters the face by crossing the base
of the mandible and then pierces the deep cervical fascia. It lies around 2
cm lateral to the angle of the mouth and travels upwards along the lateral
aspect of the nose towards the medial canthus and terminates by supplying
the lacrimal sac and joining the dorsal nasal branch of the ophthalmic artery.
Its branches include the inferior labial artery (to the lower lip), superior labial
artery (to the upper lib and nasal septum), lateral nasal artery (to the ala
and dorsum of the nose) and small unnamed branches.
Ans: A

269
2) The main sensory nerve supply to the face is by the:

A) Facial nerve
B) Cervical plexus
C) Trigeminal nerve
D) Greater auricular nerve
E) Ascending pharyngeal nerve
Ans: See end of page
Explanation
The main sensory nerve of the face is the trigeminal nerve, and it carries out
this function via its three divisions: (i) Ophthalmic division (its branches are -
supratrochlear nerve, supraorbital nerve, lacrimal nerve, infratrochlear nerve
and external nasal nerve). The areas supplied by the ophthalmic division are
the scalp to vertex, forehead, upper eyelid, conjunctive, root, dorsum and tip
of nose; (ii) Maxillary division (its branches are - infraorbital nerve,
zygomaticofacial nerve and zygomaticotemporal nerve). The areas supplied
by the maxillary division are the upper lip, ala of nose, lower eyelid, upper
part of cheek and anterior aspect of the temple, and; (iii) Mandibular division
(its braches are the auriculotemporal nerve, buccal nerve and mental nerve).
The areas supplied by the mandibular division are the lower lip, chin, lower
part of cheek, lower jaw and upper 2/3rd of the auricle.
Ans: C

270
3) Supratrochear nerve

A) is the terminal branch of occulomotor nerve


B) passes over the frontalis muscle
C) supplies the bridge of nose
D) is a mixed motor and sensory nerve
E) supplies the conjunctiva
Ans: See end of page
Explanation
The supratrochlear nerve is a branch of the ophthalmic division of the
trigeminal nerve. It emerges through the supratrochlear foramen, passes
under the corrugator supercilli and frontalis muscles and supplies the
forehead, mainly the lower aspect towards the midline, the skin of the upper
eyelid and the Conjunctiva. It is a purely sensory nerve. The bridge of the
nose is supplied by the external nasal branch, which is a branch of the naso-
ciliary nerve (that comes from ophthalmic division of the trigeminal nerve).
The supratrochlear nerve can be damaged in patients with forehead
lacerations and those undergoing excisional surgery over this area.
Ans: E

271
4) In the TNM classification of Head and Neck cancer metastasis, a
single ipsilateral lymph node more than 6cm in size is classified as:

A) N1
B) N2a
C) N2b
D) N2c
E) N3
Ans: See end of page
Explanation
The TNM classification of regional lymph nodes in head and neck cancer is
as follows:
Nx: Regional lymph-nodes cannot be assessed
N0: No regional nodes
N1: Metastasis in a single ipsilateral lymph node d” 3cm in greatest
dimension
N2a: Metastasis in a single ipsilateral node e” 3cm but not e” 6cm in
greatest dimension
N2b: Metastasis in multiple ipsilateral lymph nodes none e” 6cm in greatest
dimension
N2c: Metastasis in bilateral or contralateral nodes, none e” 6cm in greatest
dimension
N3: Metastasis in a lymph node e” 6cm in greatest dimension
Ans: E

272
5) The external landmark for the junction of level II and III lymph nodes
in the neck (Shaw classification) is the:

A) Carotid pulse
B) Crico-thyroid membrane
C) Thyroid cartilage
D) Hyoid cartilage
E) Angle
Ans: See end of page
Explanation
The lymph nodes in the neck are divided into six groups:
Level I: Submental and submandibular groups.
Level II: Upper jugular group
Level III: Middle jugular group
Level IV: Lower jugular group
Level V: Posterior triangle group
Level VI: Anterior compartment group
The internal landmark for the junction between level II and III lymph nodes is
the carotid artery bifurcation, and the external landmark is the hyoid bone.
The internal landmark for the junction between lymph node levels III and IV
is superior belly of omohyoid muscle and the external landmark is
cricothyroid cartilage.
Ans: D

273
6) The commonest site for squamous cell carcinoma of the lip is:

A) Upper lip
B) Lower lip
C) Oral commissure
D) Upper gingivobuccal sulcus
E) Lower gingivobuccal sulcus
Ans: See end of page
Explanation
About 93% of squamous cell carcinoma (SCC) of lips occur in the lower lip,
5% occur in the upper lip and 2% in the commissure. Upper lip is the
common site for basal cell carcinoma.
Cancer of the lip is associated with the following risk factors:
· Heavy smoking
· Poor dental hygiene
· Chronic alcoholism
· Chronic erosive skin disease such as lichen planus
· Immunosuppression
SCCs of the lip occur in three forms; exophytic, verrucous and ulcerative. Of
these exophytic tumours are the commonest and verrucous carcinomas
rarest.
Ans: B

274
7) The most common site of leukoplakia is the:

A) Buccal mucosa
B) Tongue
C) Upper alveolus
D) Lower alveolus
E) Hard palate
Ans: See end of page
Explanation
Leukoplakia is a premalignant lesion of the oral cavity. The term leucoplakia
means white plaque. It is most common in the buccal mucosa, followed by
alveolar mucosa, tongue, lip, palate, floor of mouth and gingiva in that order.
A small proportion of these lesions become malignant over time.
There are three clinical variants of leucoplakia:
· Leukoplakia simplex
· Verrucous leukoplakia
· Erythroleukoplakia
Of these three, erthyroleukoplakia characterised by white and red areas is
the one that is most likely to change into malignancy.
About 80% of leucoplakias show no evidence of dysplasia; 10% show
moderate dysplasia and the rest show severe dysplasia. In patients with
histological evidence of dysplasia the risk of malignant change is 13%.
Ans: A

275
8) Hairy leukoplakia of the oral cavity is characteristically seen in:

A) Systemic lupus erythematosus


B) Smokers
C) Alcoholics
D) Patients with AIDS
E) Syphilis
Ans: See end of page
Explanation
Hairy leukoplakia, first described in 1984, is a disease affecting the oral
mucosa. It develops in approximately 80% of patients with AIDS. It clinically
appears as a unilateral corrugated lesion occurring on the posterolateral
border of the tongue. It can also occur on the floor of mouth, buccal mucosa
and palate.
The clinical appearance can vary from small smooth lesions to raised and
corrugated lesions. Histologically these lesions show thickening of the
epithelium with hyperkeratosis, koilocytosis and pyknotic nuclei. This disease
is associated with Epstein barr virus infection and the risk of developing it
increases with drop in CD4+ count in HIV patients. The reason for
development of this lesion is the failure of the immune system to clear
Epstein Barr virus infection. Apart from patients with AIDS, this disease can
also occur in patients with other causes of immunosuppression such as renal
transplantation and bone marrow recipients.
Apart from patients with AIDS, this disease can also occur in patients with
other causes of immunosuppression such as renal transplantation and bone
marrow recipients.
Ans: D

276
9) Leukoplakia:

A) is commonly seen in the sun-exposed skin


B) undergoes malignant transformation in 80% of cases
C) usually presents as erythematous lesions
D) can be treated with simple surgical excision
E) could lead to the development of basal cell carcinoma
Ans: See end of page
Explanation
Leukoplakia, literally meaning “white patch,” is seen primarily on the oral,
vulval, or vaginal mucosa. Leukoplakia in the mouth is mostly seen in older
men with a history of smoking. Ill-fitting dentures and poor dental state are
also often associated with this condition. The lesions are elevated, sharply
defined patchy areas of keratinisation and are generally lighter in colour
(white to grey) than the surrounding tissues. When long-standing, the lesions
may exhibit a verrucoid. In smaller lesions, the treatment is generally non-
operative, with lip cream, emollients or ointments. Smokers should refrain
from the habit. Poorly fitting dentures are refitted and operative dentistry
carried out where indicated. Simple excision of the involved mucosa is the
primary form of treatment (over the lips, this procedure is called a lip shave
or vermilionectomy). Of untreated lesions, 15% to 20% undergo malignant
transformation into a squamous cell carcinoma.
Ans: D

277
10) The commonest site of oral cancer is the:

A) Tongue
B) Palate
C) Lower alveolus
D) Buccal mucosa
E) Retromolar trigone
Ans: See end of page
Explanation
The common sites of oral cancer in decreasing frequency of occurrence are:
Tongue - 36%
Floor of mouth - 35%
Alveolus - 16%
Buccal mucosa - 10%
Upper alveolus and hard palate - 3%
The commonest site in the tongue is along the lateral aspect of the middle
third.
Ans: A

278
11) Bilaterality is a common feature of which parotid tumour?

A) Pleomorphic adenoma
B) Warthin’s tumour
C) Mucoepidermoid tumour
D) Carcinoma ex pleomorphic adenoma
E) Adenoid cystic carcinoma
Ans: See end of page
Explanation
Salivary gland tumours are uncommon and represent 4% of neoplasms in
the head and neck region. The majority are benign. Most salivary gland
tumours (70%) originate in the parotid gland with Pleomorphic adenomas
being the most common.
Warthin’s tumour (papillary cystadenoma lymphomatosum) is the
commonest bilateral neoplasm of the parotid. It is the second commonest
benign tumour of the parotid gland overall. It is commoner in men and
smokers. They present as painless soft to firm tumours at the tail of the
parotid gland.
Risk of malignant change in a Warthin’s tumour is approximately 0.3%.
Whartin’s tumour involving the superficial lobe is treated by superficial
parotidectomy and those involving the deep lobe by total conservative
parotidectomy. The lesion is multifocal in approximately 12% of cases.
Ans: B

279
12) Elective treatment of the lymph nodes in head and neck cancer is
indicated if the risk of subclinical neck disease as:

A) >20%
B) >10%
C) >40%
D) >50%
E) >60%
Ans: See end of page
Explanation
The management of clinically node-negative head and neck cancers remain
controversial. However, the evidence from prospective and retrospective
studies suggests that elective treatment of neck is justified if the chance of
subclinical disease is more than 20-25%. The treatment can be either
elective lymph node dissection or radiotherapy.
Occult lymph node secondaries are prognostic indicators of the
aggressiveness of the tumour.
Two clinical series (Persky and Lagmay, 1999 & Yuen et al. 1997) have
demonstrated increased 5-years survival for patients with ‘N0’ necks who
underwent elective neck dissections..
Ans: A

280
13) Which of the following is a useful criterion for diagnosing a

positive lymph node in CT scan for head and neck cancer?


A) Size greater than 3cm
B) Lima-Bean shape
C) Peripheral necrosis
D) Fat atrophy around the node
E) Discrete nodes in level IV
Ans: See end of page
Explanation
CT scan is a useful investigation for imaging the neck in head and neck
cancer. The criteria for diagnosing positive nodes in a CT scan are:
· extracapsular nodal spread or extension
· cervical node diameter >1.0 cm, or >1.5 cm for jugulodigastric and
submandibular nodes
· Spherical nodal shape, which is more likely to contain malignant disease
than oval- or lima bean-shaped nodes, which are usually benign though
hyperplastic
· groups of three or more contiguous and confluent lymph nodes, each
having a maximum diameter of 8–15 mm
· evidence of central necrosis, which is seen as a central area of
radiolucency surrounded by an enhancing rim of viable cells
Ans: C

281
14) ‘Synchronous tumours’ is the term used for cancers detected
within what period of diagnosing the original head and neck
malignancy?

A) 3 months
B) 6 months
C) 9 months
D) 12 months
E) 18 months

Ans: See end of page


Explanation
The overall incidence of synchronous primary cancer is approximately 10-
15%. Most of these lesions occur in the head and neck region, lung and
oesophagus. Approximately 6% of them may be detected by endoscopy.
Synchronous tumours are diagnosed within 6 months of diagnosing the
primary and those diagnosed after 6 months are called metachronous
tumours. Treatment of synchronous tumours will require modification of the
management plan.
The incidence of metachronous tumour is 15-25% and is mostly identified
within the first three years. Metachronous tumours can occur in the head
and neck region or in the lungs.
Ans: B

282
15) The commonest malignancy of the parotid gland is:

A) Mucoepidermoid tumour
B) Adenocarcinoma
C) Acinic cell tumour
D) Carcinoma ex pleomorphic adenoma
E) Adenoid cystic carcinoma
Ans: See end of page
Explanation
Approximately 25% of neoplasms in the parotid gland are malignant, of
which the commonest type is the muco-epidermoid carcinoma. It is divided
into three grades depending on the proportion of the glandular component.
Low-grade tumours have a five-year survival of 74%, whereas it is 5% for
high grade tumours.
The prognosis is better for patients with low-grade tumours showing more
glandular remnants, tumours of increasing histological differentiation and
those without lymphatic spread in the nodes.
Tumours involving the superficial lobe are treated by superficial
parotidectomy and those involving the deep lobe by total conservative
parotidectomy.
Ans: A

283
16) The commonest cause of submandibular gland enlargement is:

a) Pleomorphic adenoma
b) Enlargement of lymph nodes within the gland
c) Mucopeidermoid carcinoma
d) Adenoid cystic carcinoma
e) Calculus in the duct
Ans: See end of page
Explanation
The commonest cause of enlargement of submandibular salivary gland is
calculus in the duct (85%). This is due to the non-dependent drainage of the
gland and the mucoid secretions as compared to the parotid gland. Calculus
disease manifests as pain and swelling of the gland especially during eating.
A calculus can sometimes be felt over the floor of the mouth. A sialogram
taken by injecting a dye into the duct can demonstrate the obstruction.
Treatment includes removal of the calculus and marsupialisation of the duct
for distal calculi. For proximal calculi near the gland, removal of the entire
gland may be indicated. Tumours can also occur in the submandibular gland
and almost 50% of the neoplasms in the submandibular gland are malignant.
Ans: E

284
17) Tumours of the lower lip:

A) have a late metastases rate of about 40%


B) may present with metastases to neck nodes in up to 30% of patients at
time of presentation
C) are significantly more frequent in males
D) are associated with poor oral hygiene
E) are most commonly basal cell carcinomas

Ans: See end of page


Explanation
Squamous cell carcinomas account for 98% of lower lip tumours. Lower lip
cancers predominantly affect males with a male: female ratio of 80:1. The
tumours are commonly located between the midline and lateral commisure.
Most lip cancers are 1 to 2 cm in size at presentation and require a full-
thickness resection of the skin, muscle and underlying mucosa. Clinical
metastases to cervical (neck) lymph nodes are seen in less than 10% of
patients and approximately 5 -15% of patients will develop cervical lymph
node metastases at some time in the future; if so, an appropriate neck
dissection may be indicated. The tumour may arise as a single lesion or as
the most neoplastically advanced area in a diffusely premalignant vermilion
with leukoplakia. This pre-malignant change can be treated by a lip-shave
and the mucosa lining the lip can be advanced for closure and resurfacing of
the lip margin.
Ans: C

285
18) Which amongst the following statements is true regarding
thyroglossal duct cysts?

A) They present as painful masses during swallowing


B) They mostly occur above the hyoid bone
C) Excision is required due to its malignant potential
D) The duct arises from the foramen caecum in the anterior tongue
E) They may be imbedded in the strap muscles
Ans: See end of page
Explanation
The thyroid gland develops as a diverticulum from the floor of the pharynx,
which later becomes the foramen caecum at the base in the posterior
tongue. The thyroid gland then descends inferiorly into the anterior neck
through a hollow canal, the thyroglossal duct. This duct normally involutes;
however, if this involution fails to occur, a thyroglossal duct cyst may
develop. Thyroglossal duct cysts present as asymptomatic masses in the
anterior neck and mostly occur below the level of the hyoid bone in the
midline. Most patients present before the age of 30. The cysts may become
imbedded in the strap muscles. They rarely turn carcinomatous, but may
become infected. Recurrent infection is an indication for surgical excision.
Ans: E

286
19) Which amongst the following statements regarding Frey’s
syndrome is CORRECT?

A) It occurs in about 75% of patients who have undergone surgery to the


parotid gland
B) It is caused by growth of the divided sympathetic nerve fibers into the
skin
C) Treatment using 1% glycopyrrolate lotion is based on a sympatholytic
effect
D) A positive starch-iodine test is diagnostic
E) It can lead to sialolithiasis if left untreated
Ans: See end of page
Explanation
Gustatory sweating associated with the parotid gland was first described by
Duphenix (1853). Lucja Frey, a Polish neurologist, reported (1923) a case of
parotid gland infection complicated by gustatory sweating and suggested a
possible role of the auriculotemporal nerve. Since then, gustatory sweating
related to parotid surgery or infection is called Frey’s syndrome. It presents
as localized flushing and sweating of the skin overlying the surgical site. It is
caused by sprouting of the divided parasympathetic nerve branches (to the
parotid) into the divided sympathetic nerve fibers to the sweat glands. The
reported incidence ranges from 7-50%. The diagnosis is usually made from
history but can be confirmed by the starch-iodine test (Minor’s test - the
affected skin is painted with iodine and dusted with starch. The appearance
of a bluish discoloration during eating is diagnostic. A +ve test is due to
reaction of the starch and iodine in the presence of moisture/sweat).
Treatment may be required if the symptoms are significant. Medical
treatment consists of topical scopolamine (may have significant central
nervous system side effects if systemically absorbed), or 1%
glycopyrrolate, a parasympatholytic cream. Botulinum toxin can be injected
into the affected area; the effect however is not durable and will need
repeated injections every 4-6 months. Surgical options are less commonly
employed; these include: (i) re-elevating the skin flap and placing temporalis
fascia or a dermal flap in the intervening space (ii) Jacobsen’s neurectomy
or division of the preganglionic parasympathetic nerve in the middle ear.

287
Ans: D

288
20) Mucoceles in the oral cavity:
A) commonly occur in the floor of the mouth
B) are due to aberrant lymphoid tissue
C) are characterized by dark red lesions filled with thrombosed blood
D) are pre-cancerous and hence excision is recommended
E) have a high recurrence rate after excision
Ans: See end of page
Explanation
Mucocoeles are mucous retention cysts commonly seen in the buccal
mucosa or the lower lip. They are benign, smooth, round nodules usually
measuring 1 - 2 cm in diameter and filled with saliva or mucus. They are
commonly caused by trauma (e.g., biting) to the inner lower lip (75% of
cases), buccal mucosa, tongue or gingiva. When they appear on the floor of
the mouth they are called ranulas. They are usually very thin-walled and
transparent or bluish in colour (because of the thin layer of epithelium
covering capillaries). Bleeding into the lesion may cause a bright red colour
and occasionally resemble a haemangioma. Mucocoeles located deep in the
lip tissue may present as ill-defined discrete masses. These lesions may
persist for several days or weeks, rupture spontaneously, usually while
eating, but often recur. If they become symptomatic, they can be
marsupialized or excised in its entirety. The recurrence rate after removal of
these lesions is however high due to the numerous minor salivary ducts and
glands present in the lip and other oral mucosal areas.
Ans: E

289
21) A torus palatinus:

A) is a mucocele on the hard palate


B) is inherited in an autosomal recessive pattern
C) is usually located on the soft palate
D) is covered by normal-appearing oral mucosa
E) requires excision due to its high malignant potential
Ans: See end of page
Explanation
Torus palatinus is a slow-growing enlargement of the bone of the hard
palate at its midportion and covered by normal-appearing mucosa. It is
usually an incidental finding on routine examination of the oral cavity and is
more frequent in females (however, males have a higher incidence in the
American Indian population). Torus palatinus is thought to be inherited in an
autosomal dominant pattern, but X-linked dominance may also be a factor.
Unless they interfere with dentures or normal function, there is no need to
remove these lesions. Occasionally they can ulcerate or become extremely
large, affecting the speech; in such instances, excision may be indicated.
Ans: D

290
22) Ameloblastoma:

A) is a precursor of osteosarcoma of the mandible


B) usually presents as a painless swelling
C) is histologically characterized by encapsulated granular cells
D) should be resected with 5 cm margins
E) appears as a large radiopaque lesion on plain radiography
Ans: See end of page
Explanation
Ameloblastomas are benign, but locally invasive tumours of odontogenic
origin accounting for about 1% of all oral tumours and cysts on the jaws.
They arise from odontogenic epithelium or the enamel and may be
associated with impacted teeth or dentigerous cysts (20% of cases). They
may occur at any age, but usually present in the third or fourth decade of
life. The mandible is affected four times more frequently than the maxilla,
predominantly in the molar ramus area. Ameloblastoma usually presents as
a slow-growing, painless intraoral swelling and is histologically characterized
by follicles lined by tall columnar cells with reversed nuclear polarity in the
follicular type. A plexiform type is featured by closely apposed epithelial
cells that appear basaloid or cuboidal, arranged in anastomosing strands or
cords. Radiologically, the appearance of multi-loculated radiolucent areas
resembling ‘soap bubbles’ or ‘honeycomb’ pathognomic. Adjacent to a tooth,
it commonly erodes its root (displacement of teeth is more commonly
associated with simple dentigerous cysts). Due to its slow growing but
locally aggressive nature, ameloblastomas have also been referred to be
the counterpart of basal cell carcinomas of the skin. Wide local excision
along with 1 cm of the surrounding cancellous bone is the recommended
treatment option. However, cortical bone should be resected sparingly.
Malignant ameloblastoma consists of cells retaining their benign histologic
pattern but they metastasize to lungs and lymph nodes. In ameloblastic
carcinoma, the cells appear cytologically malignant and metastasize to lungs
and lymph nodes. Both these types have a very poor prognosis.
Ans: B

291
23) A method of nose reconstruction in which the post auricular skin
is raised on the superficial temporal vessels is the

A) Washio flap
B) Banner flap
C) Worthin’s flap
D) Rintala flap
E) Gillie’s flap
Ans: See end of page

Explanation
Washio flap is a pedicled flap used for nasal reconstruction, which is based
on the posterior branch of superficial temporal artery. The skin paddle is
raised from the posterior auricular region. It can also be raised with a
segment of conchal cartilage as a composite flap.
The advantages of this flap are that it provides thin, pliable and hairless skin,
which is a good match for the nasal skin; the donor site can be well
concealed. The disadvantage is that it is a two-stage procedure.
Ans: A

292
24) A cross lip flap where a random pattern vermillion is transferred
from the upper lip to fill the lower lip is called the

A) Abbe flap
B) Vermillion lip switch
C) Gillies fan flap
D) McGregor flap
E) Bernard procedure
Ans: See end of page

Explanation
The vermilion lip switch, first described by Kawamoto, is used to correct
vermillion deficiency especially in case hemifacial microsomia. It is not
suitable for cancer reconstruction where the defects usually involve full
thickness of the lip including muscle. A centrally based random pattern flap
is designed on the upper lip transversely and is raised as a thin layer
involving the vermillion and a thin layer of muscle. It is then turned 180
degrees and sutured to the lower lip defect. The flap is divided and inset in
10 to 14 days.
Ans: B

293
25) The ideal flap to reconstruct a moderate sized defect on the lower
lip that maintains the sensation and sphincter function is the

A) Abbe Flap
B) Estlander Flap
C) Modified Bernard Burrow procedure
D) Karapandzic flap
E) McGregor flap
Ans: See end of page

Explanation
The Karapandzic flap is the ideal flap for reconstructing both upper and
lower lip defects since it maintains oral competence and sensation. It is a
rotation-advancement flap that preserves the neurovascular bundles to the
muscles, mucosa and skin. The blood supply is from branches of the facial
artery and motor supply is from branches of the facial nerve. The
disadvantages of this flap are the extensive perioral scarring and the
tendency to cause microstomia.
Ans: D

294
26) Level VI lymph nodes in the neck lie in the

A) Carotid triangle
B) Submandibular region
C) Paratracheal area
D) Posterior triangle
E) Superior mediastinum
Ans: See end of page
Explanation
The lymph nodes in the neck are classified into various anatomical levels:
Level I: Submental and submandibular lymph nodes. Their borders are the
anterior and posterior bellies of digastrics muscles, body of mandible and
hyoid bone.
Level II: Upper jugular nodes. Their borders are inferior border of hyoid
bone, stylohyoid muscle, lateral border of sternocleidomastoid muscle and
inferior border of hyoid.
Level III: Mid jugular nodes. Their borders are the lateral border of
sternohyoid, lateral border of sternocleidomastoid, inferior border of hyoid
bone and inferior border of cricoid cartilage.
Level IV: Lower jugular nodes
Borders: Upper border of clavicle, lower border of cricoid cartilage, lateral
border of sterno thyroid and lateral border of sternocleidomastoid.
Level V: Posterior triangle nodes.
Borders: Anterior border of trapezius muscle, posterior border of
sternocleidomastoid and clavicle.
Level VI: Paratracheal and paralaryngeal nodes
Borders: Upper border of hyoid bone, suprasternal notch and common
carotid arteries.
Ans: C

295
27) The type of neck dissection in which all the node groups are
removed along with sternocleidomastoid muscle and internal jugular
vein is

A) Modified radical neck dissection


B) Radical Neck dissection
C) Selective neck dissection
D) Extended neck dissection
E) Salvage neck dissection
Ans: See end of page
Explanation
Neck dissections can be classified into two based on the levels of lymph
nodes and structures removed or preserved.
I. Comprehensive neck dissection, which is sub divided into:
1. Radical Neck dissection: All the ipsilateral lymph nodes from
levels I to V are removed along with spinal accessory nerve,
sternocleidomastoid and internal jugular vein.
2. Modified Radical Neck Dissection (functional neck dissection):
All the ipsilateral nodes from levels I to V are removed but with preservation
of all or one of the three structures – spinal accessory nerve,
sternocleidomastoid and internal jugular vein.
3. Extended neck dissection: All the ipsilateral lymph nodes from
levels I to V are removed with an additional level (level VI –
paratracheal/paralaryngeal nodes) and/or an additional structure (e.g.,
parotid gland).
II. Selective neck dissection:
This type of neck dissection involves removal of nodes from one or more
levels depending on the site of the primary tumour. This is indicated as a
staging procedure when there is no evidence of nodal metastasis (e.g.,
supra-omohyoid neck dissection removing levels 1, 2 and 3, or posterior
neck dissection removing level 5).
Ans: A

296
28) The eponymous classification system that describes the grades of
facial nerve palsy is

A) Buck Gramco
B) House-Brackmann
C) Nagata
D) Seddon
E) Regnault
Ans: See end of page
Explanation
House Brackmann classification, published in 1985, is one of the commonly
used grading systems to assess severity of facial nerve palsy. It takes into
consideration the deformity both at rest and during movement. The salient
features are tabulated below:
Grade and Description - Characteristics
I. Normal - Facial function normal in all areas.
II. Mild dysfunction - Gross: Slight weakness and very slight synkinesis
noticed. Normal symmetry at rest.
- Dynamic: Forehead: moderate to good function.Eye: complete closure with
minimal effort.Mouth: slight asymmetry.
III. Moderate dysfunction - Gross: Obvious but not disfiguring difference
between the two sides. Noticeable but not severe synkinesis and spasms.
Normal symmetry and tone at rest.
- Dynamic: Forehead: slight to moderate movement. Eye: complete closure
with effort. Mouth: slightly weak during maximum effort.
IV. Moderately severe dysfunction - Gross: Obvious weakness and
severe asymmetry. Normal symmetry and tone at rest.
- Dynamic: Forehead – no movement. Eye – Incomplete closure. Mouth –
asymmetric during maximum effort.
V. Severe Paralysis - Gross: barely perceptible motion. Asymmetry at rest.

297
- Dynamic: Forehead – no movement. Eye – incomplete closure. Mouth -
slight movement.
VI. Total Paralysis - No movement.
*Reference: House JW and Brackmann DE. Facial nerve grading system.
Otolaryngol. Head Neck Surg., 1985: 93, 146–147.
Ans: B

298
Head and Neck
Extended Matching Questions

1) LIP RECONSTRUCTION
Options
A) Abbe Flap
B) Estlander flap
C) Karapandzic flap
D) Peri-alar crescentic advancement flap
E) Wedge excision and direct closure
F) Full thickness skin graft
G) Nasolabial flap
H) Mc Gregor flap
From the list above, select the most appropriate reconstruction. Each
option can be used once, more than once or not at all.
Questions
1) A 70-year-old man with SCC of the upper lip, resection of which would
involve one fourth of the lateral aspect of the lip not involving the commissure
2) A 65-year-old man with a SCC of the right oral commissure
3) A 50-year-old woman with a 6mm partial thickness defect below the
nostril on the philtral column of the upper lip following BCC excision

299
Answers
1-E
2-B
3-D
Explanation
The aim of lip reconstruction is to provide a sensate and competent oral
commisure. There are several techniques described for lip reconstruction
and the choice of procedure depends on the location and size of the defect.
Up to one fourth of the upper lip and almost a third of the lower lip can be
directly closed in the elderly population.
Oral commissure is difficult to reconstruct and the best results are obtained
with Estlander flap, which can be taken from the upper or lower lip
depending on the location of the defect. Abbe flaps are raised using the
same principle and are useful for the defects towards the centre of the lip.
Karapandzic flaps are used for larger defects. These are bilateral rotation
flaps, which preserve the neurovascular bundles to the lips. Partial thickness
defects of the upper lip can be a difficult problem. While full thickness skin
grafts and nasolabial flaps can provide cover, in small defects, perialar
crescentic advancement flaps often give the best results.

300
2) ANATOMY OF THE NECK

Options
A) Thoracic duct
B) Spinal accessory nerve
C) Bifurcation of carotid
D) Superior belly of Omohyoid
E) Phrenic nerve
F) Brachial plexus
G) Sympathetic chain
H) Vagus
From the list above, select the most correct answer. Each option can be
used once, more than once or not at all.
Questions
1. Which structure lies in association with lower jugular lymph nodes (level
IV) lymph nodes in the neck?
2. This structure is found in close proximity to the posterior wall of the
carotid sheath
3. This is the landmark for the junction between the lymph nodes in levels II
and III

301
Answers
1-A
2-G
3-C
Explanation
A through knowledge of the anatomy of the neck is important for the
surgeon undertaking a neck dissection.
Thoracic duct, lies on the left side of the neck, is in the same level as the
lower jugular lymph nodes (level IV). It can sometimes be injuried in neck
dissections on the left side and manifests as chyle leak post-operatively.
Treatment of established chyle leak includes, fat free diet and drainage. In
high volume leaks, total parenteral nutrition may have to be initiated.
Sympathetic chain consisting of the cervical sympathetic ganglia and the
connecting fibres lies closely on the posterior wall of the carotid sheath.
The internal landmark for the junction between the lymph node levels II and
III is the carotid bifurcation. The external landmark is the hyoid bone.
Similarly, the internal land mark for the junction between lymph node levels
III and IV is superior belly of omohyoid and the external landmark is the
cricothyroid cartilage.

302
3) FLAPS IN HEAD AND NECK SURGERY
Options
A) Pectoralis major myocutaneous flap
B) Trapezius flap
C) Radial forearm free flap
D) Free fibula osteocutaneous flap
E) Scapular osteocutaneous flap
F) Deltopectoral flap
G) Lateral arm flap
H) Free groin flap
From the list above, select the most correct answer. Each option can be
used once, more than once or not at all.
Questions
1. This is one of commonest free flaps used in soft tissue reconstruction of
intra-oral defects
2. The skin paddle of this flap is not very reliable
3. This flap provides the maximum length of vascularised bone

303
Answers

1-C
2-D
3-D

Explanation

Radial forearm flap is perhaps the most commonly used free flap in head
and neck reconstruction, although there is an increasing trend to use
anterolateral thigh flaps due to less donor site morbidity. Radial forearm flap
is used as a free fasciocutaneous flap and a small segment of the radius
can be harvested along with the flap. It can be used to provide lining to the
oral cavity or for external cover. It is a reliable flap with a predictable
vascular anatomy.
Free fibula is the most popular donor site for harvesting vascularised bone
and provides the maximum length for bone reconstruction (length of fibula
available is the native length minus 10 cm, 5cm for the upper part and 5 cm
for the lower part).
It is raised based on the peroneal artery and can be taken as a free bone
flap or as an osteocutaneous free flap. Although it provides very good
quality bone, the skin paddle is not always reliable.
Other sources of vascularised bone are:
(i) iliac crest based on deep circumflex iliac artery
(ii) a segment of the scapular bone along with scapular or parascapular
flaps and,
(iii) a segment of the ribs along with serratus anterior flap.

304
4) LYMPH NODE METASTASES IN HEAD AND NECK CANCER
Options
A) Pre-auricular
B) Upper deep cervical
C) Middle deep cervical
D) Occipital
E) Submental
F) Submandibular
G) Jugulodigastric
H) Supraclavicular
For each of the case below, select the most likely group of lymph node
involved from the list above. Each option may be used once, more than once
or not at all.
Questions
1. A 56-year-old man with an 8-month history of non-healing ulcer on the tip
of his tongue
2. A 70-year-old retired chef of Chinese origin with blood-stained discharge
from his nose. He smokes 30 cigarettes a day
3. Retired builder with a 10-month history of a non-healing ulcer with everted
edges over the vertex of his scalp.

305
Answers

1-E
2-B
3-D
Explanation
The lymph node drainage of head and neck region usually occur in a
predetermined fashion.
Lesions in the lower lip and distal tongue metastasise to submental lymph
nodes. These lymph nodes lie between the anterior belly of digastrics
muscle and form part of the level I lymph nodes.
Nasopharyngeal cancers are common in the Chinese population and
metastasise to upper deep cervical nodes. These are also known as level II
lymph nodes and lie alongside the internal jugular vein extending from the
base of carotid bifurcation.
SCCs on the scalp metastasise to occipital nodes, which ultimately drain into
deep cervical lymph nodes and posterior triangle. The knowledge of
lymphatic drainage patterns help in planning selective neck dissections.

306
5) ANATOMY OF BLOOD SUPPLY IN HEAD AND NECK REGION
Options
A) Lingual artery
B) Ascending pharyngeal artery
C) Facial artery
D) Superior thyroid artery
E) Occipital artery
F) Posterior auricular artery
G) Superficial temporal artery
H) Maxillary artery
From the list above, select the most matching answer for the following
descriptions. Each option can be used once, more than once or not at all.
Questions
1. Arises from the posterior part of the external carotid artery near the lower
margin of the posterior belly of the digastric muscle
2. Arises from the external carotid artery just below the level of the greater
cornu of the hyoid bone
3. Arises from the external carotid artery between the superior thyroid and
external maxillary artery

307
Answers

1-E
2-D
3-A
Explanation
The occipital artery arises from the posterior part of the external carotid and
near the lower margin of the posterior belly of the digastric muscle. It is
covered by the posterior belly of the digastric and stylohyoid muscles and
has the hypoglossal nerve winding around it from behind. The artery then
ascends higher up crossing the internal carotid artery, the internal jugular
vein, and the vagus and accessory nerves. It ends in the posterior part of
the scalp, where it divides into numerous branches and anastomoses with
the posterior auricular and superficial temporal arteries.
The superior thyroid artery arises from the external carotid artery just below
the level of the greater cornu of the hyoid bone, under the anterior border of
the sternocleidomastideus and ends to supply the thyroid gland.
The lingual artery arises from the external carotid artery between the
superior thyroid and maxillary arteries. Its first oblique portion is superficial
and is contained within the carotid triangle; here it is covered by the
platysma and the fascia of the neck. It then passes beneath the digastric
and stylohyoideus tendons, and runs along the undersurface of the tongue to
the tip. At this point, the artery is superficial, being covered only by the
mucous membrane.

308
6) CLASSIFICATION OF FACIAL NERVE INJURY
Options
A) Grade I
B) Grade II
C) Grade III
D) Grade IV
E) Grade V
F) Grade VI
G) Grade VII
H) Grade VIII
From the list above, select the most correct answer for the following
descriptions. Each option can be used once, more than once or not at all.
Questions
1. No movement of forehead, incomplete eye closure, slight movement of
mouth
2. No movement and obvious asymmetry at rest
3. Normal symmetry and tone at rest, complete eye closure with minimal
effort

309
Answers

1–E
2–F
3–B

Explanation

There are multiple classifications to describe facial nerve damage. The most
commonly used is the House and Brackmann scale. It classifies facial nerve
injury from grades 1 to 6 which reflects on the chances of spontaneous
nerve recovery. The grading takes into account the following two factors:
a) gross characteristics and, b) motion characteristics of facial function.
Grade I: (normal): 1) normal facial appearance in all areas; 2) normal facial
function in all areas.
Grade II: (mild dysfunction): 1) slight weakness noticeable only on close
inspection; normal symmetry and tone at rest; 2) forehead: moderate to
good function; eye: complete closure with minimal effort; mouth: slight
asymmetry.
Grade III: (moderate dysfunction): 1) obvious but not disfiguring asymmetry;
normal symmetry and tone at rest; 2) forehead: slight to moderate
movement; eye: complete closure with effort; mouth: slight weak maximum
effort.
Grade IV: (moderately severe dysfunction): 1) obvious weakness with
possible disfiguring asymmetry, but normal symmetry and tone at rest; 2)
forehead: none; eye: incomplete closure; mouth: asymmetric with maximum
effort.
Grade V: (severe dysfunction): 1) only minimally perceptible motion;
asymmetry at rest; 2) forehead: none; eye: incomplete closure; mouth: slight
movement.
Grade VI: (total paralysis): 1) no movement and obvious asymmetry at rest;
2) no movement at any level during motion (Reference: House JW,
Brackmann DE. Facial nerve grading system. Otolaryngol Head Neck Surg
1985; 93: 146– 147)

310
7) ANATOMY OF NERVE SUPPLY TO FACE
Options
A) Inferior alveolar nerve
B) Frontal branch of facial nerve
C) Marginal mandibular branch of facial nerve
D) Infra orbital nerve
E) Supratrochlear nerve
F) Supraorbital nerve
G) Mandibular nerve
H) Greater auricular nerve
For the questions given below, select the most appropriate answer from the
list of options above. Each option may be used once, more than once or
not at all.
Questions
1. A 30-year-old man presents to the A&E department after he was
punched repeatedly on his face during an altercation outside a nightclub. He
complains of inability to open his mouth and numbness along the lower lip,
chin and tooth. Which nerve is most likely to be injured?
2. A 75-year-old gentleman is seen in the post-operative dressing clinic one
week following excision of a BCC from his right temple and full thickness
skin grafting. On examination, he is noted to have right brow ptosis. Which
nerve is most likely to be injured?
3. A 45-year-old motorcyclist is brought to A&E following a high speed road
traffic accident. On examination, he is noted to have extensive peri-orbital
ecchymosis and enopthalmos. He also complains of diplopia and numbness
along the lower eyelid, nose and upper lip. Which nerve is most likely to be
injured?

311
Answers
1-A
2-B
3–D
Explanation
The inferior alveolar nerve is a branch of the mandibular division of the
trigeminal nerve. The motor component innervates the mylohyoid and the
anterior belly of digastrics muscles. The sensory part innervates the alveolar
process of the mandible, gingiva, part of the lower lip and chin. It can be
injured in fractures involving the angle of the mandible when in traverses
through the mandibular foramen.
The facial nerve divides in the substance of the parotid into five main
branches. The most superior temporal branches emerge from the upper
border of the gland, cross the zygomatic arch and supply the frontalis
muscle. The most posterior temporal branch is also called the frontal
branch. It can be injured during excisional surgery in the temple area and
manifest as brow ptosis.
The infra orbital nerve is a branch of the maxillary division of the trigeminal
nerve. It enters the infra orbital canal and exits through the infraorbital
foramen of the maxilla. It supplies the lower eyelid, part of the nose and the
cheek. It may be injured in orbital floor or maxillary fractures.

312
8) CEPHALOMETRIC ANALYSIS
Options
A) Menton
B) Gnathion
C) Pogonion
D) Gonion
E) Atriculare
F) Nasion
G) Orbitale
H) Porion
I) Supradentale
For each of the questions below, select the most appropriate answer from
the list of options above. Each option can be used once, more than once or
not at all.
Questions
1. The most inferior point on the symphyseal outline
2. The most anterior point on the bony chin
3. The mid-point of the angle of the mandible

313
Answers

1-A
2-C
3-D
Explanation
Cephalometric analysis is useful in whilst planning orthognathic surgical
procedures. A cephalogram is a lateral X-ray of the skull and facial bones
taken with the help of an instrument called Cephalostat. The cephalostat
holds the head in a standard, fixed position. The X-Ray is traced on an
acetate sheet plotting the landmarks.
Some of important landmarks in a Cephalogram are:
· MENTON (ME): The most inferior point on the symphyseal outline
· GNATHION (GN): The most anterior-inferior point on the contour of the
bony chin symphysis
· POGONION (PG): The most anterior point on the contour of the bony chin.
Determined by a tangent through Nasion
· GONION (GO): The midpoint of the angle of the mandible
· ARTICULARE (AR): The point of intersection of the inferior cranial base
surface and the averaged posterior surfaces of the mandibular condyles
· NASION (N): The junction of the frontonasal suture at the most posterior
point on the curve at the bridge of the nose
· ORBITALE (OR): The lowest point on the average of the right and left
borders of the bony orbit
· PORION (PO): The midpoint of the line connecting the most superior point
of the radio-opacity generated by each of the two ear rods of the
cephalostat
· SUPRADENTALE (SD): The most anterior inferior point on the maxilla at
its labial contact with the maxillary central incisor.

314
Paediatric Plastic Surgery
Single Best Answers
1) Which of the following is true regarding nerve supply to the ear?
A) The lesser occipital nerve supplies sensation to the lower half of the ear
B) The greater auricular nerve supplies sensation to the whole ear
C) The auriculotemporal nerve supplies sensation to the inner and outer
aspects of the superior half of the ear
D) The Arnold’s nerve is an auricular branch of the vagus nerve
E) The external auditory meatus is innervated by the lesser occipital nerve

Ans: See end of page

Explanation
The main nerves that provide the cutaneous supply to the external ear are
the greater auricular, auriculotemporal, lesser occipital and the vagus
nerves. The greater auricular nerve supplies the lower half of the ear. The
auriculotemporal nerve (from the mandibular branch of the trigeminal nerve)
innervates the upper lateral aspect of the ear and most of the external
auditory meatus. Lesser occipital nerve supplies the upper medial aspect of
the ear. The auricular branch of the vagus nerve (Arnold’s nerve) supplies
the posterior wall of the external auditory meatus and the small area on the
cranial aspect of the auricle and the adjoining part of the tympanic
membrane.
Ans: D

315
2)The external ear develops at

A) 4th week of gestation


B) 6th week of gestation
C) 8th week of gestation
D) 12th week of gestation
E) 20th week of gestation
Ans: See end of page

Explanation
The external ear is formed from the first and second branchial arches. Six
hillocks (swellings) develop on the arches at around six weeks of gestation.
The first three hillocks arise from the 1st branchial arch (mandibular arch)
and develop into tragus, the root of the helix and superior helix. The
posterior hillocks arise from the 2nd branchial arch (hyoid arch) and develop
into the antehelix, anti tragus and lobule. The external auditory meatus is
formed by the extension of the first branchial cleft.
Ans: B

316
3) One of the characteristic features of synostotic plagiocephaly that
differentiates it from deformational plagiocephaly is
A) Occipital bullet
B) Ipsilateral occipital flattening
C) Ipsilateral ear pushed anteriorly
D) Ipsilateral ear pushed posteriorly
E) Frontal bossing

Ans: See end of page

Explanation
Plagiocephaly refers to flattening of the occiput which can be from
positioning in pre-mature infants (deformational or premature fusion of the
lambdoid suture). Deformational plagiocephaly improves spontaneously
whereas lambdoid synostosis will not improve without surgery. The pertinent
anatomical differences between the two are tabulated below:
Deformational Plagiocephaly - Lambdoid Craniosynostosis
Ipsilateral forehead elongated - Contralateral forehead elongated because
of the compensatory growth
Symmetrical mastoid skull base - Bulging of ipsilateral skull base and
contralateral parietal eminence
Anterior displacement of ipsilateral ear, forehead and cheek - Posterior
displacement of ipsilateral ear, forehead and cheek.
Parallelogram shaped skull - Flattened occiput
Ans: D

317
4) Which of the following is true regarding the blood supply of the ear?

A) The posterior auricular artery is a branch of the superficial temporal


artery
B) The occipital artery is the dominant supply to the posterior ear in
approximately 90% of people
C) The posterior auricular artery runs alongside the auriculotemporal nerve
D) The posterior auricular artery supplies blood to the lobule
E) The ear gets its main blood supply from the facial artery

Ans: See end of page

Explanation
The posterior auricular and superficial temporal arteries, both branches of
the external carotid artery, mainly supply the external ear and its external
meatus. There are strong interconnecting networks between these vessels
and the ear can remain perfused through either of these systems alone. The
venous drainage of the ear is via corresponding veins, which in turn drain
into the external jugular vein. The auriculotemporal nerve is a branch of the
mandibular nerve that runs alongside the superficial temporal artery and
vein. The posterior auricular artery arises from the external carotid artery,
ascending posteriorly beneath the parotid gland. It supplies blood to the
posterior scalp and to the external ear including the lobule.
Ans: D

318
5) A five-year old boy has an ear deformity characterised by an
accessory crus extending from the antihelical fold to helical rim. This
is best described as:

A) Cryptotia
B) Pixie ear
C) Stahl’s ear
D) Prominent ear
E) Lopear

Ans: See end of page


Explanation
Stahl’s ear is a rare congenital anomaly characterised by an accessory third
crus that is horizontally oriented, extending from the antihelix to the helical
rim across the scaphoid fossa. The superior crus can also be hypoplastic or
absent. The conchal fossa is normal. It is a difficult problem to correct
surgically. Neonatal splinting has been used with success. In cryptotia, the
upper pole of the ear is buried under the skin, with absence of the superior
auriculocephalic sulcus. Pixie ear is a term used to describe an ‘elf-like’
appearance of the ear when the lobule is attached directly to the cheek.
Improper resetting of the earlobe into the cheek after a facelift can also
result in this deformity. Lop ear is a congenital ear deformity characterised
by tightening or constriction of the rim.
Ans: C

319
7) Which amongst the following statements regarding the Superficial
Musculo-Aponeurotic System (SMAS) is correct?

A) Both the sensory and motor nerves lie deep to the SMAS
B) The SMAS is tightly adherent to the zygomatic arch
C) The SMAS is not contiguous with the platysma
D) The great auricular nerve lies superficial to the SMAS
E) The composite rhytidectomy technique provides less blood supply to the
skin than the superficial plane technique
Ans: See end of page
Explanation
The SMAS is a layer of facial fascia that is contiguous with the frontalis,
galea aponeurotica, temporoparietal fascia and platysma. This forms a
continuous layer of superficial fascia in the forehead, temple, face and neck.
Sensory nerves tend to lie superficial to it, whilst motor branches are deep.
It is tightly adherent to the zygomatic arch and thins out at the nasolabial
fold. The great auricular nerve supplies sensation to the skin of the ear and
is the most commonly injured nerve in a face lift operation. This nerve lies
deep to the SMAS about 6.5 cm below the external auditory canal.
The composite rhytidectomy technique (skin and SMAS lifted as a
composite block of tissue) allows preservation of a better blood supply to
the facial skin flaps although it increases the likelihood of injury to the facial
nerve.
Ans: B

320
8) Which nerve is considered most susceptible to injury when
rhytidectomy is performed in full?
A) Frontal branch of facial nerve
B) Marginal mandibular branch of the facial nerve
C) Buccal branch of the facial nerve
D) Great auricular nerve
E) Cervical branch of facial nerve
Ans: See end of page
Explanation
Although during a full rhytidectomy (face lift), the branches of the facial
nerve and great auricular nerve may be injured, the most susceptible nerve
is the great auricular nerve. This is due to its close proximity to the skin and
superficial musculo-aponeurotic system (SMAS).
The great auricular nerve is a sensory branch from the cervical plexus. With
the head turned 45º to the side, the nerve crosses the belly of
sternomastoid 6.5 cms below the caudal edge of the external auditory canal.
The nerve lies in close proximity to the external jugular vein just underneath
the SMAS. Injury to this nerve can result in loss of sensation to the lower
half of the ear and adjacent scalp.
Ans: D

321
9) A one-year-old baby is referred to you with hypospadias. On
examination he has a coronal meatus with minimal chordee. Both
testes are felt in the scrotum and there is no evidence of any inguinal
hernia. His general examination is unremarkable. The next step in
managing this baby is:
A) Ultrasound of kidneys
B) Genetic testing for intersex
C) Urethroscopy for any urethral abnormalities
D) Discharge with reassurance
E) Plan for correction of hypospadias
Ans: See end of page
Explanation
Distal hypospadias account for the majority of cases and in the absence of
other congenital anomalies do not require any further investigations. These
patients could be operated on any time after six months of age, but most
British units tend to operate near the age of three after they are out of their
nappies.
Ans: E

322
10) Choose the correct statement regarding congenital anomalies of
the ear:
A) Microtia is commonest in Africa
B) Stahl’s ear has an accessory tragus
C) 60% of prominent ears are noted at birth
D) Tanzer Type II malformations are characterised by constricted ears
E) Middle ear surgery in microtia should precede pinna reconstruction
Ans: See end of page
Explanation
Microtia is commonest in Japan. Stahl’s ear, sometimes also known as ‘Mr
Spock’s ear’, are characterised by an abnormal third crus of the antihelix.
Sixty percent of prominent ears are noticed at birth. This gives an
opportunity to use neonatal splintage to correct these deformities.
Tanzer classified congenital ear anomalies into five types:
1) Anotia
2) Complete hypoplasia (microtia)
a) With atresia of external auditory canal
b) Without atresia of external auditory canal
3) Hypoplasia of middle third of the auricle
4) Hypoplasia of the superior third of the auricle
a) Constricted(cup and lop) ear
b) Cryptotia
c) Hypoplasia of the entire superior third
5) Prominent ears
Middle ear surgery for microtia is generally delayed until the external ear is
reconstructed to prevent scarring around the site.
Ans: C

323
11) You are called to the neonatal unit to assess a pre-term baby born
32-weeks gestation. The baby has got cleft lip and palate as well as a
small jaw. The baby has had a pulse oximetry and is found to have a
SpO2 of 80% on room air and seems to have difficulty breathing. The
first step in managing this baby is:
A) Nasopharyngeal airway
B) Orotracheal intubation
C) Tracheostomy
D) Prone positioning
E) Emergency surgery to attach tongue to anterior part of mouth.
Ans: See end of page
Explanation
Babies with Pierre Robin sequence often have difficulty breathing. This is
due to the small jaw and the posteriorly displaced tongue. This improves
often with prone positioning or positioning on the side. If the respiratory
obstruction is not relieved, a nasopharyngeal airway will improve their
breathing.
Historically glossopexy, wherein the tongue is attached to the anterior part
of the mouth to prevent it falling back (Routledge procedure) has been
decribed but is hardly ever necessary. Similarly, tracheostomy is not needed
in these babies.
Ans: D

324
12) Choose the correct statement regarding microtia:

A) Brent repair is usually done between 10 and 15 years


B) Nagata repair is done at around 5 years
C) A step common to both Brent and Nagata techniques is the use of
cartilage graft to elevate the reconstructed ear
D) The second stage of Brent repair is elevation of the reconstructed
ear
E) Nagata repair has three stages
Ans: See end of page
Explanation
Brent repair is done between 4 and 6 years, whereas Nagata repair is done
at age 10 or when the chest circumference reaches 60 cm. Brent repair is
done in three or four stages and the second stage is lobule transposition.
Both techniques use a block of cartilage to maintain the projection of the ear
after elevating it.
The steps in Brent method are:
1) Insertion of cartilage frame
2) Lobule transposition
3) Elevation of frame, cartilage graft and split skin grafting.
4) Tragus reconstruction and deepening of concha.
More recently Brent has started using a tragal component in the frame itself.
Nagata technique has two stages:
1) Insertion of cartilage frame and lobule transposition, and
2) Elevation of frame, cartilage graft, temporoparietal flap and split
skin grafting
Ans: C

325
13) You have been called to assess a newborn baby. The baby has
been born with cleft palate and parents are very anxious. On
examination, the baby has a very small jaw and also has a cleft palate.
The first line of management of this baby is:

A) Arrange for outpatient appointment to see a Cleft surgeon


B) Counsel regarding feeding advice
C) Investigation for chromosomal anomalies
D) Plan for early repair of cleft palate in view of parents’ anxiety
E) Assess and secure the airway
Ans: See end of page
Explanation
The baby in this scenario has Pierre Robin sequence, which is characterised
by micrognathia, glossoptosis and cleft palate. The primary pathology in
these cases is a small jaw, probably due to a flexed posture of the neck in
utero. This pushes the tongue upwards and prevents fusion of the palatal
shelves.
The primary concern in these babies is the airway and this should be
assessed with a pulse oximeter. If they have an airway problem, the first
step is to try a change in position. Positioning on the side or prone
positioning may relieve the airway obstruction. If this is not successful, a
nasopharyngeal airway will relieve the obstruction. It is very unlikely that any
other intervention will be required for the airway problem. Once the airway is
secured, the next aspect is feeding. These babies may need nasogastric
feeding.
Ans: E

326
14) Choose the correct statement regarding anomalies of the ear:

A) They are commonly associated with cardiac and renal anomalies


B) Isolated microtia can be the mildest form of craniofacial microsomia
C) Tanzer Type III include anomalies of the lower third of ear
D) Splinting the ear in the neonatal stage has no proven value for prominent
ear correction
E) The Mustarde technique of prominent ear correction uses cartilage
scoring
Ans: See end of page
Explanation
Ear anomalies may rarely be associated with facial clefts, cardiac, limb and
renal anomalies. Isolated microtia is now considered a mild form of
craniofacial microsomia, which in its full expression has hypoplasia of the
orbit, mandible and soft tissues of the face along with facial nerve pasly.
Tanzer classification does not include deformities of the lower third of the
ear. Tanzer type III is hypoplasia of the middle third of the ear. Splinting the
ear is useful in the neonatal stage since high circulating maternal estrogens
and relaxin makes the cartilage mouldable at that stage. Mustarde technique
uses sutures from scaphoid fossa to concha for correcting prominent ears.
The cartilage scoring technique is named after Chongchet.
Ans: B

327
15) You have been asked to give an opinion on an one-month old baby
with a haemangioma over the right eyelid. The paediatricians are
concerned that it is growing and may be blocking the vision. They are
wondering if early surgical removal could be done. Your advice would
be to:

A) Plan for emergency debulking


B) Plan for emergency LASER treatment
C) Start oral steroids with close monitoring
D) Reassurre, as haemangiomas regress quickly
E) Start intravenous Interferon treatment
Ans: See end of page
Explanation
Haemangiomas are benign vascular tumours, which appear a few days after
birth. They have a history of proliferation followed by spontaneous
resolution. Most of the time, these tumours are treated conservatively.
However haemangiomas over the eyelid can block the baby’s visual fields
and can cause amblyopia (lazy eye). These need treating early and the
mainstay of treatment in this case is using steroids. If oral steroids are
unsuccessful intralesional steroids could be used.
LASERs do not penetrate more than 1mm in depth and are not useful in
treating haemangiomas. Interferons are used as a second line in treating
haemangiomas if there is no response to steroids or if steroids are
contraindicated.
Ans: C

328
16) Which of the following statement is correct about giant congenital
naevi?

A) They have a 75% risk of malignant transformation


B) MRI of the head is indicated after the age of one year
C) They are not seen in non-Caucasians
D) Early surgery will significantly reduce the risk of malignancy
E) Giant congenital naevi are defined as more than 20 cm diameter in
adulthood
Ans: See end of page
Explanation
There are several definitions used for defining giant congenital naevi. One of
the commonly used definitions is a naevus, which occupies more than 20 cm
in diameter in adulthood or 2% of total body surface area. They have a risk
of transformation into melanoma, which is variably estimated between 2 -
45% depending on the series. These patients have a risk of neurocutaneous
melanosis or melanin deposition in the brain, typically around the ventricles.
MRI of the brain is indicated before the age of 6 months to rule out
involvement of the brain. They can occur in all races. There is no convincing
evidence that early surgical intervention will reduce the risk of malignancy
and most surgical procedures are aimed at improving the cosmetic
appearance rather than reducing the risk of malignancy.
Ans: E

329
17) Which statement is correct regarding cleft lip and palate?

A) The incidence is highest amongst Asians


B) The incidence in Caucasians is 0.5%
C) Isolated cleft palate is commoner in boys
D) Right side is more commonly affected in unilateral clefts
E) Isolated cleft palate is the commonest anomaly amongst clefts
Ans: See end of page
Explanation
Cleft lip and combined cleft lip and palate have a genetic aetiology. The
incidence amongst various ethnic groups is as follows:
• Caucasian: 1 in 1,000
• Asian 1 in 500
• Afro-Caribbean: 1 in 2,000.
The above racial heterogeneity is not however observed in isolated cleft
palate, which has an overall incidence of 0.5per 1,000 live births.
Combined cleft lip and palate is the commonest anomaly (46%) followed by
isolated cleft palate at 33%, and then isolated cleft lip at 21%. The majority
of bilateral cleft lips (86%) and unilateral cleft lips (68%) are associated with
a cleft palate.
Unilateral clefts are nine times as common as bilateral clefts, and occur
twice as frequently on the left side than on the right. The ratio of left side:
right side: bilateral clefts are 6:3:1. Combined cleft lip and palate are more
common in males, whereas isolated cleft palate occurs more commonly in
females; isolated cleft palate has a more environmental aetiology.
Ans: A

330
18) Which statement is true regarding secondary deformities after
cleft lip repair?

A) Short lip or vertical deficiency is uncommon after Millard repair


B) Abbe flap is useful for correcting vertical deficiency of the upper lip
C) A whistling deformity is characterised by a notch in the white line
D) Vertical excess or long lip is common after Tennison repair
E) Le Fort II osteotomy is indicated to correct the facial deformity in cleft
patients
Ans: See end of page
Explanation
Excess length of the lip following repair occurs following Tennison (triangular
flap) and Le Mesurier (quadrangular flap) techniques. Le
Mesurier’s technique has largely been abandoned due to this problem. Short
lip or vertical deficiency is a problem after Millard (rotation advancement)
and straightline (Rose-Thompson) repairs.
Abbe flap is commonly used for correcting horizontal deficiency of lip
following bilateral lip repair. A whistling deformity is a deficiency of the
vermilion resulting in a notch. It is corrected by z-plasty or V-Y advancement
flaps. Le Fort I osteotomy is indicated to correct facial deformity in cleft
patients.
Ans: D

331
19) Choose the correct statement regarding the genetics of cleft lip
and palate:

A) The risk of cleft lip and palate with one previously affected sibling is 4%
B) The risk of cleft lip and palate with two previously affected siblings is
17%
C) Van der Woude’s syndrome is characterised by autosomal recessive
transmission
D) Isolated cleft palate is commonly associated with genetic causes
E) Pierre - Robin sequence has a strong genetic aetiology
Ans: See end of page
Explanation
Combined cleft lip and palate have a recongised genetic aetiology. With one
affected sibling the risk of cleft lip and palate in a future pregnancy is 4%,
whereas with two affected siblings the risk is 9%. With one parent and one
sibling affected the risk is 17%.
Van der Woude’s syndrome is characterised by cleft lip, multiple pits in the
lower lip and absent second premolar teeth. It is transmitted as autosomal
dominant.
Isolated cleft palate is twice as common in females and is associated with
environmental causes. Pierre-Robin sequence does not have a strong
genetic link. It is a developmental anomaly, which occurs in utero and is
characterised by retrognathia, glossoptosis and cleft palate.
Ans: A

332
20) Choose the correct statement regarding the timing of cleft lip and
palate repair:

A) Alveolar bone grafting should be done prior to the eruption of the lateral
incisor
B) Better midfacial growth is seen with earlier palate repairs
C) The conventional timing for lip repair is 3 months
D) The usual timing of alveolar bone grafting is between 5-6 years
E) Rhinoplasty for cleft patients, if indicated, is done between 9 and
11 years of age
Ans: See end of page
Explanation
There is no consensus on the ideal timing of palate repair but lip is
conventionally done at 3 months in many centres in the UK.. The timing of
alveolar bone grafting is between 8-12 years when the permanent canine is
about to erupt. The timing of palate repair is a balance between speech and
midfacial growth. Since the growth centres of the face are in the maxilla,
early disruption of periosteum can lead to midfacial hypoplasia in adult life.
However, if the palate repair is delayed it can affect the development of
speech. Rhinoplasty is indicated in post-adolescent patients after the facial
growth is complete.
Ans: C

333
21) Choose the correct statement regarding the anatomy of cleft lip
and palate:

A) Lip develops between and 4 and 8 weeks of intra-uterine life


B) Levator veli palatini tendon curves around pterygoid hamulus
C) Passavant’s ridge is formed by musculus uvulae
D) The main blood supply to palate is the ascending pharyngeal artery
E) Posterior tonsilar pillar is formed by salpingopharyngeus muscle
Ans: See end of page
Explanation

Lip develops between 4 and 8 weeks and palate between 7 and 10 weeks
of intrauterine life. It is the tensor veli palatini that curves around the
pterygoid hamulus. Passavant’s ridge is a mucosal bulge on the posterior
pharyngeal wall seen due to hypertrophy of the superior constrictor muscle
fibres. This is a compensatory mechanism to aid palatal closure in
velopharyngeal incompetence.
Greater palatine artery enters palate through the greater palatine foramen in
the posterolateral hard palate and is the main blood supply to the palate.
Posterior tonsillar pillar is formed by palatopharyngeus muscle.
Ans: A

334
22) Choose the correct statement regarding velopharyngeal

incompetence:
A) Adenoidectomy can improve velopharyngeal incompetence in
selected children
B) Velocardiofacial syndrome is due to deletions on short arm of
chromosome 22
C) Hyne’s pharyngoplasty uses salpingopharyngeus muscle
D) Orticochea technique for correcting velopharyngeal incompetence
does not include muscles
E) CT scan with 3D reconstruction is the best investigation to diagnose
velopharyngeal incompetence
Ans: See end of page
Explanation
In velopharyngeal incompetence (VPI) the palate fails to close against the
posterior pharyngeal wall and often an enlarged adenoid aids in closure.
Following adenoidectomy, VPI is likely to worsen.
Velocardiofacial syndrome or Di George’s syndrome is due to
microdeletions on long arm of chromosome 22(22q).
Hyne’s pharyngoplasty is a static technique using salpingopharyngeus
muscle whereas Orticochea technique is a dynamic procedure using
palatopharyngeus muscle or the posterior tonsillar pillar.
The useful investigations for diagnosing velopharyngeal incompetence are
videofluoroscopy and nasoendoscopy.
Ans: C

335
23) Which amongst the following statements is true regarding
Orticochea pharyngoplasty?

A) Consists of flaps taken from the anterior tonsillar pillar


B) Contains salpingopharyngeus muscle
C) The flaps are sutured to each other and to a small inferiorly based flap
taken from the posterior pharyngeal wall
D) Is a static procedure
E) Is useful in correcting hyponasal speech
Ans: See end of page
Explanation
Several surgical procedures have been described for treating
velopharyngeal incompetence. One such procedure is Orticochea
pharyngoplasty described by Orticochea in 1968. It utilizes flaps taken from
the posterior tonsillar pillars with the flaps containing the palatopharyngeus
muscle. It is a dynamic procedure as opposed to Hyne’s pharyngoplasty that
uses salpingopharyngeus muscle and the overlying mucosa; Hyne’s
pharyngoplasty is a static procedure. Clinically, Orticochea pharyngoplasty
has been found useful in correcting hypernasal speech associated with
velopharyneagl incompetence. The flaps are sutured to each other as well
as to a small inferiorly based musculomucosal flap taken from the posterior
pharyngeal wall.
Ans: C

336
24) Haemangiomas:

A) Occur in less than 2% of Caucasian babies


B) Are never present at birth
C) Affecting capillaries are called port wine stains
D) Occur in the cervico-facial region in up to two-third of the affected babies
E) Are best treated by total surgical excision
Ans: See end of page
Explanation
Haemangiomas are the commonest cutaneous congenital vascular
abnormality. They occur in 10 to 12 percent of Caucasian babies.
Haemangiomas are not always present at birth: they are seen in
approximately 55 percent of babies at birth, and in the remaining, occur in
the first few weeks of life. They are dynamic cellular lesions, which
proliferate rapidly over a few weeks, then remain static for some months
before gradually regressing over a number of years.
Two-thirds of haemangiomas occur in the cervicofacial region; they are rare
on the trunk or in the retroperitoneum. Various treatment modalities,
including corticosteroids and interferon are used in the management of this
condition with varying degrees of success. Recently, systemic use of
Propranol (a blocker) used in a gradually, reducing dosage has been found
to be effective in the management of this condition. Because of the high
vascularity of these lesions (and the fact that many regress spontaneously
over time), surgery is not the preferred choice of treatment; it may,
however, be indicated in selected cases. Port wine stain is a capillary
malformation and not a haemangioma.
Ans: D

337
25) Hypospadias:

A) Is characterised by proximal position of the urethral meatus on the dorsal


aspect of the penis
B) May occur in up to 1:3,000 live male births
C) Is more common in boys born as a result of in vitro fertilization
D) Is associated with undescended testes in approximately 50% of cases
E) May result from oestrogen receptor deficiency during development
Ans: See end of page
Explanation
Hypospadias is a congenital condition characterised by an abnormally
proximal position of the urethral meatus on the ventral aspect of the penis or
scrotum, a hooded prepuce (foreskin) and an abnormal fibrous band
between the meatus and the glans penis (chordee). Some degree of
hypospadias is present in approximately 1:300 live male births. Hypospadias
is four times more common in boys born as a result of in vitro fertilization.
Fifteen percent of cases are associated with undescended testes and 50
percent of cases are associated with inguinal hernias.
The following factors have been implicated in the aetiology of hypospadias:
(i) androgen receptor deficiency; (ii) a decrease in the level of epidermal
growth factor within the penis during development and (iii) an increase in the
level of exogenous (environmental) oestrogens during development.
Ans: C

338
26) The Tubularised Incised Plate method of hypospadias repair was
pioneered by:

A) Duckett
B) Bracka
C) Snodgrass
D) Mathieu
E) Horton
Ans: See end of page
Explanation
The Snodgrass technique is a single stage hypospadias repair, which uses
the native urethral plate by tubing it after incising the centre to mobilise the
tissues. The raw area in the centre heals by epithelialisation.
Duckett described a single stage hypospadias repair using a tubed flap
taken from the prepuce.
Bracka popularised a two-stage hypospadias repair, the first stage involves
release of chordee tissue and resurfacing the defect using a full thickness
skin graft from the prepuce. The second stage includes creating a neo-
urethra by tubing the skin-grafted area and providing a waterproofing layer
using dartos fascia underneath the skin closure.
Mathieu popularised a ‘flip-flap’ technique for distal hypospadias whereas
Horton described the artifical erection test to examine for chordee.
Ans: C

339
27) Which among the following statements regarding Pfeiffer
syndrome is INCORRECT?

A) It is an autosomal dominant condition


B) It is associated with mutations in the FGFR-1 gene
C) It is characterised by craniosynostosis
D) It may present with partial syndactyly on hands and feet
E) Mid-face hyperplasia is a consistent feature
Ans: See end of page
Explanation
Pfeiffer syndrome is a rare autosomal dominant condition (affecting about 1
in 100,000 individuals) caused by mutations in the fibroblast growth factor
receptor genes FGFR-1 or FGFR-2. It is characterised by craniosynostosis
(skull is prematurely fused and unable to grow normally), broad and
deviated thumbs and big toes, and partial syndactyly of hands and feet. The
other associated signs and symptoms include hydrocephaly, ocular
proptosis (bulging wide-set eyes due to shallow eye sockets), ankylosed
elbows, dental problems (due to crowded teeth and a high arched palate)
and delayed development.
Based on the severity of the phenotype, Pfeiffer syndrome can be divided
into three clinical subtypes: Type 1 (or the “classic” Pfeiffer syndrome) is
mild and consists of brachycephaly, mid-face hypoplasia (NOT hyperplasia),
and finger and toe abnormalities; it is associated with normal intelligence
and generally good outcome. Type 2 consists of ‘clover-leaf’ skull, extreme
proptosis, finger and toe abnormalities, elbow ankylosis or synostosis,
developmental delay and neurological complications. Type 3 is similar to
type 2 but without a ‘clover-leaf’ skull.
Ans: E

340
28) Which among the following is NOT a feature of Poland’s
syndrome?

A) Hypoplasia of the breast


B) Abnormalities of rib cage
C) Absence of clavicular head of pectoralis major muscle
D) Deficiency of axillary hair
E) Upper extremity anomalies
Ans: See end of page
Explanation
Named after Sir Alfred Poland, the classic features of Poland syndrome
include the following: (i) Hypoplasia and/or aplasia of breast or nipple (ii)
Abnormalities/hypoplasia of the rib cage (iii) Absence of the sternocostal
head of the pectoralis major muscle (iv) Deficiency of subcutaneous fat and
axillary hair, and (v) Upper extremity anomalies such as short upper arm,
forearm or fingers (brachysymphalangism).
Some patients may have the following additional features: (i) Hypoplasia or
aplasia of serratus anterior, external oblique, pectoralis minor, latissimus
dorsi, infraspinatus, and supraspinatus muscles, (ii) total absence of the
antero-lateral ribs and herniation of lungs, and (iii) Symphalangism, with
syndactyly and hypoplasia or aplasia of the middle phalanges.
Ans: C

341
29) Which amongst the following statements regarding genital
development is true?

A) In females the paramesonephric duct disappears


B) In males the Müllerian duct degenerates to form the appendix testis
C) The mesonephric duct develops into the uterus.
D) After the 12 week of gestation, the gonads differentiate into male and
th

female forms
E) In the female the genital tubercle forms the cervix
Ans: See end of page
Explanation
Development of the genitourinary tract is the same for either sex during
week 1 to 7 of gestation. By the end of 5th week, the genital tubercle
appears and the gonads start to differentiate into male or female forms. The
genital tubercle forms the penis in the male and the clitoris in the female.
The internal sexual organs form from the paramesonephric (Müllerian) duct
and mesonephric (Wolffian) duct. The Sertoli cells in the male gonads
secrete Müllerian-inhibiting substance under the influence of which the
Müllerian duct disappears and degenerates to form a vestigial structure
known as the appendix of testes. In the female, the Müllerian duct develops
into the uterus, cervix, upper vagina and Fallopian ducts. Testosterone in the
Leydig cells induces the development of the Wolffian duct system in males.
Ans: B

342
30) The treatment of 5 mm ptosis in a ten-year-old boy who has 3 mm
levator function is best carried out by:

A) Levator plication
B) Levator advancement
C) Fasanella Servat Mullerectomy
D) Frontalis sling
E) Tarsorrhaphy
Ans: See end of page

Explanation
Ptosis or blepharoptosis means drooping of the upper eyelid and can be
congenital or acquired. The treatment of ptosis is determined by the degree
of ptosis and the amount of levator function. For minor degrees of ptosis
with good levator function a full thickness excision of conjunctiva, tarsal plate
and Muller’s muscle is the management (Fasanella Servat Mullerectomy).
For moderate degrees of ptosis levator plication or advancement may be
required. When the levator function is poor, frontalis sling is the
recommended option.
Ans: D

343
31) External ear of a child reaches adult proportions by:

A) 3 years of age
B) 6 years of age
C) 9 years of age
D) 12 years of age
E) 15 years of age
Ans: See end of page

Explanation

Several anthropometric studies have been carried out to analyse the growth
of the ear in normal subjects. Although around 95% of the ear width is
achieved by 1st year of life and the height continues to grow, adult
proportions are reached by around 6 years of age. This is an important
consideration in planning ear reconstruction in patients with microtia.
Ans: B

344
32) Which amongst the following statements is true regarding
submucous cleft plate?

A) Bifid uvula is always present


B) Is often associated with incomplete unilateral cleft lip
C) A pharyngeal flap done at six months of age gives the best results
D) All patients do not develop velopharyngeal incompetence during
childhood
E) Antenatal high resolution ultrasound can diagnose the condition
Ans: See end of page

Explanation
Submucous cleft palate presents as separation of the palatal musculature
while the mucosa remains intact. It is not possible to diagnose during
antenatal screening because the palate appears grossly intact. The classic
triad of submucous cleft palate are bifid uvula, zona pellucid (translucent
area in the midline of the palate where the levator palatini has failed to fuse)
and the presence of notched posterior hard palate. All three features may
not always be present and notch in the hard palate is the most consistent
finding. It is not usually associated with cleft lip.
They are usually diagnosed late when the child develops hypernasal speech
due to velopharyngeal incompetence, although not seen in all children.
Surgical correction is indicated in the presence of velopharyngeal
incompetence and pharyngeal flaps are not indicated during infancy.
Ans: D

345
33) The anatomical landmark that separates the primary palate from
the secondary palate is the
A) Greater palatine foramen
B) Foramen of Scarpa
C) Incisive foramen
D) Lesser palatine foramen
E) Palatine process of maxilla
Ans: See end of page

Explanation
The primary palate is derived embryologically from the fusion of median
nasal prominances and the two maxillary prominences by around six weeks
of gestation. The structures that develop from the primary palate are the
nose, lip, prolabium and premaxilla. The formation of secondary palate
starts at six weeks of gestation. The secondary palate is derived from the
fusion of the palatine shelves derived from the maxillary prominences. The
incisive foramen separates the primary palate from secondary palate, and
the fusion of the secondary palate starts from the incisive formen backwards
towards the uvula.
Ans: C

346
34) The muscle that contributes to the formation of Passavant’s ridge
is the

A) Superior constrictor
B) Middle constrictor
C) Inferior constrictor
D) Palatoglossus
E) Levator veli palatini
Ans: See end of page

Explanation
Velopharyngeal closure is achieved by the sphincter mechanism of the soft
palate that results in the soft palate rising towards the posterior pharyngeal
wall, thus separating the nose from the mouth. This intrinsic function of the
soft palate aids in breathing, swallowing, phonation and blowing. The
muscles of the soft palate form the anterior aspect of the sphincter function
in the form of a palatal sling. The posterior aspect of this sphincter is formed
by the superior constrictor muscle. In children with velopharyngeal
incompetence, this muscle is hypertrophied and is termed the Passavant’s
ridge.
Ans: A

347
35) Which one of the following muscles do not form part of the soft
palate’s sphincter mechanism

A) Palatoglossus
B) Glossopharyngeus
C) Palatopharyngeus
D) Tensor veli palatini
E) Levator veli palatini
Ans: See end of page

Explanation
Five pairs of muscles form the sphincter mechanism of the soft palate. They
are levator veli palatini, tensor veli palatini, palatoglossus (originate from the
midline of the palate to the tongue), palatopharyngeus (originate from the
midline of the palate to the pharyngeal wall) and musculus uvulae. The
sphincter function is further augmented by the superior constrictor muscle.
Ans: B

348
36) A 6-year-old boy presents with bilateral ear deformities since birth.
On examination, there is an accessory crus running from the anti-helix
to the helical rim in the upper pole. What is the most likely diagnosis?
A) Lop ear
B) Cryptotia
C) Stahl’s ear
D) Constricted ear
E) Lobule type microtia

Ans: See end of page

Explanation
Stahl’s ear is a congenital deformity of the ear that presents with the
following features:
· Presence of third crus that traverses the scapha
· Flar antehelix
· Malformed scaphoid fossa
This is a difficult problem to correct. If diagnosed in the neonatal stage,
splintage can be attempted. In older children, there are several surgical
techniques described with variable success. These include suture
techniques, cartilage scoring and excision of the accessory crus.
Ans: C

349
Paediatric Plastic Surgery
Extended Matching Questions
1) FACIAL NERVE
Options
A) Zygomatic branch
B) Buccal branch
C) Mandibular branch
D) Cervical branch
E) Frontal branch
F) Temporal branch
G) Postauricular branch
H) Tympanic branch
From the list above select the branch corresponding to the description
below. Each option may be used once, more than once or not at all.
Questions
1. Lies under the SMAS, 6 cm horizontally anterior from the ear lobe sulcus
2. Lies under the SMAS, 3 cm horizontally anterior to the tragus of the ear
3. 4cm below the ear lobe sulcus deep to the cervical fascia

350
Answers

1-B
2-A
3-C
Explanation
The exact location of the facial nerve branches in relation to the SMAS is
critical to a face dissection in rhytidectomy.
The facial nerve travels through the substance of the parotid gland and exits
this gland dividing into branches traversing the superficial surface of the
masseter muscle, often covered by sub–SMAS fat.
The frontal branch traverses the zygomatic arch and penetrates the SMAS
layer to innervate the frontalis muscle along its deep surface. The anterior
branch of the superficial temporal artery often accompanies this nerve.
The zygomatic branch can usually be encountered 3 cm anterior to the
tragus of the ear deep to the SMAS.
The buccal branches lie in the buccal pocket, 5.5 - 6 cm horizontally anterior
to the ear lobe sulcus. It lies in close relation to the parotid duct.
The marginal mandibular nerve exits the parotid gland approximately 4 cm
below the ear lobe sulcus and crosses over the facial vessels to run
anteriorly over the mandibular border. Posterior to facial artery, the nerve
courses above the lower border of the mandible in 80% of cases. In the rest
it travels 1cm below the border of the mandible. Anterior to the facial artery
it is always above the lower border of the mandible [Dingman & Grabb,
1962].
The cervical branch lies deep to the platysma and supplies it.

351
2) CLEFT LIP AND PALATE REPAIR

Options
A) Millard repair
B) Manchester repair
C) Veau Wardill Kilner repair
D) Tennison’s repair
E) Rose-Thompson repair
F) Le Mesurier’s repair
G) Furlow’s repair
H) McComb technique
For each description below, select the correct answer from the list of
options above. Each option can be used once, more than once or not at all.
Questions
1. Technique is used for bilateral cleft lip repair with a narrow prolabium
2. Straight-line repair used in incomplete cleft lips
3. Technique for correcting cleft lip nasal deformity

352
Answers

1-B
2-E
3-H
Explanation

There are two main Schools of thought in bilateral cleft lip repair.
(i) Millard repair uses only part of the native prolabial tissue and is useful for
cleft lips with wide prolabium.
(ii) Manchester repair uses most of the prolabium and is used when the
prolabium is narrow.
Veau Wardill Kilner is a type of palatoplasty, which is now less commonly
used. Tennison’s repair is a type of cleft lip repair, which uses a triangular
flap. Rose-Thompson technique is a straight - tline repair used in minor
incomplete cleft lips.
Le Mesurier’s technique is a cleft lip repair, which uses a quadrangular flap.
It is not used nowadays since it lengthens the lip excessively.
Furlow’s repair is a double opposing Z-plasty, which is used for cleft palate
revisional surgery. It involves two Z-plasties, which are placed reciprocally in
the soft palate. The muscle layer is on the posteriorly based flaps and once
they are transposed, the muscle sling is anatomically realigned and also
lengthens the palate.
There are several techniques used for primary correction of the nasal
deformity in cleft lip patients. McComb technique uses sutures placed in the
alar cartilage for correction of nasal deformity in cleft patients.

353
2) SYNDROMES IN PAEDIATRIC PLASTIC SURGERY
Options
A) Stickler syndrome
B) Nager syndrome
C) Binder’s syndrome
D) Pfeiffer syndrome
E) Carpenter syndrome
F) Crouzon’s syndrome
G) Di George syndrome
H) Down’s syndrome
For each description below, select the correct answer from the list of
options above. Each option can be used once, more than once or not at all.
Questions
1. High myopia and retinal detachment along with cleft palate
2. Syndrome characterised by nasomaxillary hypoplasia
3. Craniosynostosis syndrome associated with broad thumbs and toes

354
Answers

1-A
2-C
3-D
Explanation
Stickler syndrome is a disorder of collagen synthesis characterised by high
myopia, retinal detachment and cleft palate. The mode of inheritance is
autosomal dominant.
Binder’s syndrome is a rare disorder causing nasomaxillary hypoplasia and
relative mandibular prognathism.
Nager syndrome is characterised by maldevelopment of structures arising
from first and second branchial arches (underdevelopment of the cheek and
mandible, downward-sloping of the opening of the eyes, lack or absence of
the lower eyelashes, lack of development of the internal and external ear,
possible cleft palate along with hypoplasia of thumbs.
Pfeiffer syndrome characteristically has craniosynostosis with broad toes
and thumbs).
Carpenter syndrome is characterised by craniosynotosis, hypertelorism,
exorbitism and preaxial polydactyly.
Crouzon’s syndrome is characterised by craniosynotosis, mid-face
hypoplasia, cleft palate and class III malocclusion. Hand anomalies are not a
feature.
Di George syndrome is due to deletions on the long arm of chromosome.

355
3) CRANIOFACIAL SYNDROMES
Options
A) Craniofacial microsomia
B) Treacher-Collins syndrome
C) Goldenhar’s syndrome
D) Romberg’s disease
E) Macrostomia
F) Cherubism
G) Binder’s syndrome
H) Moebius syndrome
For each description below, select the correct answer from the list of
options above. Each option can be used once, more than once or not at all.
Questions
1. Epibulbar dermoids and vertebral anomalies are seen in
2. This disorder occurs bilaterally and is inherited as autosomal dominant
3. This condition is also known as Tessier 7 cleft

356
Answers

1-C
2-B
3-E
Explanation
Goldenhar’s syndrome is characterised by craniofacial microsomia,
epibulbar dermoids and vertebral anomalies.
Treacher-Collins syndrome is inherited as autosomal dominant and has
bilateral involvement. It is considered to be a combination of Tessier 6, 7, 8
clefts. Tessier 7 cleft is due to failure of fusion of maxillary and mandibular
processes, and results in macrostomia.
Craniofacial microsomia is a non-inherited condition characterised by
unilateral hypoplasia of the face, especially orbit, ear, mandible along with
facial nerve palsy. It is believed to be due to a haemorrhage from the
stapedial artery, which is an embryonic structure.
Romberg’s disease is also known as progressive hemifacial atrophy. It is a
disease of unknown aetiology characterised by progressive atrophy of soft
tissues of one side of the face. There are several theories about the
aetiology of this disease including trigeminal neuritis, viral infection and
sympathetic dysfunction.
Cherubism is a term used to describe fibrous dysplasia of the facial
skeleton. It involves maxilla and mandible and is inherited as autosomal
dominant with variable penetrance.
Binder’s syndrome includes absence of anterior nasal spine and hypoplasia
of the nose (flat nose with absent nasofrontal angle and acute nasolabial
angle). It is also known as maxillonasal dysplasia.
Children with Moebius syndrome usually present at an early age with facial
and ocular symptoms, such as facial diplegia in the form of incomplete
eyelid closure, drooling and difficulty sucking. The facial nerve (CN VII) is
involved in all cases, the abducens nerve (CN VI) in a high percentage of
cases (75%), and the hypoglossal nerve (CN XII) in only a minority of cases.

357
4) NERVE EMBRYOLOGY OF HEAD AND NECK

Options
A) Trigeminal nerve
B) Facial nerve
C) Glossopharyngeal nerve
D) Recurrent laryngeal nerve
E) Oculomotor nerve
F) Superior laryngeal nerve
G) Lingual nerve
H) Abducens nerve
For each description below, select the correct answer from the list of
options above. Each option can be used once, more than once or not at all.
Questions
1. The nerve that develops from the second branchial arch
2. The nerve that develops from the sixth branchial arch
3. The nerve that develops from the first branchial arch

358
Answers
1-B
2-D
3-A
Explanation
Early in the 4th week, branchial arches develop from the connective and
muscle tissue elements of the neural crest. The paired brachial arch
decrease in size from cranial to caudal. By the end of the 4th week, the first
three cranial arches can be identified on the external aspect of the embryo.
The 4th arch is less distinct; the 5th branchial arch is usually absent, and;
the limits of the 6th branchial arch cannot be defined externally although their
derivatives can be traced.
The nerves of the branchial arches are as follows:
(i) First branchial arch – trigeminal nerve
(ii) Second branchial arch – facial nerve
(iii) Third branchial arch – glossopharyngeal nerve
(iv) Fourth arch – superior laryngeal nerve
(v) Fifth branchial arch – recurrent laryngeal nerve
(vi) Sixth branchial arch – recurrent laryngeal nerve.

359
5) MUSCLE EMBRYOLOGY OF HEAD AND NECK

Options
A) Levator veli palatini
B) Tensor tympani
C) Muscles of facial expression
D) Anterior belly of digastric
E) Muscles of mastication
F) Stylopharyngeus
G) Intrinsic muscles of larynx
H) Stapedius muscle
For each description below, select the correct answer from the list of
options above. Each option can be used once, more than once or not at all.
Questions
1. Derivative of the third branchial arch
2. Derivative of the fourth branchial arch
3. Derivative of the second branchial arch

360
Answers

1-F
2-A
3-C
Explanation
Each branchial arch contains four essential tissue components: (i) Cartilage
(ii) Aortic arch artery (iii) Nerve, and (iv) Muscle. Muscles may migrate from
the site of origin, but the original nerve supply to these muscles are
maintained during migration. Muscles originate from different branchial
arches and therefore have different patterns of innervation. Nerve fibres
enter the mesoderm of the branchial arches and initiate muscle development
in the mesoderm.
The muscles of the branchial arches are as follows:
I. First branchial arch – muscles of mastication, anterior belly of digastric,
mylohyoid, tensor tympani and tensor veli palatini
II. Second branchial arch – muscles of facial expression, posterior belly of
digastric, stylohyoid and stapedius
III. Third branchial arch – stylopharyngeus
IV. Fourth arch – constrictors of pharynx, cricothyroid, levator veli palatini
and palatoglossus
V. Fifth and sixth branchial arches – intrinsic muscles of larynx except
cricothyroid and striated muscles of oesophagus

361
6) CRANIOSYNOSTOSES
Options
A) Synostotic frontal plagiocephaly
B) Deformational occipital plagiocephaly
C) Synostotic occipital plagiocephaly
D) Scaphocephaly
E) Trigonocephaly
F) Frontal brachycephaly
G) Occipital brachycephaly
H) Oxycephaly
For each of the following descriptions select the correct answer from the
list above. Each option may be used once, more than once or not at all.
Questions
1. This results from metopic synostosis and most often associated with
hypotelorism
2. This results from the most common isolated single suture synostosis
3. This results from supine positioning and characterised by a parallelogram-
shaped head

362
Answers

1-E
2-D
3-B
Explanations
Premature fusion of cranial sutures results in compensatory growth along a
plane parallel to the fused suture and decrease in growth of the skull
perpendicular to the axis (Virchow’s law).
Metopic synostosis results in a palpable midline forehead ridge and is
associated with a decreased distance between the bony orbits. The
resulting deformity is called trigonocephaly.
Sagittal synostosis is the most common single suture synostosis and is
characterized by a narrow, elongated cranial vault and reduced bitemporal
dimension.
Deformational plagiocephaly is a result of constant supine positioning in
infancy. It is characterised by a parallelogram-shaped head.
Affected suture - Shape of Head
Metopic - Trigonocephaly
Sagittal - Scaphocephaly
Unilateral coronal - Frontal plagiocephaly
Bilateral coronal - Frontal brachycephaly
Unilateral lambdoid - Occipital plagiocephaly
Bilateral lambdoid - Occipital brachycephaly

363
Skin and Soft Tissues
Single Best Answers
1) Bowen’s Disease:
A) is a form of chronic inflammatory skin disorder
B) can affect the mucous membranes
C) usually presents as multiple lesions
D) is commonly associated with internal malignancies
E) can be treated with topical corticosteroid agents
Ans: See end of page
Explanation
Bowen’s disease represents an intra-epithelial squamous cell carcinoma
(carcinoma-in-situ). It can involve the skin or mucous membranes, including
the mouth, anus or genitalia. These lesions have a long clinical course,
generally years. Clinically, the lesion usually appears as a solitary,
erythematous, scaly plaque. Pruritus, superficial crusting and discharge may
be noted. There is approximately a 7% increased incidence of internal
malignancies; cancers of the bladder, bronchus, breast, and oesophagus
have been reported in patients with Bowen’s disease. The common
modalities of treatment include surgical excision or a combination of
curettage and electrodessication. Adequate excision is essential, as these
lesions may subsequently become invasive squamous cell carcinomas and
metastasize. Topical therapy, including 5-fluorouracil (in propylene glycol), is
effective, particularly when multiple lesions are present.
Ans: B

364
2) Kaposi’s sarcoma:

A) in the early stages are characterised by purplish patches of purpura


B) associated with AIDS is mostly seen in older or middle-aged
individuals
C) should not be biopsied as there is a risk of further spread
D) in non-AIDS patients is rapidly fatal within a year of diagnosis
E) is usually treated by chemotherapy
Ans: See end of page
Explanation
Kaposi’s sarcoma is a multi-centric, proliferative disorder of vasoformative
tissue that usually follows a malignant neoplastic course. In the early stages,
the lesions present as purplish pigmented patches of purpura with
haemosiderosis, mostly over the lower legs and feet. Later, indurated
plaques and firm vascular polypoid excrescences appear. Kaposi’s sarcoma
is of two major types: AIDS-related and non-AIDS related. AIDS-related
disease is mostly seen in young individuals whilst non-AIDS-related disease
generally affects the older or middle-aged men of Mediterranean racial
origin. Biopsy confirmation of the skin lesions is essential; however, caution
should be exercised when handling such specimens in view of the potential
for transmission of AIDS-related disease from blood products and biopsy
tissue. The course of the disease in non-AIDS patients is slow, and
frequently they survive for more than 10 years after the onset of the
disease. This pattern contradicts to that of AIDS-related disease, which can
be radly fatal within 2 years of diagnosis. The treatment of choice for
Kaposi’s sarcoma is radiotherapy. There is little reason to surgically excise
Kaposi’s sarcoma since the disease is multi-centric and therefore recurs
rapidly.
Ans: A

365
3) Actinic keratosis:

A) is more common in dark-skinned individuals


B) is usually diffuse and blends with the adjacent normal skin
C) commonly develop into basal cell carcinomas
D) can be treated using topical 5 fluorouracil cream
E) if treated surgically, require wide excision margins
Ans: See end of page
Explanation
Actinic keratosis, also known as solar keratosis, is the most common
premalignant lesion. This can progress to an invasive malignancy and the
resulting cancer is mostly a squamous cell carcinoma. It is usually seen in
older, light-complexioned individuals. They are rare in individuals with dark-
skin. Actinic keratosis appear primarily on sun-damaged or exposed skin
and are frequently multiple. The lesions are discrete, well circumscribed,
erythematous and maculopapular. Curettage and electrodessication are the
most accepted forms of treatment.
Cryotherapy with liquid nitrogen and treatment with 5-fluorouracil (5-FU) in a
1% - 5% concentration (Efudix) are also effective. Squamous cell
carcinomas that arise from actinic keratoses rarely metastasize, suggesting
that surgical resection should be conservative with narrow margins.
Ans: D

366
4) A 72-year-old man presents to the out-patient clinic with a long
standing history of a pigmented lesion over his right cheek. On
examination, it is a light brown and macular patch, measuring 4x5 cm
in size. A few dark areas in the centre are noted but the patient is
unsure of any change. The most appropriate initial step of
management in this patient would be:

A) Excision of the entire lesion and resurface with full thickness skin graft
B) Refer for radiotherapy
C) Excision of the entire lesion and local flap cover
D) Incision biopsy of the darker areas
E) Watchful monitoring
Ans: See end of page
Explanation
The signs and symptoms in this patient suggest a probable diagnosis of a
melanoma developing in an area of lentigo maligna (melanoma-in-situ).
Although incision biopsies are generally contraindicated in melanomas, in
situations such as this, an incision or a punch biopsy of the darker areas
should be performed to rule out a melanoma.
Lentigo maligna, where the atypical melanocytes do not breach the
basement membrane, can be managed with minimal excision margins.
However, an invasive lentigo maligna melanoma would require appropriate
excision margins as dictated by Breslow thickness.
Ans: D

367
5) Regarding malignant melanomas:

A) Lentigo maligna melanoma is the most common subtype


B) Nodular melanoma commonly occurs on the palms and soles
C) Acral lentiginous melanoma accounts for the majority of melanomas in
non-Caucasians
D) Clarke’s level is more accurate and predicts the risk of metastatic
disease more precisely than Breslow’s thickness
E) Stage IIB disease has a 5-year survival of <25%
Ans: See end of page
Explanation
Superficial spreading melanoma is the most common subtype, accounting
for about 70% of all cases. It is more common on the upper back of men
and women as well as the lower extremities of women. Nodular melanoma,
the second most common subtype, occurs in 15-30% of patients. As with
superficial spreading melanomas, legs and trunk are the most frequent sites
of occurrence. Acral lentiginous melanomas is the least common sub-type,
representing 2-8% of melanoma in Caucasians, although it accounts for
about 50-90% of melanoma in non-Caucasians. It typically occurs on the
palms or soles, or beneath the nail plate (subungual variant). Breslow
thickness avoids the confounding effect of the variable thickness of the
reticular dermis. Breslow’s method is thus more accurate than the Clarke’s
level and predicts the risk of metastatic disease more precisely. Stage IIB
disease has about 70% 5-year survival. Stage III (limited nodal metastasis)
has a 35% 5-year survival and Stage IV (advanced local/distant metastasis)
has <25% 5-year survival.
Ans: C

368
6) Which amongst the following characteristic is NOT a major criterion
for diagnosing Gorlin syndrome?

A) Two or more basal cell carcinomas in persons younger than 20


years of age
B) Odontogenic keratocysts of the jaw
C) Cleft palate
D) Three or more palmar pits
E) First-degree relative with Gorlin syndrome
Ans: See end of page
Explanation
Gorlin syndrome is an autosomal dominant condition characterised by
multiple neoplasms, including basal cell carcinomas and medulloblastoma.
They may also develop multiple abnormalities of the skin, skeleton and
nervous systems. Evans et al (1993) and Kimonis et al (1997) proposed that
Gorlin syndrome can be diagnosed when 2 major or 1 major and 2 minor
criteria are present: The major criteria include: (i) multiple (>2) basal cell
carcinomas at any age, or two or more basal cell carcinomas in persons
younger than 20 years (ii) Odontogenic keratocysts of the jaw (iii) Three or
more palmar or plantar pits (iv) Bilamellar calcification of the falx cerebri (v)
Bifid, fused, or markedly splayed ribs, and (vi) First-degree relative with
Gorlin syndrome. The minor criteria include: (i) Macrocephaly (ii) Congenital
malformations (e.g., cleft lip or palate, frontal bossing, coarse face,
hypertelorism) (iii) Skeletal abnormalities such as Sprengel deformity,
marked pectus deformity, or syndactyly of the digits (iv) Radiological
abnormalities such as bridging of the sella turcica, vertebral anomalies
(e.g., hemivertebrae, fusion or elongation of the vertebral bodies), modelling
defects of the hands and feet (v) occipitofrontal circumference > 97th
percentile, with frontal bossing (vi) Ovarian fibroma, and (vii)
Medulloblastoma.
Ans: C

369
7) The most likely site of lymphatic metastasis of a SCC involving the
upper lip is to the:

A) Jugulo-digastic nodes
B) Submandibular nodes
C) Parotid nodes
D) Submental nodes
E) Jugulo-omohyoid nodes
Ans: See end of page
Explanation
The lymphatic drainage of the face is primarily through three groups of
lymph nodes:
Areas drained by pre-auricular or parotid nodes:
· Most of the forehead
· Lateral aspect of eyelid
· Conjunctiva
· Lateral aspect of cheek
· Parotid area
Areas drained by submandibular nodes
· Median part of forehead
· External nose
· Upper lip
· Lateral part of lower lip
· Medial aspect of eyelid
· Medial aspect of cheek
· Most of the lower jaw
Areas drained by submental nodes
· Central aspect of lower lip
· Chin
Ans: B

370
8) Which amongst the following statement is true regarding UV
radiation?

A) Over 95% of the UV radiation reaching the earth’s surface is UV- C


B) The most carcinogenic radiation is UV-A
C) UV-A is filtered out by ozone layer
D) The wavelength of UV-A is 400-315 nm
E) The wavelength of UV-C is 290-315 nm
Ans: See end of page
Explanation
The electromagnetic spectrum consists of radiowaves, infrared, visible light,
ultraviolet, x-ray, gamma ray and cosmic ray waves. The ultraviolet band is
further divided according to the wavelength into UV-A (400–315 nm), UV-B
(315–290nm), and UV-C (290–200 nm).
Only UV-B and UV-A reach the earth’s crust. Over 95% of solar UV
radiation reaching the earth’s surface is in the UV-A waveband; the rest is
UV-B. The solar radiation that penetrates the atmosphere is almost devoid
of UV-C. Of the three, the UV-B rays are the most carcinogenic.
Ans: D

371
9) The five-year survival of a melanoma less than 1 mm thick is about:

A) 95%
B) 80%
C) 60%
D) 50%
E) 30%
Ans: See end of page
Explanation
The survival figures for melanomas based on their thickness are as follows:
· In situ melanoma : 95–100%
· Less than 1 mm: 95–100%
· 1–2 mm: 80–96%
· 2–4 mm: 60–75%
· > 4 mm: 50%
The above figures are based on:
Revised U.K. guideline for the management of cutaneous melanoma 2010;
163: 238 – 256
Ans: A

372
10) A 2-year-old female baby of Asian origin is brought to the
outpatient clinic with a bluish discoloration around the peri-orbital
region. What is the most likely diagnosis in this baby?

A) Blue naevus
B) Naevus of Ito
C) Naevus of Ota
D) Lentigo maligna
E) Mongolian spots
Ans: See end of page
Explanation
Naevus of Ota is a benign disorder that is considered to be a hamartoma of
dermal melanocytes presumably due to failure of migration of melanocytes
from the neural crests to the epidermal level. It commonly affects Asians
with a female preponderance. There are two peaks of occurrence – first in
infancy and the other during adolescence.
Commonly unilateral, these present as blue or grey pigmentation around the
peri-orbital region along the distribution of ophthalmic and maxillary branches
of trigeminal nerve. Malignant Transformation is rare but a few cases have
been reported. Pulsed ‘Q’ switch LASERs are an effective method of
treating this lesion.
They can be associated with other conditions such as Nevus of Ito (similar
pigmentation in the shoulder girdle and upper arm area), Phakomatosis
pigmentovascularis, Nevus flammeus, Sturge-Weber syndrome,
Neurofibromatosis and leptomeningeal melanosis.
Ans: C

373
11) Marjolin’s ulcer:

A) is a BCC arising on a background of Bowen’s disease


B) is a premalignant skin condition
C) is an aggressive ulcerating SCC
D) commonly occurs in poorly-controlled diabetic patients
E) is an ischaemic ulcer in patients with chronic arterial insufficiency
Ans: See end of page
Explanation
Although Jean Marjolin (1828) first described an indolent ulcer arising in a
burn scar, this term currently encompasses SCCs arising from any form of
long standing chronic ulcers or scars. Other chronic conditions such as
sinuses and chronic lymphoedema can give rise to Marjolin’s ulcer. Most
series indicate a recurrence of 20-25%. However, it remains rare with an
estimated incidence of 1.7% of chornic wounds. Osteomyelitis also
predisposes to the development of a Marjolin’s ulcer. It arises secondary to
chronic inflammation in a non-healing wound and may occur after a long
time, sometimes decades following initial injury. It has a 30-40% rate of
metastases Most series indicate a recurrence of 20-50%. However, it
remains rare with an estimated incidence of approximately 1.7% of chronic
wounds.
Ans: C

374
12) Choose the correct statement regarding the risk factors for the
development of skin cancer:

A) Ultraviolet B sunlight is only associated with melanomas


B) Actinic keratosis is a precursor of BCC
C) p53 gene is the most commonly mutated tumour suppressor gene
D) Organ transplant recipients have twice the risk than the normal population
E) Fitzpatrick type 6 skin have a higher incidence of skin cancer
Ans: See end of page
Explanation
Ultraviolet radiation causes mutation in the p53 gene, a tumour suppressor
gene. This gene, in the normal state, prevents a cell with DNA damage to
progress through the cell cycle and induces apoptosis. p53 is the most
commonly mutated tumour suppressor gene and is found in more than 90%
of SCCs and in most BCCs and actinic keratoses.
Actinic keratosis (AK) is a precursor of SCC. Approximately 16% of AKs will
progress to invasive SCC. Clinical manifestations of this development include
erythema, pain, ulceration or hyperkeratotic thickening. Patients who have
been on long-term immunosuppressants, e.g., organ transplant recipients
are at a greater risk (50-100 times) of developing skin cancers (due to
reduced cell-mediated immunity).
Fitzpatrick type 6 skin is seen in people of Afro-Carribean descent.
People with a fair skin complexion (Fitzpatrick types 1 and 2) have a higher
incidence of skin cancer.
Ans: C

375
13) Xeroderma pigmentosum:

A) is an autosomal dominant disorder


B) is not associated with an increased risk of melanomas
C) is characterized by the absence of melanin
D) causes extreme photosensitivity of the skin and eyes
E) is a contraindication for surgical excision
Ans: See end of page
Explanation
Xeroderma pigmentosum is a rare autosomal recessive disorder, which is
characterized by extreme photosensitivity of the skin and eyes. It also leads
to premature aging of the skin and has an increased risk (up to 1,000 times)
of developing cutaneous malignancies - BCCs, SCCs and melanomas. The
main feature in this disorder is the inability to repair certain types of DNA.
Attempts should be made to treat as many lesions as possible at an early
stage using topical chemotherapeutic agents or by surgical excision.
Unfortunately, the long-term prognosis in these patients remains poor and
many die young (in their twenties).
Ans: D

376
14) Keratoacanthoma:

A) was originally described by Gorlin


B) is a tumour of the stratum lucidum
C) is histologically similar to actinic keratosis
D) has a predilection for sun-exposed areas
E) usually resolves within four weeks
Ans: See end of page
Explanation
Keratoacanthoma (KA), originally described by Hutchinson in 1889, is usually
a rapidly growing solitary tumour with a predilection for sun-exposed areas.
It usually grows rapidly over 4 to 8 weeks and can spontaneously involute,
usually within 4 to 6 months. It is thought to be derived from hair follicles
often carrying a keratin plug and is histologically similar to SCC (hence the
term ‘self-healing epitheliomas’). Patients with multiple KAs or with
sebaceous differentiation should be evaluated for Muir -Torre syndrome,
which is associated with visceral malignant tumours. Surgery is a widely
accepted treatment modality; however, there are reports of successfully
treating these lesions with close monitoring to watch for spontaneous
resolution.
Ans: D

377
15) Neurofibromas:

A) are encapsulated by epineurium


B) are also called neurilemmomas
C) of the plexiform type may have extensive involvement
D) never become malignant
E) associated with von Recklinghausen disease are inherited in an
autosomal recessive pattern
Ans: See end of page
Explanation
Neurofibromas are solitary unencapsulated spindle cell tumours arising from
the neural sheath. Schwannomas or neurilemomas are distinct nerve sheath
tumours that are encapsulated by epineurium. Multiple neurofibromatosis
occuring in von Recklinghausen disease have an autosomal dominant
inheritance pattern with variable penetrance. They carry a risk of malignant
transformation. Plexiform neurofibromata may involve large anatomical
areas and can cause gross deformity of the underlying skeleton, in particular
of the craniofacial region. Hence, imaging in the form of CT or MRI should
be considered prior to debulking these lesions.
Ans: C

378
16) Ewing’s sarcoma:

A) usually affects the skull


B) spreads by haematogenous route
C) rarely metastasizes
D) is an eponym for chondrosarcoma
E) is resistant to chemotherapy
Ans: See end of page
Explanation
Ewing’s sarcoma usually involves the long bones or the soft tissue.
Involvement of the skull or facial bones is however rare. In the craniofacial
region, the mandible is the most common site of occurrence of these
tumours. Patients present with severe pain and secondary swelling of the
affected bone. It disseminates via the haematogenous route commonly to
the lungs and bones (15-30% of patients may have metastatic disease at
the time of presentation). Primary radiotherapy and neo-adjuvant
chemotherapy have improved survival rates.
Ans: B

379
17) Rhabdomyosarcomas:

A) are the most common soft tissue sarcoma of childhood


B) are more common in Afro-Carribeans
C) arise from migratory neural crest cells
D) most commonly affect the long bones
E) are resistant to systemic chemotherapy
Ans: See end of page
Explanation
Although rhabdomyosarcomas are the third most common solid extracranial
tumour of childhood after Wilms tumour and neuroblastoma, they are the
most common soft tissue sarcoma of childhood. There is no established
racial predilection. It is a malignant tumour arising from cells of
mesenchymal origin of a skeletal muscle lineage. They usually manifest as
an expanding mass with symptoms dependant on the location; for example,
orbital rhabdomyosarcomas may present with proptosis. It commonly
involves the soft tissues of the head and neck region (about 25-30%), the
extremities (20-25%) and the genitourinary tract (about 18-20%). Surgical
excision is the treatment of choice.
However, survival may be increased by adjuvant radiotherapy and multi-
agent chemotherapy.
Ans: A

380
18) A 16-year-old girl is concerned with a bluish discoloration around
the shoulder region that has been present since birth. She is
asymptomatic and her only concern is cosmetic blemish. What is the
most likely diagnosis?

A) Blue naevus
B) Naevus of Ito
C) Naevus of Ota
D) Lentigo maligna
E) Mongolian spots
Ans: See end of page
Explanation
Naevus of Ito is a benign disorder that is considered to be a hamartoma of
dermal melanocytes presumably due to failure of migration of melanocytes
from the neural crests to the epidermis. Classically occurring in the shoulder
girdle and upper arm area, it may often be associated with Naevus of Ota
(blue or grey pigmentation around the peri-orbital region along the
distribution of ophthalmic and maxillary branches of trigeminal nerve).
Malignant transformation is extremely rare and hence treatment is aimed at
improving the cosmesis. Pulsed ‘Q’ switch LASERs including ruby,
alexandrite and Nd- YAG LASERs are effective in treating this condition
(usually about 4 to 8 cycles).
Ans: B

381
19) The type of malignancy that is most commonly associated with
naevus sebaceous of Jadassohn is:
A) Squamous cell carcinoma
B) Basal cell carcinoma
C) Sebaceous carcinoma
D) Eccrine carcinoma
E) Porocarcinoma
Ans:See end of page
Explanation
Naevus sebaceous of Jadassohn is a type of congenital skin lesion that
occurs at birth, which is commonly present in scalp. It progresses through
three distinct stages:
1. Birth or early infancy – appears as a solitary, linear or round, raised,
yellow-orange plaque with a velvety surface.
2. During adolescence - becomes verrucous, nodular, round or oval. They
can vary in length from 1cm to 10cm. Usually solitary but can be multiple.
3. Later life – may develop various types of appendymal tumours -
trichoblastoma; syringocystadenoma papilliferum; basal cell carcinoma; and,
less commonly, nodular hidradenoma, sebaceous epithelioma, apocrine
cystadenoma, eccrine carcinoma, squamous cell carcinoma, sebaceous
carcinoma, spiradenoma, and keratoacanthoma.
The most common type of malignancy is the Basal cell carcinoma that has
been reported in 10 to 15 % of the cases.
Ans: B

382
Skin and Soft Tissues
Extended Matching Questions
1) CUTANEOUS LESIONS
Options
A. Ephelis
B. Nodular melanoma
C. Giant congenital naevus
D. Lentigo maligna melanoma
E. Dysplastic naevi
F. Solar lentigo
G. Amelanotic melanoma
H. Acral lentiginous melanoma
From the options above, choose the most appropriate answer for the
clinical scenario described below. Each option can be used one, more than
once or not at all.
Questions
1. A 30-year-old woman presents to the out-patient clinic with a 3- month
history of a raised, nodular and dark pigmented lesion over her right knee. It
is itchy and bleeds occasionally.
2. A 68-year-old farmer presents to the out-patient clinic with an irregular,
brown lesion over his right cheek for nearly 10 years but this has recently
got bigger and there is a darker patch within the lesion.
3. A 73-year-old man presents to the out-patient clinic with a fleshy lump
over the sole of his foot that is rapidly increasing in size. It does not appear
pigmented and there are palpable lymph nodes in his groin.

383
Answers

1-B
2-D
3-G
Explanation
Nodular melanomas form the second most common subtype of all
melanomas. They may occur over any part of the body, but are more
common over the legs and trunk. They are raised, dark pigmented and
bleed or ulcerate. Histologically, the cells are predominantly in the vertical
growth phase. Lymphatic involvement occurs early.
Lentigo maligna melanoma (Hutchinson’s melanotic freckle) commonly arises
over the sun damaged skin of the face. It is the least malignant variety and it
presents as an irregular brown patch. The precursor in-situ lesion, lentigo
maligna, is usually present for many years before progressing to
malignancy. Malignant degeneration is characterised by thickening and the
development of a discrete tumour nodule within the lesion.
Amelanotic melanoma is a variant of melanoma in which the cells do not
make melanin. Classically, the lesions are pink or red appearing as
erythematous papules or nodules. Patients frequently present with advanced
disease and lymph node involvement.

384
2) SURGICAL MARGINS FOR SKIN CANCERS
Options
A) 2mm
B) 4mm
C) 5mm
D) 1cm
E) 2cm
F) 3cm
G) 4cm
H) 5cm
For the questions below, select the recommended surgical margin for the
skin cancer described below. Each option may be used once, more than
once or not at all.
Questions
1. The recommended surgical excision margin for a well-defined nodular
BCC of 1cm in diameter, on the back, to obtain a 95% clearance
2. The maximum recommended surgical excision margin for an in-situ
Melanoma
3. The recommended surgical excision margin for a Merkel cell tumour

385
Answers

1-B
2-C
3-F
Explanation
The surgical margins for BCCs depend mainly on the histological type and
size. For well-defined lesions which are less than 20mm in diameter, a 3mm
margin will give 85% clearance and a 4mm margin will give 95% clearance.
Morpheic and large BCCs require a larger margin to get similar clearance.
For morpheic BCCs, 3mm margins give 82% clearance whereas to obtain
95% clearance 10-13mm margins are required.
Guidelines for the management of basal cell carcinoma. Br J Dermatol
2008; 159:35-48
The recommended surgical excision margins for melanomas are:
· In situ: 2–5 mm
· Less than 1 mm: 1 cm
· 1–2 mm: 1–2 cm
· 2–4 mm: 2–3 cm (2 cm preferred)
· Greater than 4 mm: 2–3 cm
Revised U.K. guideline for the management of cutaneous melanoma 2010;
163: 238 - 256
Merkel cell tumours are unusual neoplasms and the recommendation is to
excise them with atleast 3cm margins.

386
3) BENIGN SKIN LESIONS

Options
A) Cylindroma
B) Pilomatrixoma
C) Naevus sebaceous
D) Syringoma
E) Cellular blue naevus
F) Naevus of Ota
G) Naevus of Ito
H) Spitz naevus
From the list above select the lesion for the following descriptions. Each
option may be used once, more than once or not at all.
Questions
1. Calcifying epithelioma of Malherbe
2. A yellow-orange plaque on the scalp described by Jadassohn
3. Typically found in periocular region in patients of Asian ancestry

387
Answers

1-B
2-C
3-F
Explanation
A pilomatrixoma (calcifying epithelioma of Malherbe) is a firm, flesh coloured
nodule derived from hair follicles and most commonly involves the head and
neck and upper limbs in children. They are treated by surgical excision.
Jadassohn (1895) described a naevus that has a yellow-orange, waxy
smooth appearance before puberty, and becomes a rough, verrucous
orange plaque after puberty. It is frequently seen on the scalp and in
approximately 10-15% of the cases degenerate into BCC.
Naevus of Ota is a melanocytic lesion, which appears at birth in patients of
Asian ancestry, typically in areas innervated by first and second trigeminal
nerve branches presenting as a large, blue-gray patch in the periocular
region. Treatment is by LASER therapy. A similar lesion over the shoulder
region is called Naevus of Ito.
Cylindromas are skin appendage tumours seen in head and neck region as
solitary or multiple lesions. When tumours coalesce over the scalp they are
described as ‘turban tumour’. They are benign tumours and treatment is by
excision.
Syringomas are skin coloured or yellowish papules, which commonly occur
in the cheeks or eyelids. They are benign skin adnexal neoplasms and may
be excised or ablated for cosmetic reasons.
Blue naevi are elevated bluish lesions, which are due to the presence of
melanocytes in the dermis. There are two types: common blue naevus and
cellular blue naevus. The latter occurs in the sacral area and buttock and
tends to be larger in size.
Spitz naevi are reddish or pigmented raised lesions commonly seen in
children. They have histological similarities to melanomas but are essentially
benign lesions.

388
4) LESS FREQUENT LESIONS AND TUMOURS

Options
A) Dermatofibrosarcoma protuberans (DFSP)
B) Spitz naevus
c) Sebaceous carcinoma
D) Merkel cell carcinoma
E) Extramammary Paget’s disease
F) Epithelioid sarcoma
G) Xeroderma pigmentosum
H) Sebaceous naevus
From the list above select the lesion described below. Each option may be
used once, more than once or not at all.
Questions
1. A locally aggressive tumour of the dermis mostly found on the trunk
predominantly in young patients
2. It may resemble BCC and 75% are found on the eyelid
3. A radiosensitive small cell tumour with a high rate of local Recurrence

389
Answers

1-A
2-C
3-D
Explanation
Dermatofibrosarcoma protuberans (DFSP) is a rare locally aggressive
fibroblastic tumour of the dermis with infiltrative nature presenting mostly in
young patients on the trunk (50-60%). Local recurrence is common due to
poorly defined clinical and histologic margins.
Sebaceous carcinoma or meibomian gland carcinoma is a malignant tumour
derived from the adnexal epithelium of the sebaceous glands.
Approximately, 75% occur on the eyelid and may clinically resemble BCC,
SCC or keratoconjunctivitis. A delay from onset to diagnosis frequently
occurs.
Merkel cell carcinoma is a rare tumour consisting of painless indurated
solitary dermal nodules with a smooth surface. It occurs more frequently in
older patients in the head and neck region or the trunk. Surgical excision
with wide margins of 3 cm is the treatment of choice for primary tumours.
They are radiosensitive and radiation therapy should be considered for all
patients, in particular recurrent lesions.
Extramammary Paget’s disease is a cutaneous adenocarcinoma in which
the epidermis becomes infiltrated with tumour cells. Patients present with
non- healing eczematous lesions in the genital area or perineum. They may
clinically resemble intertrigo or fungal infection. Treatment is by surgical
excision.
Epithelioid sarcoma is a type of soft tissue sarcoma and is the most
common type of sarcoma affecting the hand. It may resemble Dupuytren’s
disease and unlike other sarcomas, can have nodal spread.
Xeroderma pigmentosum is an autosomal recessive condition characterised
by photosensitivity, premature ageing and development of skin cancers.

390
5) PIGMENTED LESIONS

Options
A) Ephelis
B) Lentigo simplex
C) Café au lait macule
D) Lentigo maligna
E) Intradermal naevus
F) Junctional naevus
G) Spitz naevus
H) Blue naevus
From the above list, select the correct answer for the following
descriptions. Each option may be used once, more than once or not at all.
Questions
1. It has increased melanin in basal keratinocytes
2. It has increased melanocytes along the basal layer
3. Dome-shaped papule or nodule histologically resembling melanoma

391
Answers

1-A
2-B
3-G
Explanation
Ephelis is a benign lesion which, under microscopy, shows increased
melanin in basal keratinocytes.
Lentigo is a general term for pigmented macular lesions with a reticulated
pattern. Simple lentigo is the common brown mole, which clinically is similar
to a junctional naevus. On microscopy, it is characterized by a cluster of
increased melanocytes.
Spitz naevi are benign, compound melanocytic lesions which occur in
children or young adults. They appear as dome-shaped papules or nodules
enlarging over a few months, and are often red or deeply pigmented.
Microscopically there is a proliferation of enlarged spindle cell or epithelioid
melanocytes. They may resemble melanomas histologically.
Café au lait macules are flat brown irregular lesions seen in
neurofibromatosis and fibrous dysplasia of bone. They can also be
idiopathic.
Junctional naevi are flat and uniform coloured well-defined lesions. They
usually appear between four to twelve years of age.
Intradermal naevi are raised nodular lesions which distort the normal
anatomy of the skin. Hairs may be present in some of the lesions.

392
Statistics
Single Best Answers
1) Which of the following statements regarding research and
statistical methods is correct?
A) With a 95% confidence interval there is a 1 in 20 chance of finding a
significant result by chance alone
B) A p-value less than 0.05 indicates the difference is too small to be
detected by the study
C) A non-parametric test is used for analyzing observations from a
population with a normal distribution
D) A type I error is a false-negative result
E) A meta-analysis is a review of multiple case reports
Ans: See end of page
Explanation
Conventionally, a confidence interval of 95% is taken as the level of
statistical significance. This means that a sample difference has a 1 in 20
chance of occurring. A p-value greater than 0.05 does not provide evidence
that there is no difference between the groups; rather it states that the
difference is too small to be detected by the study.
Parametric tests (e.g. Student’s t-test) are based on the known parameters.
If no distributional assumptions can be made, samples must be analysed by
non-parametric methods (e.g. Wilcoxon test).
A type I error occurs if the null hypothesis is rejected, i.e. a significant result
is obtained when the null hypothesis is in fact true, thus producing a false-
positive result. A type II error is a false-negative result when an insignificant
result is obtained, when the null hypothesis is in fact not true.
Meta-analyses are systematic reviews in which the measures of effect from
individual studies are combined into a single overall measure that
synthesizes the findings. They are particularly well suited to combine data
from randomized controlled trials
Ans: A

393
2) Which amongst the following statements is true?

A) A type I error occurs if the null hypothesis is accepted


B) In a study with high power there is a low chance of making a type I
error
C) If the sample size is too small there is the likelihood that a type I error
will occur
D) A type II error occurs if the null hypothesis is rejected
E) A type II error is a ‘false negative’ result

Ans: See end of page

Explanation

In clinical trials, often the differences between two treatment groups is


compared and analysed using a hypothesis test. A type I error occurs if the
null hypothesis is rejected, when it is in fact true. A type II error occurs when
the null hypothesis is accepted, when the null hypothesis is in fact false (i.e.,
an insignificant result is obtained). A type II error is a false negative result.
A study with high power means that there is a high chance of detecting a
significant difference, if there is one, and the chance of making a type II
error is thus low. If a study has a too small sample size, there is the
likelihood that a type II error will occur.
Ans:E

394
3) It is generally recommended that the power of a clinical trial
should be at least:

A) 65 - 70%
B) 75%
C) 75 – 80%
D) 80 – 90%
E) >95%
Ans: See end of page
Explanation
Power is the probability that a study would be able to detect a true
significant difference in outcome between the standard/control group and
intervention group. It is generally accepted that the power of a clinical trial
should be at least 80-90%. A study with a power set at 80% accepts a
likelihood of one in five (20%) misses such a real difference. The power for
large trials is usually set at 90%, to reduce the possibility to 10% of a ‘false-
negative’ result.
Ans: D

395
4) The most appropriate test to compare two independent groups
with non-normal numerical data is:
A) Wilcoxon test
B) Friedman’s test
C) McNemar test
D) Mann-Whitney U test
E) Chi squared test
Ans: See end of page
Explanation
The Mann-Whitney U test is a non-parametric test and is used to test the
null hypothesis that two samples come from the same population, or
whether observations in one sample tend to be larger than observations in
the other. It is based on a comparison of every observation in the first
sample with every observation in the other sample.
NUMERICAL - CATEGORICAL
Normal distribution - Non-normal distribution - Ordinal - Nominal

Comparing Paired Groups

2 groups - Paired t-test - Wilcoxon matched pairs signed rank sum test -
McNemar test
>2 groups - Repeated ANOVA - Friedman’s test - Cochran Q test
Comparing Independent Groups

2 groups - Unpaired t-test - Mann-Whitney U test - Chi squared test,


Fisher’s exact test
> 2 groups - One-way ANOVA - Kruskal-Wallis ANOVA - Chi squared test
Ans:D

396
5) Which amongst the following statement regarding p-value is true?
A) It is the estimated probability of rejecting the null hypothesis in a study
B) The smaller the p-value, the greater the evidence for the null hypothesis
C) If it is >0.05, by definition, the results are then statistically significant
D) The significance level of a test is determined by the p-value
E) If the p-value is <0.3, there is a 3% chance that the observed
difference is due to chance
Ans: See end of page
Explanation
The p-value is the estimated probability of rejecting the null hypothesis of a
study question when that hypothesis is true. The null hypothesis is usually of
‘no difference’; the smaller the p-value, the greater the evidence against the
null hypothesis. Conventionally, a p-value of <0.05 is considered sufficient to
reject the null hypothesis as there is only a 5% (small) chance of the results
occurring if the null hypothesis was true. The results are then considered
significant at the 5% level. The choice of 5% is arbitrary. The significance
level for a study must be selected before the data are collected. In
situations in which clinical implications of incorrectly rejecting the null
hypothesis are severe, stronger evidence before rejecting the null
hypothesis may be required, e.g., 0.01 or less. If the p-value is <0.03 (not
<0.3) then it means that there is a 3% chance that the difference observed
is due to a chance.
Ans:A

397
6) A pharmaceutical company has found a new product that dissolves
fat selectively in body areas. A study is performed on 150 people to
evaluate its effectiveness and safety. This is a:
A) screening trial
B) phase I clinical trial
C) phase II clinical trial
D) phase III clinical trial
E) phase IV clinical trial

Ans: See end of page


Explanation
Screening trials detect certain diseases or health conditions in a population.
Clinical drug trials would undergo four phases whilst testing a drug. In phase
I trials, an experimental drug or treatment is evaluated for the first time for
its safety, to determine a safe dosage range and to identify side effects.
The group of subjects is relatively small, 20-80 people. In phase II, the
experimental drug or treatment is given to a larger group, 100 to 300
people, to test its effectiveness and to further evaluate its safety. In phase
III, the drug or treatment is given to larger groups, 1,000 to 3,000 people, to
confirm its effectiveness, monitor side effects and compare to commonly
used treatments. In phase IV trials, post-marketing studies delineate
additional information such as benefits, optimal use and risks.
Ans:C

398
7) Poisson regression:
A) is used to estimate odds ratios
B) is not linear
C) is used to analyse the rate of some event when participants have
different follow up times
D) is a chi-squared distribution
E) is a clinical observation and cannot be calculated by a computer
Ans: See end of page
Explanation
Named after the probability theory of Siméon Poisson, a French
mathematician, the Poisson regression is a form of regression analysis. The
Poisson distribution is a discrete probability distribution of a given number of
events occurring randomly over an interval of time at a constant average
rate and independently of the time since the last event. In contrast to logistic
regression, which only takes into account whether or not the event occurs
and is used to estimate odds ratios, the Poisson regression is used to
analyse the rate of some event (disease) when subjects have different
follow-up times. The rate of the event among subjects with the same
variables (e.g., age, sex) is constant over the whole study period. The aim is
to find out which variables influence the rate at which the event occurs,
and/or to compare this rate in different exposure groups and/or predict the
rate for groups of subjects with particular characteristics.
Ans:C

399
8) Which amongst the following statements is true?
A) The median is less than the mean if the data are skewed to the left
B) The median can be distorted by skewed data
C) The mean cannot be distorted by outliers
D) The mode is the value that occurs most frequently in a data set
E) A mode must be present in every data set
Ans: See end of page
Explanation
The median is similar to the mean if the data are symmetrical, but is less
than the mean if the data are skewed to the right and greater than the mean
if the data are skewed to the left. The median is the middle value of an
ordered dataset (if they were arranged in order of magnitude, from the
smallest to the largest value / ascending to descending or vice versa),
dividing the ordered values into two halves, with an equal number of values
both above and below it. It is hence not distorted by skewed data.
The mean is calculated by adding up all the values of a dataset and dividing
this sum by the number of values in the set. The mean can be distorted by
outliers or skewed data.
The mode is the value that occurs most frequently in a data set. Some data
sets do not have a mode because each value may occur only once;
however, it is possible to have more than one mode. Mode is rarely used as
a summary measure.
Ans:D

400
9) Which amongst the following is a parametric method?
A) Pearson correlation
B) Spearman’s rank
C) Wilcoxon rank sum
D) Kruskal-Wallis
E) Mann-Whitney U
Ans: See end of page
Explanation
Parametric tests assume known parameters and known probabilities of a
distribution. In a set of observations from a population with a normal
distribution, parametric tests can be used. These tests utilize the actual
values of the data and are more likely to identify significant differences
between samples of data from different populations. If no distributional
assumptions can be made, non-parametric methods must be used for
analysis and they are often based on an analysis of the ranks of data rather
than the data themselves. The Pearson correlation coefficient is a
quantitative measure of the extent to which points in a scatter diagram
conform to a straight line. It ranges from -1 to +1.
Ans: A

401
10) Cohen’s kappa is
A) a post hoc adjustment to the p-value to take account of the number of
tests performed in multiple hypothesis testing
B) the ratio of two odds
C) the value of a single variable that occurs most frequently in a data set
D) a measure of discrimination equivalent to the area under a receiver
operating characteristic curve
E) a measure of agreement between two sets of categorical measurements
on the same individuals or data

Ans: See end of page


Explanation
Cohen’s kappa is a measure of agreement between two sets of categorical
measurements on the same individuals or data. If kappa=1, then there is
perfect agreement. There are no objective criteria for judging intermediate
values. However, agreement is often provided as:
· Poor, if kappa<0
· Slight, if between 0 and 0.20
· Fair, if between 0.21 and 0.40
· Moderate, if between 0.41 and 0.60
· Substantial, if between 0.61 and 0.80
· Almost perfect, if >0.80
Ans:E

402
11) Specificity is
A) proportion of individuals with the disease who are correctly identified by
the test
B) proportion of individuals without the disease who are correctly identified
by the test
C) proportion of individuals with a positive test result who have the disease
D) proportion of individuals with a negative test result who do not have the
disease
E) proportion of individuals in a study who have the disease

Ans: See end of page


Explanation
Test result - Disease - No disease - Total
Positive - A - b - a + b
Negative - C -d - c + d
Total - a + c - b + d - n=a+b+c+d
Sensitivity is proportion of individuals with the disease who are correctly
identified by the test: a / (a + c).
Specificity is proportion of individuals without the disease who are correctly
identified by the test: d / (b + d).
Positive predictive value is proportion of individuals with a positive test
result who have the disease: a / (a + b)
Negative predictive value is proportion of individuals with a negative test
result who do not have the disease: d / (c + d).
Prevalence is the presence of disease in a proportion of individuals studied:
(a + c) / n
Ans: B

403
Statistics
Extended Matching Questions
1) EPONYMOUS NAMES IN STATISTICS
Options
A) Likert
B) McNemar
C) Poisson
D) Pearson
E) Wilcoxon
F) Mann-Whitney U
G) Kruskal-Wallis
H) Kolmogorov-Smirnov
From the list above, select the most appropriate answer for the following
statements. Each option may be used once, more than once or not at all.
Questions
1. Psychometric scale, typically used in questionnaires and in surveys.
2. A nonparametric test to determine equality of continuous, one-dimensional
probability distributions. Very useful nonparametric method for comparing
two samples.
3. A non-parametric method of testing whether samples originate from the
same distribution. The parametric equivalence of this the one-way analysis
of variance (ANOVA).

404
Answers

1 –A
2 -H
3 -G

Explanation
The Likert scale is a psychometric scale, typically used in questionnaires
and in research that involves a survey. Most commonly a 5-point Likert item
is used, in which respondents specify their level of agreement to a
statement. (strongly disagree, disagree, neither agree nor disagree
(neutral), agree and strongly agree).
A Kolmogorov-Smirnov test is a nonparametric test for the equality of
continuous, one-dimensional probability distributions (to determine whether
data are normally distributed or not). It can be used to compare a
sample with a reference probability distribution (one-sample K–S test) or to
compare two samples (two-sample K–S test). The two-sample K-S test is
considered to be one of the most useful nonparametric methods for
comparing two samples.
The Kruskal-Wallis test is an extension of the Wilcoxon rank sum test (which
compares the distributions of two independent groups of observations). It is
a non-parametric method testing whether samples originate from the same
distribution and compares more than two samples that are independent or
not related. The parametric equivalence of the Kruskal-Wallis test is the
one-way analysis of variance (ANOVA).

405
2) BIAS
Options
A) Assessment bias
B) Lead-time bias
C) Observer bias
D) Attrition bias
E) Response bias
F) Central tendency bias
G) Measurement bias
H) Reporting bias
From the list above, select the most appropriate answer for the following
statements. Each option may be used once, more than once or not at all.
Questions
1. Often arises when the majority of responders tend to move towards the
mid-point of a Likert scale
2. Occurs when changes in survival over time are investigated and patients
entered later into the study are diagnosed at an earlier stage in their
disease
3. Arises when the study participants are lost to follow-up

406
Answers

1–F
2–B
3-D
Explanation
Assessment or observer bias occurs when one observer tends to under- or
over-report a particular variable.
Lead-time bias occurs in studies where changes in survival over time are
investigated. Development of more accurate diagnostic tools may mean that
patients entered later into the study are diagnosed at an earlier stage in
their disease, resulting in an increase in survival from time of diagnosis.
Observer bias occurs when the researcher/observer knows the goals of
their study and/or the hypotheses, and allows this knowledge to influence
their observations during the study. They may be prejudiced by prior
knowledge or experience of the situation / study participants, thus influencing
the results.
Attrition bias arises when those who are lost to follow up in a study differ in
a systematic way from those who are not.
Response bias occurs, because differences in characteristics between
those who choose/volunteer to participate in a study and those who do not
are present.
Central tendency bias may occur when the majority of responders tend to
move towards the mid-point of a Likert scale (‘no opinion’ or ‘just right’).
Measurement bias occurs when a systematic error is introduced by an
inaccurate measurement tool (e.g., poorly calibrated scales).
Reporting bias occurs when participants give answers in a particular
direction to please the researcher or under-report embarrassing conditions.

407
3) DATA DISPLAY

Options
A) Box-plot
B) Histogram
C) Pie chart
D) Bar chart
E) Kaplan-Meier curve
F) Scatter diagram
G) Stem-and-leaf plot
H) Receiver operating characteristic curve
From the list above, select the most appropriate answer for the following
statements. Each option may be used once, more than once or not at all.
Questions
1. Most commonly used to display frequency with continuous data
2. Illustrates the distribution of a variable and describes the spread
3. Describes survival probability against time

408
Answers

1–B
2–A
3-E
Explanation
The box-plot (also called box-and-whisker plot) is a diagram that illustrates
the distribution of a variable, indicating the median, upper and lower
quartiles, and often the maximum and minimum values. It thus describes the
spread.
The histogram is a diagram that depicts the (relative) frequency distribution
of a continuous variable by using connected bars and is the commonest way
to illustrate a frequency distribution.
The pie chart splits a circular pie into sectors, one for each category and
each pie area is proportional to the frequency in that category.
The bar chart has a separate horizontal or vertical bar for each category, its
length being proportional to the frequency in that category. The bars are
separated to indicate that the data are categorical or discrete.
The Kaplan-Meier curve is a survival curve in which the survival probability is
plotted against the time from baseline. It is used when exact times to reach
the endpoint are known.
Scatter diagrams are useful to show a relationship between two variables.
The stem-and-leaf plot is a mixture of a diagram and a table and looks
similar to a histogram turned on its side.
The receiver operating curve is a two-way plot of sensitivity against 1-
specificity for different cut-off values for a continuous variable.

409
4)STATISTICAL METHODS

Options
A) Assessing agreement
B) Survival analysis
C) Systematic review
D) Meta-analysis
E) Linear regression
F) Sample size calculation
G) Bayesian approach
H) Hypothesis testing
From the list above, select the most appropriate match for the following
questions. Each option may be used once, more than once or not at all.
Questions
1. A method that gives the estimated probability of rejecting the null
hypothesis of a study question when that hypothesis is true.
2. A stringent process of combining the information from all relevant studies,
usually clinical trials and/or observational studies, to support evidence-based
medicine.
3. Analyses the relationship between numerical variables. Values usually
plotted in a scatter diagram.

410
Answers

1–H
2–C
3-E
Explanation
There are five stages to carry out a hypothesis test: 1) define the null and
alternative hypotheses under the study; 2) collect relevant data from a
sample of individuals; 3) calculate the value of the test statistic specific to
the null hypothesis; 4) compare the value of the test statistic to values from
a known probability distribution; 5) interpret the p-value and results.
The p-value is the estimated probability of rejecting the null hypothesis of a
study question when that hypothesis is true.
The Cochrane Database of Systematic Reviews is a regularly updated
source to support evidence-based medicine (EBM). A systematic review is
an integral part of EBM and is a formalized and stringent process of
combining the information from all relevant studies, usually clinical trials but
may include observational studies. A meta-analysis, in contrast to a
systematic review, is an overview that focuses on numerical results. The
main aim is to combine the results from several independent studies to
produce an estimate of the overall or average effect of interest.
Linear regression analyses the relationship between two variables – e.g., X
and Y. One variable is considered to be an explanatory variable, and the
other a dependent variable. To investigate the relationship between
numerical variables (e.g, x and y), the values of x and y are measured on
each of the number of individuals in a sample. The points are plotted on a
scatter diagram and if there is a straight line, a linear relationship exists in
the data. If y is dependent on x, and any change attributed to a change in x,
then a linear regression line can be determined that best describes the
straight line relationship between the variables.

411
5) STUDY DESIGNS

Options
A) Cross-sectional
B) Case-control
C) Cohort
D) Stratified randomization
E) Blocked randomization
F) Cluster randomization
G) Cross-over
H) Multicentre
From the list above, select the most appropriate match for the following
statements. Each option may be used once, more than once or not at all.
Questions
1. A multi-variable regression analysis is often required
2. A survey of how many obese patients have low self-esteem and poor
body image
3. A single point in time is studied

412
Answers

1–H
2–A
3-A
Explanation
A cross-sectional study is carried out at a single point in time and is
particularly suitable to estimate the point prevalence of a condition in the
population or to survey individuals’ beliefs or attitudes towards a particular
issue in a large sample of the population.
A multicentre study involves two or more centres performing the same
study and all centres will use the same study protocol. The analysis is
usually performed in a single coordinating centre and need to take account
of any centre effects. This can be done by adjustment for the centre in a
multivariable regression analysis.
A case-control study compares the features of a group of patients with a
particular disease outcome to a group without the disease outcome.
A cohort study takes a group of individuals and follows them forward in time
to investigate whether a particular aetiological factor will affect the incidence
of a disease outcome in the future.
Randomized controlled trials are regarded as optimal studies.
Randomization can occur in various ways: Stratified, which controls for the
effects of important factors ensuring each factor is equally distributed
across treatment groups. Blocked/restricted, which ensures roughly equal-
sized treatment groups at end of patient recruitment. Cluster, whereby a
group of individuals, rather than each individual, is randomly allocated to a
treatment.

413
6) EVIDENCE-BASED MEDICINE - LEVELS OF EVIDENCE
Options
A) Level Ia
B) Level Ib
C) Level IIa
D) Level IIb
E) Level IIIa
F) Level IIIb
G) Level IV
H) Level V
For each of the questions below, select the most appropriate answer
from the list of options above. Each option can be used once, more
than once or not at all.
Questions
1. Case series
2. Systematic review of cohort studies
3. Individual randomized controlled trials

414
Answers

1-G
2-C
3-B
Explanation
Level Ia: Systematic review of randomized controlled trials
Level Ib: Individual randomized controlled trials
Level IIa: Systematic review of cohort studies
Level IIb: Individual cohort study
Level IIIa:Systematic reviews of case-controlled studies
Level IIIb:Individual case-control study
Level IV: Case series or case reports
Level V: Expert opinion without explicit critical appraisal, or based on
physiology, bench research or “first principles”
Oxford Centre for Evidence-based Medicine Levels of Evidence

415
7) STATISTICAL TEST

Options
A) Spearman
B) Student’s T-test
C) Mann-Whitney
D) Kaplan-Meier (curve)
E) Kolmogorov-Smirnov
F) Wilcoxon
G) Fisher’s
H) Kruskal-Wallis
From the list above, select the most matching answer for the following
descriptions. Each option can be used once, more than once or not at
all.
Questions
1. A non-parametric test that compares more than two groups
2. A non-parametric measure and correlation coefficient
3. Describes survival characteristics

416
Answers

1–H
2–A
3–D
Explanation
The Kruskal-Wallis test is a non-parametric test that is used when there are
more than two groups to compare. The formula is based on the ranks of the
scores, rather than the scores themselves. The test is used to look for a
significant difference between the mean ranks of some or all of the
conditions.
A correlation coefficient is a number between ‘-1’ and ‘1’, which
measures the degree to which two variables are linearly related. A
correlation coefficient of ‘0’ means that there is no linear relationship
between the variables. The Spearman rank correlation coefficient is used in
non-parametric studies and is usually calculated on occasions when it is not
convenient or possible to give actual values to variables, but only to assign a
rank order to instances of each variable. It is a better indicator to suggest
that a relationship exists between two variables when the relationship is non-
linear.
In clinical trials, the time until participants in a particular study present
specific events or endpoint is often a crucial point of interest. This event is
usually a clinical outcome and the time can be described using the Kaplan-
Meier curve.

417
Trauma
Single Best Answers

1) Which statement is true regarding Le Fort fractures?


A) Le Fort I is the commonest type
B) The fracture line in Le Fort I passes obliquely across the maxillary sinus,
root of nose and pterygoid plates
C) The pterygoid plate are not involved in Le Fort fracture I
D) The bony fragment of Le Fort II fracture contains the lacrimal crests, the
alveolus and the palate
E) Le Fort III fracture is also known as maxillo-mandibular dysfunction
Ans: See end of page
Explanation
Fractures of the maxilla were classified by René Le Fort into three types.
Relative frequency of the three fracture types are in the order of: Le Fort II
> Le Fort I > Le Fort III.
In Le Fort I fracture, the fracture line passes transversely across the base
of the piriform aperture, the base of the maxillary sinus and the pterygoid
plates. In Le Fort II fracture, the bony fragment contains the lacrimal crests,
the bulk of the maxilla, the piriform margin, the alveolus and the palate. Le
Fort III fracture is associated with detachment of the entire mid-facial
skeleton from the cranial base. Movement of the alveolus and nasofrontal
region suggests an associated Le Fort II fracture; movement of the alveolus
alone suggests an associated Le Fort I fracture. It is hence known as
craniofacial dysfunction.
Ans: D

418
2) A 25-year-old man has sustained an open fracture of his right leg in
a road traffic accident. He has a transverse fracture of the lower third
of tibia and fibula and a large soft tissue defect measuring 5 X 9cm
over the medial aspect. The rest of his leg is bruised and covered with
abrasions. His ankle pulses are not palpable and the foot feels cold.
His blood pressure is 70/50mmHg. Clinically he has no other injuries.
The most important step in the immediate management of this patient
is:

A) Urgent exploration for repair of vessels


B) Angiography to locate vascular injury
C) Urgent CT scan abdomen to rule out intra-abdominal injury
D) Amputation of foot since unlikely to be salvageable
E) IV fluids or O-ve to normalise haemodynamic status
Ans: See end of page
Explanation
Vascular injury can accompany open fractures of the lower limb and may
require urgent exploration. However, when a patient is hypotensive, it is not
possible to diagnose a vascular injury of the leg. The first line of
management of any trauma patient should be along ATLS guidelines. In
these patient intravenous fluids blood are the immediate first step in the
management. Once the patient is haemodynamically stable, the vascular
status of the leg should be reassessed. The underlying cause of
hypovolemia needs to be urgently investigated.
Ans: E

419
3) The commonest cause of traumatic brachial plexus palsy is:

A) Motor vehicle accidents


B) Military trauma
C) Fall from height
D) Assault
E) Iatrogenic during lymph node biopsy in the neck
Ans: See end of page
Explanation
The commonest cause of traumatic brachial plexus injury is motor vehicle
accidents/road traffic accidents (RTA). Narakas developed his rule of “seven
seventies” based on his experience with 1068 patients:
· Approximately 70% were motor vehicle accidents
· Of the RTAs, 70% were motorcycles or bicycles
· Of the cycle riders, 70% had multiple injuries
· Of the multiple injuries in cycle riders, 70% were supraclavicular injuries
· Of the supraclavicular injuries, 70% had at least one root avulsed
· Of the avulsed roots, 70% were lower C7, C8, and T1
· Of the 70% avulsed roots, 70% of those were associated with chronic
pain.
Reference: Narakas AO. The treatment of brachial plexus injuries. Int
Orthop. 1985; 9(1):29-36
Ans: A

420
4) A 23-year-old man is brought to the Accident and Emergency
department with a gunshot injury to his right upper thigh. On
examination, the wound lies about 4 cm below the inguinal ligament.
Local neurological examination reveals numbness over the anterior
thigh and medial aspect of his leg. Although he is able to flex the hip,
he is unable to extend the knee on the affected side. The nerve likely
to be injured is:

A) Pudendal nerve
B) Sciatic nerve
C) Lateral cutaneous nerve of thigh
D) Saphenous nerve
E) Femoral nerve
Ans: See end of page
Explanation
The femoral nerve arises from the lumbar plexus (L2-4). It exits the pelvis by
passing beneath the inguinal ligament to enter the femoral triangle. The
femoral nerve innervates the iliopsoas, which helps in flexion of the hip, and
the quadriceps, which helps in extension of the knee. The motor branch to
the iliopsoas originates in the pelvis proximal to the inguinal ligament. The
sensory branch of the femoral nerve, the saphenous nerve, innervates the
skin over the medial aspect of the thigh and the anterior and medial aspects
of the calf. Motor loss includes weakness of the quadriceps muscle and
decreased patellar reflex (knee jerk). The ankle jerk is preserved, since it is
innervated by the tibial nerve [S1-S2].
Ans: E

421
5) The commonest cause of enophthalmos following a reduced
zygomatic fracture is:

A) Atrophy of orbital fat


B) Paralysis of orbital muscles
C) Entrapment of muscle
D) Increased orbital volume due to inadequate reduction
E) Paralysis of Müeller’s muscle
Ans: See end of page
Explanation
Zygoma forms a significant portion of the lateral orbital wall and orbital floor.
Fractures of zygoma therefore increase orbital volume and cause
enophthalmos. Displaced zygomatic fractures can be treated by open
reduction and fixation using miniplates. Enophthalmos is seen in three
percent cases of zygomatic fractures due to inadequate reduction. Lateral
rotation of the fracture results in posterior displacement of the globe.
Accurate reduction is therefore important in orbital fractures.
The correction of enophthalomos is based on patient’s concerns about the
cosmetic deformity. The most accurate way of correcting enophthalmos is
through osteotomy and ensuring accurate reduction. An alternative method
is to increase the orbital contents by using fat graft material in the orbital
cone. For every 1.5mL of fat graft material, 1mm movement of the globe
can be expected.
Ans: D

422
6) The commonest site of mandibular fracture is the:

A) Angle
B) Body
C) Parasymphyseal
D) Condyle
E) Coronoid process
Ans: See end of page
Explanation
Although a strong bone, the mandibula is prone to trauma due to its
prominent position and mobility. The commonest cause of mandibular
fractures is assault with road traffic accidents being the next common.
The anatomical areas of mandible are the:
· Symphysis
· Body
· Angle
· Ramus
· Coronoid process
· Condyles.
The commonest site of mandibular fracture is the body followed by angle
and the condyle. Fractures of the ramus and symphysis are relatively rare.
Coronoid process is the least common site for fractures.
Ans: B

423
7) Which of the following is true about Mangled Extremity Severity
Score?
A) The maximum possible score is 15
B) A score greater than 12 is predictive of amputation
C) Takes into account the age of the patient
D) There is no score for the haemodynamic status of the patient
E) Co-morbidities are taken into account
Ans: See end of page
Explanation

Mangled Extremity Severity Score (Helfet, 1990) is as follows:


Type and severity of Injury - Score
Skeletal / soft tissue - Low-energy gunshot - 1
- Moderate-energy open fracture - 2
- High-energy gunshot - 3
- High-energy - 4
Limb ischemia - Perfusion normal,pulse reduced - 1
- Pulseless, paresthesia - 3
- Cool, paralyzed, insensate - 4
Shock - Systolic BP > 90 - 0
- Transient hypotension - 1
- Persistent hypotension - 2
Age (years) - <30 - 0
- 30-50 - 1
- > 50 - 2
The maximum possible score is 12. A score >7 is predictive of amputation
Ans: C

424
8) The minimum number of incisions required to decompress all the
compartments in the leg are:

A) One
B) Two
C) Three
D) Four
E) Five
Ans: See end of page
Explanation
Compartment syndrome is characterised by increase in pressure in a closed
osteofascial compartment and was first described by Vogt in 1943.
There are four fascial compartments in the leg that comprises the following
muscles:
Anterior compartment: Tibialis anterior, extensor hallucis longus and
extensor digitorum longus
Lateral compartment: Peroneus longus and brevis
Superficial posterior compartment: Gastrocnemius and Soleus
Deep posterior compartment: Flexor hallucis longus, flexor digitorum longus
and tibialis posterior.
All these compartments can be decompressed through two incisions. One
incision is made 2cm posterior to the medial border of tibia and through this
the posterior two compartments (superficial and deep) are decompressed.
The second incision is made 2cm lateral to the lateral border of tibia and
through this the anterior and lateral compartments can be decompressed.
Ans: B

425
9) The commonest pathogen isolated from osteomyelitis of the lower
limb following trauma is:

A) Coagulase negative Staphylococcus


B) Coagulase positive Staphylococcus
C) Group B Streptococcus
D) Pseudomonas
E) Enterobacter
Ans: See end of page
Explanation
Osteyomyelitis is a term used to describe acute or chronic infection of the
bone. It is a recognised complication of open fractures of the tibia.
Ger (1970) identified four major causes of chronic osteomyelitis following
lower limb fractures:
· Retained necrotic or infected bone
· Avascular or infected scar
· Dead space in the surgical site
· Inadequate skin cover
Prophylactic antibiotics have a role in preventing osteomyelitis. Patients who
receive prophylactic antibiotics have 4.5% risk of developing osteomyelitis
whereas it rises to 24% in those who did not receive any antibiotics. The
commonest organism isolated is Coagulase positive Staphylococcus.
Ans: B

426
10) Which of the following is a contraindication to topical negative
pressure therapy?

A) Bacterial colonisation of the wound


B) Wounds on the face
C) Exposed tendon
D) Communication with cerebrospinal fluid
E) Advanced age
Ans: See end of page
Explanation
In 1997, Argenta and Morykwass published their results of successfully
treated wounds with topical negative pressure therapy. Since then Vacuum
Assisted Closure (VAC therapy) has been popular in management of difficult
wounds where surgical closure is difficult.
Vacuum Assisted Closure can be used as a temporizing measure or a
definitive wound closure method. It is not suitable in the following instances:
· Communication with cerebrospinal fluid (this poses a risk of coning).
· Malignancy in the wound bed
· Wounds with necrotic tissue or slough (although not an absolute
contraindication). They have to be surgically debrided prior to application of
topical negative pressure therapy
· Patients on anticoagulation or those with bleeding problems
Ans: D

427
11) Which of the following statements about gastrocnemius flaps is
true?

A) Lateral head is more useful clinically


B) The vascular pedicles come from the posterior tibial artery
C) It is a type I muscle
D) It is most useful for middle third defects of the leg
E) Commonly used a musculocutaneous flap
Ans: See end of page
Explanation
Gastrocnemius is a Type I muscle (one dominant pedicle) according to
Mathes and Nahai classification. It has a lateral and a medial head both
uniting to form the Achilles tendon. The blood supply is from the medial and
lateral sural arteries, which are branches of the popliteal artery. The nerve
supply is from posterior tibial nerve.
Gastrocnemius flap is useful as a muscle flap for covering defects over the
proximal third of tibia and knee joint. A common indication for using this flap
is to salvage an exposed knee joint prosthesis. The medial head is more
useful since the lateral head has less reach over the fibula. The lateral head
may also compress the common peroneal nerve as it winds around the neck
of fibula.
Gastrocnemius can be harvested as a musculocutaneous flap but due to the
bulk it is not common practice. It is usually taken as a muscle flap and
covered with a split skin graft.
Ans: C

428
12) Which of the following statements is true regarding compartment
syndrome?

A) There are three compartments in the leg


B) An open wound does not exclude compartment syndrome
C) The superficial and deep posterior compartment are released through an
incision 2cm lateral to subcutaneous border of tibia
D) The hand has eight compartments
E) A compartment pressure more than 20 mmHg is always an indication for
fasciotomy
Ans: See end of page
Explanation
There are four compartments in the leg (anterior, lateral, superficial
posterior and deep posterior) that can be decompressed through two
incisions; one placed 2cm posterior to the medial border of tibia to release
the two posterior compartments and the second placed 2cm lateral to the
lateral border of tibia to decompress the anterior and lateral compartments.
An open wound in the leg does not indicate released or decompressed
compartments.
There are 10 compartments in the hand: thenar, hypothenar, adductor
pollicis, four dorsal interossei, three volar interossei. These can be released
with longitudinal incisions over the dorsum of the index and ring metacarpals,
the ulnar aspect of the little finger metacarpal and the radial side of the
thumb metacarpal or in first dorsal web space.
Normal compartment pressure is: 2 – 7 mmHg. A rise >30 mmHg is
considered to be an indication for fasciotomy although a rise in pressures
measured serially is more significant. In addition, the diastolic pressure of
the patient has to be taken into consideration since a fall in diastolic
pressure may decrease the threshold for a fasciotomy.
Ans: B

429
13) Which statement is true about facial fractures?

A) The miniplates for facial fractures have bi-cortical screw fixation


B) Elevation of a zygoma using Gillies temporal lift is done through a plane
between superficial temporal fascia and deep temporal fascia
C) Le Fort I fracture is also called Guerin’s fracture
D) Le Fort II fracture does not involve pterygoid plates
E) The commonest facial bone to be fractured is the mandible
Ans: See end of page
Explanation
The basic tenets of management of facial fractures are accurate anatomical
reduction of fractures, rigid fixation and primary bone grafting of significant
defects. Plate fixation of facial fractures evolved following the work of the
Swiss AO group and was first used for mandibular fractures. The earlier
mini-plate system used for facial fractures was the same as the ones used
for hand fractures. The miniplates for facial bones used currently have uni-
cortical fixation.
In Gillies temporal lift, the elevator is passed deep to the deep temporal
fascia.
Le Fort I fracture is also called Guerin’s fracture. Le Fort II fracture runs
from the root of the nose and obliquely across the maxilla to end in the
pterygoid plates. The commonest facial bone to be fractured is the nasal
bone followed by zygoma.
Ans: C

430
14) Which of the following statements regarding tetanus is correct?

A) The average incubation period for tetanus is less than 24 hours


B) There are no contra-indications to administering tetanus toxoid
C) Tetanus prophylaxis is not required in patients with a frostbite injury
D) Passive immunization is not necessary in patients who have received two
or more tetanus toxoid injections
E) It is caused by a Gram-negative micro-organism
Ans: See end of page
Explanation
Tetanus, caused by Clostridium tetani, an obligate anaerobic gram-positive
bacillus is characterized by acute onset of hypertonia and painful muscular
contractions. This forms spores, which are resistant to heat, desiccation,
and disinfectants and can persist in normal tissue for months to years. They
produce tetanospasmin, a neurotoxin, which causes the clinical
manifestations of tetanus. The average incubation period for tetanus is 10
days, with a range of 4 – 21 days. In severe trauma cases, however,
tetanus can appear as early as 1 to 2 days.
Tetanus immunization depends on previous immunization status and the
tetanus-prone nature of the wound. Tetanus-prone wounds include wounds
older than six hours, stellate or avulsed wounds of more than 1cm depth,
wounds caused by crush, burn or frostbite injuries, those with contaminated
/ ischaemic tissue. People who have received tetanus toxoid injections more
than two or more doses in their life time do not require passive
immunization. A history of neurologic or severe hypersensitivity reaction to a
previous dose is a contraindication to administration of tetanus toxoid.
Ans: D

431
15) A 30-year-old gentleman is seen in A&E following a blunt trauma to
the face from an RTA. On examination, he has peri orbital ecchymosis
with diplopia, enopthalmos and restriction of eye movement. The most
likely diagnosis is

A) Ruptured globe
B) Inferior rectus entrapment
C) Infra orbital nerve palsy
D) Hyphaema
E) Retrobulbar haematoma
Ans: See end of page
Explanation
Inferior rectus entrapment is most commonly associated with blow out
fractures of the orbit secondary to blunt trauma. The inferior rectus muscle
gets trapped within the fracture fragments, causing restriction of movements
of the eye, diplopia and enopthalmos. CT scan is a useful investigation to
confirm the diagnosis. The other structures which may become entrapped
by the fracture include inferior oblique muscle, Lockwood’s ligament,
Tenon’s capsule, inter-muscular membrane or peri-orbital fat. This should be
treated by exploration, release of orbital contents and reconstruction of the
orbital floor. Undue delay may result in fibrosis of the muscles and
permanent restriction in eye movements.
Ans: B

432
16) Acute orbital compartment syndrome

A) can present with enophathlmos


B) should be investigated using an urgent MRI before surgical exploration
C) should be decompressed in less than 2 hours
D) can be decompressed by dividing orbital septum
E) can be decompressed using endoscopic division of the orbital floor
Ans: See end of page
Explanation
Acute orbital compartment syndrome arises as a result of an increase in
pressure within the orbit due to trauma, surgery (retrobulbar haematoma
following blepharoplasty), or rarely due to bleeding from venous
malformations or aneurysms of the ophthalmic artery. The patient may
present with pain, diplopia, restricted ocular movements, proptosis,
chemosis, peri-orbital bruising and raised intra-orbital pressure.
This is a surgical emergency since irreversible visual damage can occur
within 90 to 120 minutes. Surgical decompression can be carried out by
lateral canthotomy and inferior cantholysis.
Ans: C

433
Trauma

Extended Matching Questions

1) REGARDING NERVE INJURIES TO THE LOWER LIMB


Options
A) Common peroneal nerve
B) Saphenous nerve
C) Medial plantar nerve
D) Pudendal nerve
E) Femoral nerve
F) Sural nerve
G) Lateral plantar nerve
H) Tibial nerve
From the options above select the likely injured nerve for the clinical
scenario described below. Each option can be used once, more than once
or not at all.
Questions
1. A 21-year-old man has sustained a closed fracture of the fibular neck. He
is unable to dorsiflex or evert his left foot and also complains of decreased
sensation over the dorsum of his foot.
2. A 27-year-old man has a deep stab wound injury to his left upper thigh.
He presents with numbness over the anterior thigh and medial aspect of his
leg. He is unable to extent his knee and the knee jerk is diminished.
3. A 31-year-old man sustained a posterior dislocation of the knee following
a road traffic accident. He is unable to flex the toes or invert the ankle on the
right side. Sensation is lost over the sole of the foot. Ankle jerk is lost on the
right side.

434
Answers

1-A
2-E
3-H
Explanation
Injury to the common peroneal nerve may occur following fracture of the
neck of fibula, since the nerve winds down the neck and is relatively
superficial at this point. The common peroneal nerve can also be injured
following a trauma or injury to the knee, use of tight plaster casts, and
pressure to the fibular neck region from positions during deep sleep or
coma. Common peroneal nerve gives motor supply to the dorsiflexor and
evertor muscles of the ankle and toes. Its sensory branches supply the
anterior and lateral aspect of the leg and whole of the dorsum of the foot
and toes except the lateral aspect of the foot (supplied by the sural nerve).
Trauma in the region of the femoral triangle may result in injury to the
femoral nerve. It innervates the iliopsoas (a hip flexor) and the quadriceps
muscle (a knee extensor). The motor branch to the iliopsoas originates in
the pelvis proximal to the inguinal ligament and injury at or above this level
leads to loss of hip flexion. The sensory branch of the femoral nerve, the
saphenous nerve, innervates skin of the medial thigh and the anterior and
medial aspects of the calf. Damage to the femoral nerve causes weakness
of the quadriceps muscle and decreased patellar reflex.
The posterior tibial nerve may be damaged by posterior dislocation of the
knee, posterior displaced fracture of the tibia and fractures around the knee
joint. Tibial nerve supplies the flexor compartments of the leg. It divides into
medial and lateral plantar branches to supply the intrinsic muscles of the foot
and provides sensation to the plantar surface of the foot. It also provides
cutaneous and articular branches to the medial side of the ankle and foot.

435
2) FACIAL FRACTURES

Options
A) Le Fort I
B) Le Fort II
C) Le Fort III
D) Zygomatic fractures
E) Nasoethmoid fractures
F) Mandibular fractures
G) Frontal bone fractures
H) Orbital blow out fractures
From the list above, select the most correct answer. Each option can be
used once, more than once or not at all.
Questions
1. Guerin’s fracture is another term used for:
2. Craniofacial dysjunction is seen in:
3. ‘Favourable’ and ‘Unfavourbale’ patterns are described in relation to
fractures of:

436
Answers

1-A
2-C
3-F
Explanation
René Le Fort’s studies on cadavers made landmark discovery about the
fracture patterns on the face based on points of weakness.
He described three patterns:
Le Fort I: This fracture is also called Guerin’s fracture. The fracture-line
runs from anterior nasal spine across the maxilla transversely to the
pterygoid plates.
Le Fort II: This fracture is also called pyramidal fracture and the fracture-
line runs obliquely from the root of the nose, through the orbital floor and
maxilla to pterygoid plates.
Le Fort III: This fracture is also called craniofacial dysfunction. The fracture-
line runs from the root of the nose, across the floor of the orbit and lateral
orbital wall. The zygomatic arch is fractured as well, the facial skeleton is
separated from the cranial part.
‘Favourable’ and ‘Unfavourable’ are terms used to describe mandibular
fractures. They can be favourable in the vertical plane or horizontal ane.
Fractures which are favourable in the vertical plane are where the fracture-
line runs anteriorly and downwards from the superior border. Here, the
masseter and temporalis muscles pull the posterior fragment inferiorly
preventing displacement of the fracture.
The horizontally favourable fractures run anteriorly and outwards. Here the
pull of the medial pterygoids prevent displacement of the fracture.

437
3) RECONSTRUCTIVE TECHNIQUES FOR LOWER LIMB

Options
A) Soleus flap
B) Gastrocnemius flap
C) Proximally based fasciocutaneous flap
D) Split skin graft
E) Free gracilis flap
F) Free latissiumus dorsi flap with split skin graft
G) Peroneus tertius flap
H) Sural artery flap
From the list above, select the most appropriate soft tissue reconstruction
for the description below. Each option can be used once, more than once
or not at all.
Questions
1. The flap of choice for a wound 5 cm in diameter with exposed bone over
the lower third of the tibia
2. A wound over knee joint with exposed prosthesis following Total Knee
Replacement
3.A total avulsion of the sole of the foot with loss of soft tissue

438
Answers

1-E
2-B
3-F
Explanation
Lower third of the leg is an area that is difficult to cover using local flaps.
The commonly used reconstructive methods for this area are distally based
fasciocutaneous flaps, propeller type perforator flaps and free flaps.
Amongst the free flap options, for a 5 cm defect, a free gracilis muscle flap
is a good choice. It is a type II muscle which has its main supply from the
medial circumflex femoral artery. It has a reliable pedicle which is about 8cm
long, and the donor mobidity is not significant. A latissimus dorsi muscle flap
can also be used for such a defect but it has more donor site morbidity and
also involves turning the patient for harvesting the flap.
Gastrocnemius flaps are the workhorse flaps for covering defects over the
knee. Gastrocnemius muscle has a lateral head and a medial head, which
unite to form the Achilles tendon. The lateral head is supplied by lateral sural
artery and the medial head by medial sural artery, both branches of the
popliteal artery. Both the lateral and medial heads can be used as pedicled
muscle flaps for defects around the knee and proximal leg. Medial head is
preferred since the lateral head has to wind around the fibula and therefore
has less reach. There is also a risk of compressing the common peroneal
nerve with the lateral gastrocnemius flap.
Avulsion injuries to sole of foot are challenging to manage. If reconstruction
is undertaken it will require a large free flap such as latissimus dorsi covered
with a split skin graft.

439
Miscellaneous
Single Best Answers
1) Nikolsky sign refers to
A) Extensive petechial rash in Waterhouse-Friederichsen syndrome
B) Prodromal ulceration in Fournier’s gangrene
C) Crepitus of the skin in necrotizing fasciitis
D) Epidermolysis with digital pressure on the skin affected by drug-induced
exfoliative disorders
E) Epidermolysis bullosa
Ans: See end of page
Explanation
Nikolsky sign refers to immediate epidermolysis with lateral digital pressure
on the skin affected by drug-induced exfoliative disorders such as toxic
epidermal necrolysis (TEN) and Steven-Johnson syndrome (SJS). Both
these syndromes present with large areas of purpuric macules.
Waterhouse-Friederichsen syndrome (fulminant meningococcaemia) is
characterised by skin necrosis in areas of the prodromal petechial rash,
mental changes and haemodynamic collapse. TEN and SJS have a drug-
induced aetiology. Prodromal symptoms of TEN are influenza-like followed
by the cutaneous eruption. SJS is featured by widespread and central
confluence of lesions with epidermal detachment, severe stomatitis and
conjunctivitis. Epidermolysis bullosa is characterised by massive blisters
developing in response to minor trauma and is due to an underlying genetic
defect coding for keratins.
Ans: D

440
3) Which of the following statements regarding lymphoedema is true?

A) Secondary lymphedema is due to an abnormal development of the


lymphatic system
B) Milroy’s disease typically affects the lower extremities of young boys
C) Filariasis is a form of primary lymphoedema affecting the lower
extremities
D) Lymphoedema praecox usually affects isolated lower limb
E) Lymphoedema praecox is more common in young boys than girls
Ans: See end of page
Explanation
Lymphoedema can be classified as primary and secondary.
Congenital lymphoedema is a primary form and is often noted at birth or
soon after. Multiple limbs, genitalia and face may be involved. Bilateral lower
limb oedema is the most common presentation.
Milroy’s disease is an autosomal dominant form of hereditary lymphoedema
both congenital lymphoedema and lymphoedema praecox with a female
preponderance.
Lymphoedema praecox is usually unilateral and limited to the foot and the
calf.
Ans: D

441
4) Which parasite is associated with filariasis?

A) Schistosoma mansoni
B) Wuchereria bancrofti
C) Trypanosoma brucei
D) Echinococcus species
E) Clonorchis sinensis
Ans: See end of page
Explanation
There are several causes of secondary lymphoedema but filariasis due to
parasitic infection is the commonest cause worldwide. In Western countries,
however, damage to the lymphatic system by surgery, radiation or tumour
invasion is the commonest cause. There are two main parasites causing
filariasis: Wuchereria bancrofti and Brugia malayi. Wuchereria bancrofti is a
nematode worm spread by a mosquito vector. It affects about 120 million
people worldwide, primarily in Africa and other (sub) tropical countries. After
infection, the worms reside in the lymphatic channels in the lower limbs and
disrupt the lymphatic flow causing lymphoedema. In early stages, filariasis
can be successfully treated with Diethyl Carbamazine. However, in
established cases of filariasis with skin changes, even surgical excision fails
to obtain satisfactory results.
Ans: B

442
5) Buried dermal flap to treat lymphoedema was described by:

A) Treves
B) Homans
C) Charles
D) Handley
E) Thompson
Ans: See end of page
Explanation
In 1912, RH Charles described radical excision of lymphedematous tissue
from the scrotum and in his series made a mention about treatment of
lymphoedema of the leg. Though he never described radical excision of
lmyphoedematous tissue and split skin grafting in the lower limb, this
procedure eponymously bears his name. Complications can be high including
poor cosmesis but Charles’ procedure may be an option for patients with
severe oedema and skin changes.
Staged subcutaneous excision beneath flaps was described by Sistrunk and
later popularised by Homans.
Lymphangioplasty as described by Handley unfortunately was proven to be
ineffective with a high risk of severe complications.
Thompson used a buried dermal flap, tunnelled through the fascia into a
muscular compartment, in an attempt to improve lymphatic drainage.
The Stewart-Treves syndrome is the occurrence of lymphangiosarcoma in
chronic lymphoedema following mastectomy.
Ans: E

443
6) Which of the following statements regarding lymphoedema is
INCORRECT?

A) Lymphoedema is limited to tissues superficial to the deep fascia


B) Muscles do not have an established lymphatic drainage system
C) Embryologically, lymphatics develop from the arterial system
D) About 50% of the body’s albumin is processed daily via the lymphatic
system
E) In secondary lymphoedema, 30% of the cases are caused by
lipodystrophy
Ans: See end of page
Explanation
Lymphatics are located in the superficial tissues and therefore
lymphoedema is limited to tissues above the deep fascia. Muscles do not
have an established lymphatic system. Embryologically, lymphatics develop
from the venous system. About 50% of the body’s albumin is proce ssed
daily in the lymphatic system.
Lymphoedema can be classified as primary or secondary. Primary
lymphoedema is due to hypoplasia of lymphatics that manifest in childhood
(lymphoedema congenita), adolescence (lymphoedema precox) or in later
life (lymphoedema tarda). Causes of secondary lymphoedema include:
obstruction or disruption in the lymphatic system by tumour invasion,
infections, irradiation or iatrogenic causes.
The clinical picture of lymphoedema should not be confused with
lipodystrophy since they are distinct entities. Lipodystrophy is a disorder of
adipose tissue characterized by selective loss of fat from different parts of
the body. It can be classified as (i) Congential [e.g., Congenital generalized
lipodystrophy (Beradinelli-Seip syndrome) or Familial partial lipodystrophy]
and (ii) Acquired [e.g., Barraquer-Simons syndrome; Centrifugal abdominal
lipodystrophy; Localized lipodystrophy; and HIV-associated lipodystrophy].
Ans: E

444
7) Which amongst the following statements is true regarding treatment
of leg ulcers?

A) Class III compression stockings provide about 40 to 50 mmHg of


pressure at the ankle
B) Diuretics have no role in the treatment of leg ulcers
C) Occlusive dressings are contra-indicated
D) Flaps should not be never used in the treatment of venous ulcers
E) The commonest location of a venous ulcer is over the lateral malleolus
Ans: See end of page
Explanation
Compression therapy plays a significant role in the management of chronic
venous insufficiency and venous ulcers. Graded compression bandages
come in four classes depending on the pressure. Class III (40-50 mmHg)
and Class IV (> 60mmHg) stockings are recommended for severe oedema
and chronic venous insufficiency. Diuretics, used in conjunction with
compression therapy, may assist in fluid mobilization.
Occlusive dressings are less painful to the patient and provide a moist
environment promoting a more rapid wound healing.
Surgical intervention for venous ulcers is indicated in intractable pain or
failure of non-operative treatment. In exceptional circumstances,
fasciocutaneous or free flaps can be used to reconstruct defects. The
commonest location of venous ulcers is over the medial malleolus.
Ans: A

445
8) Pressure ulcers:

A. may develop in up to 10% of hospitalised patients


B. most frequently occur over the malleoli
C. are staged by the size of the wound
D. can be predicted using Water’s scale
E. in elderly patients should always be treated conservatively
Ans: See end of page
Explanation
In general, about 10% of patients in hospital develop pressure ulcers. They
most frequently occur around the pelvic girdle (up to 75%).
The Waterlow scale is often used to score the risk of developing pressure
ulcers and include patient’s weight, skin type, continence, mobility, sex and
age. Additional points are given for specific risk factors such as poor
nutrition, sensory disturbances, smoking and previous orthopaedic surgery
or fractures below the waist.
Pressure ulcers are classified into four grades (or stages) depending on the
depth (not size) of the ulcer.
Well-motivated, young patients with stable clinical conditions are the best
candidates for surgery, whilst patients who are expected to regain their
mobility are generally treated conservatively as their pressure ulcers will
improve once pressure is relieved.
Ans: A

446
9) Which amongst the following is correct regarding pressure ulcers?

A) In Grade III pressure ulcers, the ulcer involves the tendon or bone
B) The initial pathologic changes occur in the muscle overlying the bone
before involvement of the skin
C) The mainstay of treatment in pressure ulcers is prevention of infection
D) A parabolic relationship exists between time and pressure in the
development of pressure ulcers
E) Patients with a Waterlow score of 6 have a very high risk of developing a
pressure ulcer
Ans: See end of page
Explanation
Pressure ulcers are staged as follows:
Stage I: Non-blanchable erythema without breach of the epidermis
Stage II: Partial skin loss involving the epidermis and dermis
Stage III: Full-thickness skin loss extending into the subcutaneous tissue but
without breach of the underlying fascia
Stage IV: Ulceration through the underlying fascia with extensive deep
destruction where bone, muscle, joint or tendon may be involved.
Studies have demonstrated an inverse parabolic relationship between
pressure and time. It has also been proven that initial pathologic changes
occur in the muscle, followed by the more superficial soft tissue and skin
(inverted cone). The mainstay of treatment is therefore relieving the source
of pressure.
Patients with a Waterlow score of above 10 have a risk of developing a
pressure ulcer.
Ans: B

447
10) Which amongst the following considerations are correct in the
treatment of pressure ulcers?

A) Radical ostectomy is important in preventing pressure ulcers


B) A tensor fasciae latae flap is easily performed with excellent reliability
and is a preferred method to reconstruct sacral ulcers
C) Free flaps have no role in the treatment of pressure ulcers
D) Release of muscle insertions to relieve spastic hip contractures are
important, particularly in active wheelchair-bound patients
E) In grade IV pressure ulcers, a bone scan may be positive even in the
absence of osteomyelitis
Ans: See end of page
Explanation
A normal bone scan excludes osteomyelitis, but patients with grade IV
pressure ulcers can have positive findings on a bone scan due to the
inflammation of the bone (osteitis) at the base of the wound. Thus, the single
most useful test to confirm osteomyelitis is a bone biopsy.
Tensor fascia latae flap, based on the transverse branch of the lateral
circumflex femoral artery, is mainly useful to reconstruct defects of pressure
ulcers in the trochanteric region.
Removal of bony prominences are important in the surgical management of
pressure ulcers, but radical ostectomy should be avoided to avoid skeletal
instability, excessive bleeding and redistribution of pressure points to
adjacent areas. Spasticity is common in spinal cord injuries and contributes
to the development of pressure ulcers. However, in active wheelchair-bound
patients, releasing the the hip contracture can contribute to a flail extremity
interfering with transfers.
In extreme cases of multiple pressure ulcers or multiple previous
procedures, free tissue transfer may be indicated to close the wound.
Ans: E

448
11) Which amongst the following statements is true regarding surgical
management of pressure ulcers?

A) Primary closure is preferred when adequate local tissue is available


B) skin grafting has a success rate of approximately 50%
C) Fasciocutaneous flaps are preferred in large ulcers
D) The most commonly used musculocutaneous flap is the biceps femoris
flap
E) Bilateral radical ostectomies of the ischium can result in perineal pressure
ulcers
Ans: See end of page
Explanation
In the surgical planning of pressure ulcers, one must consider not only the
present surgery, but also the need for subsequent surgical procedures.
Since there is deficiency of tissue in pressure ulcers, primary closure leads
to tension, dehiscence and a scar over the original bony prominence;
primary closure should be avoided.
Skin grafts tend to provide unstable coverage and are not advocated.
Fasciocutaneous and musculocutaneous flaps provide excellent blood supply
and durable coverage, but fasciocutanous flaps offer limited bulk for the
treatment of large ulcers.
The most commonly used musculocutaneous flaps are based on the gluteus
maximus muscle. Wherever possible, excision of the ischium should be
limited as radical ostectomies transfer the pressure onto other adjacent
areas.
Ans: E

449
12) The flap described by Sushrutha for nasal reconstruction was the:
A) Forehead flap
B) Cheek flap
C) Radial forearm flap
D) Medial arm flap
E) Lateral arm flap
Ans: See end of page
Explanation
Sushrutha, the ancient Indian Plastic Surgeon, is regarded as the pioneer in
nasal reconstruction. He has championed several surgical operations and
also described the cheek flap for nasal reconstruction. The famous Sushruta
Samhita was written by him and describes several surgical procedures. The
Indian forehead flap though often attributed to Sushruta, was, however, not
described by him. The forehead flap technique was used by the Hakkims
(traditional physicians) in Himachal Pradesh, a state in Northern India. The
Indian forehead flap rhinoplasty technique was brought to the Western world
by the British during the colonial era. Joseph Constantine Cartue (18th
century) spent many years in India learning various rhinoplasty techniques.
Ans: B

450
13) Which of the following statements regarding the World Health
Organization surgical safety checklist is correct?

A) The checklist is to be done by one senior health care professional of the


surgical team
B) Intra – operative venous thrombo-embolism prophylaxis is not an item on
the checklist
C) The checklist should be done once the patient has been anaesthetized
D) Display of essential imaging is an item on the checklist
E) The checklist is only used to confirm patient’s identity and marking of
surgical site
Ans: See end of page
Explanation
In industrialised countries, major complications have been reported to occur
in 3–16% of inpatient surgical procedures, with permanent disability or death
rates in approximately 0.4–0.8%. In June 2008, the World Health
Organization launched the Global Patient Safety Challenge (‘Safe Surgery
Saves Life’) to reduce the number of surgical deaths across the world.
A set of safety checks (WHO surgical safety checklist) is designed to be
used in any operating theatre environment. This involves three phases
(before induction of anaesthesia, before start of surgical intervention and at
the end of operation) by three members of the surgical team, including the
anaesthetist, surgeon and a registered practitioner (nurse). Verbal
confirmation is undertaken regarding patient’s identity, surgical site, risk
factors (such as allergy, blood loss, airway difficulties, surgical site infection,
venous thrombo-embolism prophylaxis), anticipated critical events,
instrumentation and equipment concerns, and essential imaging display.
Ans: D

451
14) Which of the following statements regarding body mass index
(BMI) is correct?

A. It is calculated by dividing height by weight


B. The unit for measuring BMI is kg/cm2
C. It is a measure of body fat against muscle mass
D. The upper limit of the normal range is 28
E. The lower limit of the normal range is 18
Ans: See end of page
Explanation
Body mass index (BMI) is a measure of body fat based on the height and
weight that applies both to adult men and women.
It is calculated by dividing the body weight (in kg) by the square of the height
(in m). Hence the unit is Kg/m2.
· Normal limits is 18 – 25
· 25 - 30 is considered overweight
· 30 – 40 is considered obese and
· Greater than 40 is morbidly obese
A high BMI is associated with increased complications, in particular in
reduction mammaplasty, abdominoplasty and TRAM flaps.
Ans: E

452
15) Which amongst the following statements regarding Positron
Emission Tomography (PET scan) is CORRECT?

A) The tracer is highly specific for cancer cells


B) Inflammation and neoplasia can easily be differentiated
C) PET combined with CT scans provide excellent soft tissue resolution
D) The role of PET scans is in the assessment of the unknown primary
E) PET scans are able to define tumour deposits of only a few cells
Ans: See end of page
Explanation
Positron Emission Tomography is a noninvasive nuclear medicine imaging
technique that measures the metabolic activity of cells (thus it is a metabolic
imaging modality looking at function). PET-FDG scans use FDG (fluoro-
deoxyglucose – a glucose molecule tagged with a small amount of
radioactive element), which is concentrated in cells exhibiting high rates of
glycolysis. The tracer-uptake is therefore high in neoplastic cells as well as
inflammatory cells. When used in conjunction with CT-scans, PET scan is
very effective in detection of cancer or its spread. A significant tumour load
more than a few cells is required to define tumour deposits. Although
anatomical localization of tumours has improved in combination with CT-
scans, efforts to enhance the resolution of soft tissues is currently ongoing.
Ans: D

453
16) Metoidioplasty

A) involves reconstruction of the penis using free tissue transfer


B) involves mobilization of the clitoris to approximate the position of penis
C) enables penetrative sexual intercourse
D) does not retain genital orgasm
E) is a more complex procedure as compared to phalloplasty
Ans: See end of page
Explanation
Metoidioplasty (‘meta’ - towards, ‘oidio’ - male genitals and ‘plasty’ – to
mould) is used as an alternative to phalloplasty in transgender patients with
an enlarged clitoris following testosterone treatment. In this procedure, the
clitoral tissue is mobilized and moved towards a position which approximates
that of penis. It is a simpler procedure than phalloplasty, which involves
reconstruction of the penis using various tissue transfer techniques. Also,
this procedure has the advantage of retaining genital orgasm unlike a
phalloplasty. However, the penis created is small in size and therefore not
capable of penetrative sexual intercourse.
Ans: B

454
17) The dye used in tattoo that most commonly causes skin reaction
is the

A) Blue cobalt
B) Yellow Cadmium
C) Red Cinnabar
D) Green Chromium
E) Black Henna
Ans: See end of page
Explanation
Although the dyes used in tattoo are relatively inert, hypersensitivity
reactions can sometimes occur. Anaphylactic reactions however are rare.
The commonest dye that has been reported to cause reaction is the red dye
derived from mercuric sufide (Cinnabar). The most common type of reaction
noticed is the eczematous hypersensitivity reaction due to contact dermatitis
and photo allergenic reaction. They can rarely cause exfoliative dermatitis.
Ans: C

455
18) At 2 am, your junior rings to discuss a 31-yer-old lady who
underwent bilateral breast reduction for ‘G-cup’ breasts in the
morning list the previous day. Your colleague is concerned that she
has developed haematomas bilaterally. Her Hb taken 30 mins ago is
reported as 6.5 gm/dl (pre-op Hb was 10.9). She feels dizzy, with a
heart rate of 108 beats/min and a blood pressure of 98/70 mm Hg. You
examine her and reckon that she is bleeding actively from both sides
and arrange for theatre. In the interim, you request her to be
transfused with 2 units of blood. However, she states that she is
Jehovah’s Witness and refuses a blood transfusion.

What do you think would be the best treatment option at this


instance?
A) Prescribe ’O’ negative blood
B) Prescribe gelofusine
C) Prescribe platelets
D) Prescribe factor VIII
E) Prescribe epoetins
Ans:See end of page
Explanation
Jehovah’s witnesses do not accept blood or individual blood cellular
products, including platelets and factor VIII. (In this case scenario, in
addition, platelets and factor VIII do not have any role in restoring blood
volume). However, they do not oppose other forms of medical treatment,
such as non-blood-volume expanders and blood-conservation surgical
techniques.
In this scenario, the patient is actively bleeding, is symptomatic and
manifesting early evidence of hypovolemic shock. In addition to evacuating
the haematoma and controlling the bleeding in theatre, she needs a volume
expander such as Gelofusine (a plasma substitute based on 40g (4%)
succinylated gelatin with a molecular weight of 30,000, 154mmol/l sodium
and 120mmol/l chloride). Macromolecular substances are metabolized
slowly and may used to expand and maintain blood volume as an immediate
short-term measure.

456
Epoetins, recombinant human erythropoietin, is used to treat erythropoietin
deficiency in chronic renal failure, preterm neonates of low birth weight or
patients with symptomatic anaemia associated with cancer receiving
chemotherapy. It is not appropriate in this situation.
Ans:B

457
Miscellaneous
Extended Matching Questions
1) AETIOLOGY OF ULCERS
Options
A) Venous ulcer
B) Marjolin’s ulcer
C) Neuropathic ulcer
D) Pyoderma gangrenosum
E) Ischaemic ulcer
F) Basal cell carcinoma
G) Squamous cell carcinoma
H) Sickle cell ulcer
For each of the case below, select the most likely aetiology from the list
above. Each option may be used once, more than once or not at all.
Questions
1. A 65-year-old man with Type I diabetis Melitus with a non-helaing ulcer
over the heel. He has palpable foot pulses, but sensations are impaired over
the sole of foot.
2. A 70-year-old woman with bilateral pedal oedema and a 3cm sized ulcer
with sloping edges over the medial malleolus. She has normal sensation and
distal foot pulsations.
3. A 54-year-old woman presents with a non-healing ulcer over the pretibial
region of one-year duration. This was a skin-grafted area for a burn
sustained in her childhood. It had healed initially but in the last few years has
recurrently broken down and now the area is getting larger. On examination,
there is a 2cm diameter ulcer with everted edges.

458
Answers

1-C
2-A
3-B
Explanation
There are several causes for leg ulcers and it is important to diagnose the
underlying causes while treating these patients. The etiology of leg ulcers
can be remembered using the pneumonic VITAMIN:
Venous
Infectious
Traumatic
Arterial
Metabolic
Inflammatory
Neoplastic and Neuropathic
Deep ulcers over pressure areas in the absence of ischaemia are likely due
to diabetic neuropathy. Venous ulcers develop in patients with chronic
venous insufficiency and characteristically seen over the medial gaiter area.
Marjolin’s ulcers are SCCs arising in previously traumatized locations such
as an old burn scar or a non-healing wound. These lesions have a high risk
of metastases (30-40%) and an overall poor prognosis. The recurrence rate
reported in most series range from 20-50%

459
2) INNERVATION OF LOWER LIMB
Options
A) Femoral nerve
B) Obturator nerve
C) Saphenous nerve
D) Tibial nerve
E) Sural nerve
F) Superficial peroneal nerve
G) Deep peroneal nerve
H) Sciatic nerve
For each of the muscles below, select the nerve that supplies it. Each
option may be used once, more than once or not at all.
Questions
1. Tibialis anterior
2. Gracilis
3. Vastus lateralis

460
Answers

1-G
2-B
3-A
Explanation
Sciatic nerve divides into common peroneal nerve and posterior tibial nerve
in the superior part of the popliteal fossa. Common peroneal divides into
superifical peroneal nerve and the deep peroneal nerves. Also known as the
anterior tibial nerve, the deep peroneal nerve supplies the muscles of the
anterior compartment of the leg. These include extensor digitorum longus,
extensor hallucis longus and tibialis anterior. It continues as a sensory nerve
and supplies the dorsum of the first web space of the foot (first interdigital
cleft).
The superficial peroneal nerve supplies the peroneal compartment muscles
(peroneus longus and peroneus brevis) and also carries sensations from the
dorsum of the foot (apart from first web space). Posterior tibial nerve
supplies the muscles of the superficial and deep posterior compartment
(gastrocnemius, soleus, flexor hallucis longus, flexor digitorum longus and
tibialis posterior) and continues onto the sole of the foot. It divides into
medial and lateral plantar nerves which supplies the small muscles of the
foot as well as carries sensations from the sole of the foot.
The obturator nerve supplies the muscles of the medial compartment of the
thigh. The femoral nerve supplies the quadriceps femoris, the rectus and
three vastus muscles. It continues as the saphenous nerve (sensory) along
the medial aspect of the thigh and leg and carries sensation from the medial
aspect of the leg, ankle and foot. It runs in close proximity to the long
saphenous vein.

461
3)REPORTS AND INQUIRIES IN THE NHS:

Options:

A) Lord Darzi ‘s review


B) Sir John Tooke’s report
C) Professor Sir John Temple report
D) Bristol Royal Infirmary Inquiry
E) Dame Fiona Caldicott report
F) Health and Social care bill
G) Mid Staffordshire Inquiry
H) Royal Liverpool Children’s Inquiry
For the questions given below, select the most appropriate answer from the
list of options above. Each option may be used once, more than once or
not at all.
Questions:

1. Introduced restriction for Plastic Surgical units to store skin graft


harvested from patients for delayed application
2. Recommended Consultants to have more involvement in out-of-hours care
and placed more emphasis on Consultant-led service
3. Recommends doctors in training to be given a representation in the trust
management to improve the understanding of the NHS system

462
Answer

1-H
2-C
3–B

Explanation:

The Royal Liverpool Children’s Inquiry, also popularly known as the Alder
hey Inquiry, was a result of patients’ tissue being stored unauthorized
without their consent in Alder Hey hospital between 1988 and 1995. This
resulted in a public inquiry and subsequent passage of the new Human
Tissue Act in 2004 that introduced strict regulations in the way human
tissues are handled in the UK hospitals. The whole process was overseen
by Human Tissue Authority.
Sir John Temple report was published as a result of the introduction of
European Working Time Directive. The recommendations included an
emphasis towards Consultant-led care in the NHS with less reliance on the
trainees to provide out-of-hours emergency care.
Sir John Tooke report was published to review the changes to Medical
training introduced as a result of Modernizing Medical Careers (MMC). It
made recommendations for a clear structure for medical training. It
suggested PMETB to be merged with GMC, which will be in charge of
accreditation and registration. The report also emphasized for trainees being
given a representation in the trust management to enhance their
understanding of the NHS system.

463
4)REGARDING TRAINING IN PLASTIC SURGERY IN THE UK:

Options:

A) ISB
B) PMETB
C) SAC
D) JCHST
E) ISCP
F) BAPRAS
G) GMC
H) Deanery
For the questions given below, select the most appropriate answer from the
list of options above. Each option may be used once, more than once or
not at all.
Questions:

1. The body/organization responsible for advising the Royal College of


Surgeons in all matters related to surgical training.
2. This body/organization responsible for assessing doctors who did not
undergo a formal training programme but can be considered to enter the
Specialist Register
3. The body/organization that is responsible for the management and
delivery of the post graduate medical and dental training in a region.

464
Answer

1-C
2-G
3–H

Explanation:

The Specialist Advisory Committee comprises of intercollegiate bodies for


each specialty. They are responsible for designing the curriculum for higher
specialty training, setting the requirements for training and also advice the
Royal Colleges on the issues related to the training. They also set up
requirements for completion of specialty training based on which the JCST
signs a trainee for completion of specialty training (CCT).
Although the assessment of doctors who did not undergo formal training to
enter specialist register and also to maintain the standards of specialty
training used to be the responsibility of PMETB, from 1st April 2010, this role
has been taken over by the GMC. Along with providing the doctors in with a
license to practice and also to regulate the medical practice, the GMC has
also taken over the role of assessing the CCT and CESR (Certificate of
Eligibility for Specialist Registration).
The deaneries are responsible for the frontline delivery of training in a
specific region. They manage and deliver the postgraduate medical
education of all the medical and dental specialties. They are also
responsible for analyzing each training post to provide necessary opportunity
for the trainees. In addition, they also have a role in educating the trainers
including the educational supervisors.

465
Bibliography
To ensure strict factual accuracy, the explanations in this book have been compiled after extensive
study of the relevant material from the following sources. Being an exam-preparation resource rather
than textbook, certain parts of the text in the explanation might resemble the contents in the sources
below. This is intentional and to ensure that the explanations do not deviate from the established facts
found in the thoroughly researched contents of these time-honoured textbooks. This also avoids
inadvertent subjective opinion of the authors. The authors gratefully acknowledge the sources on which
this exam-preparation resource is based.

· Atlas of Microsurgical Composite Tissue Transplantation. D Serafin


(author). W.B. Saunders Company, USA, 1996; ISBN 9780721628707
· Experimental and Clinical Reconstructive Microsurgery. S Tamai (Ed-
in-Chief). M Usui and T Yoshizu (eds). Springer Verlag, Tokyo, 2003; ISBN-
13: 978-4431703181
· Flaps and Reconstructive Surgery. FC Wei and S Mardini (Eds).
Saunders Elsevier, 2009; ISBN-13: 978-0721605197
· Grabb and Smith’s Plastic Surgery. CH Thorne, SP Bartlett, RW
Beasley, SJ Aston, GC Gurtner and SL Spear (Editors). Lippincott Williams
and Wilkins, 2006, 6th edition; ISBN: 978-0781746984
· Green’s Operative Hand Surgery. SW Wolfe, RN Hotchkiss, WC
Pederson, SH Kozin (Authors) Churchill Livingstone, 2010, 6th edition, Vol 1-
2; ISBN: 978-1416052791
· Key topics in clinical research. FG Smith and JE Smith (editors). BIOS
Scientific Publishers, Oxford, 2003; ISBN-13: 978-1859960288
· Last’s Anatomy: Regional and Applied (MRCS Study Guides). CS
Sinnatamby (Editor). Churchill Livingstone, 10th edition. ISBN-13: 978-
0443056116

· Medical Ethics and Law: The Core Curriculum. T Hope, J Savulescu


and J Hendrick (authors). Elsevier Health Sciences, 2003; ISBN
9780443062551
· Medical Law and Ethics. J Herring (author). Oxford University Press,
2010, 3rd edition. ISBN 9780199576487
· Medical Statistics: A guide to data analysis and critical appraisal. J Peat

466
and B Barton (authors). BMJ Books, Blackwell Publishing, 2007. ISBN 978-
0-7279-1812-3
· Medical Statistics at a Glance. A Petrie and C Sabin (authors). Wiley
Blackwell, 3rd edition, 2009; ISBN-13: 978-1405180511
· Michigan Manual of Plastic Surgery. DL Brown and GH Borschel
(Editors); Lippincott Williams and Wilkins, Jun 2004; ISBN-13: 978-
0781751896
· Plastic Surgery. SJ Mathes and VR Hentz (Editors). Volume 1-8.
Saunders 2005
· Plastic Surgery: Indications, Operations, Outcomes. M Achauer, E
Eriksson, B Guyuron, JJ Coleman III, RC Russell. Philadelphia, Pa, Mosby,
2000. ISBN: 0-8151-0984-9.
· Plastic and Reconstructive Surgery Board Review: Pearls of Wisdom.
SJ Lin and JB Hijjawi (Authors). BMP publishers, 2nd edition, 2006. ISBN-
13: 978-0071464475
· Selected Readings in Plastic Surgery. http://www.srps.org/

467
462

468

You might also like